WBUHS (2008-2020) rd 3 Prof. M.B.B.S, Part - II

Paper I –

Answers of Surgery Question Papers for M.B.B.S Students

Dr. Arkaprovo Roy ASSOCIATE PROFESSOR DEPARTMENT OF GENERAL SURGERY MEDICAL COLLEGE, KOLKATA The West Bengal University of Health Sciences

M.B.B.S. 3rd Professional Part – II Examination, 2020

Subject: Surgery Time: 21/2 hrs.

Paper: I Marks: 60

Group – A 1. What are Hospital Acquired Infections (HAI) and Surgical Site Infections (SSI)? Define Bacterimia and Systemic Inflammatory Response Syndrome. How will you prevent infections (broad outline only). 21/2+21/2+21/2+21/2+5

Group – B 2. a) A middle aged gentleman presents with profuse haematemesis following analgesic intake. How will you investigate and manage this patient? What are the complications of chronic peptic ulcer? 5+5+5

Or

b) A lactating woman presents to emergency with painful lump in the right breast which is associated with fever. Write down the clinical examination, investigations and treatment of this patient. 5+5+5

Group – C 3. Answer in brief on any three of the following: 3x5 a) Parotid fistula b) Cold abscess c) Volvulus d) Idiopathic Thrombocytopenic Purpura (ITP) e) Acute necrotizing pancreatitis.

Group – D

4. Write short notes (any three): 3x5 a) Sequestrum b) Pott’s paraplegia c) Tennis elbow d) Avascular necrosis e) Dupuytren’s contracture

Answers.

1. Hospital-acquired infections, also known as healthcare-associated infections (HAI), are nosocomially acquired infections that are typically not present or might be incubating at the time of admission. These infections are usually acquired after hospitalization and manifest 48 hours after admission to the hospital.

A surgical site infection (SSI) is an infection that occurs after surgery in the part of the body where the surgery took place.

Bacterimia:

Bacteremia is the presence of viable bacteria in the circulating blood.This may or may not have any clinical significance because harmless, transient bacteremia may occur following dental work or other minor medical procedures; however, this bacteremia is generally clinically benign and self-resolving in children who do not have an underlying illness or immune deficiency or a turbulent cardiac blood flow. The concern with occult bacteremia is that it could progress to a more severe local or systemic infection if left untreated. Most episodes of occult bacteremia spontaneously resolve, and serious sequelae are increasingly uncommon. However, serious bacterial infections occur, including pneumonia, septic arthritis, osteomyelitis, cellulitis, meningitis, brain abscesses, and sepsis, possibly resulting in death.

Systemic inflammatory response syndrome.

Criteria for Four Categories of the Systemic Inflammatory Response Syndrome Systemic Inflammatory Response Syndrome (SIRS) Two or more of the following: ▪ Temperature (core) >38°C or <36°C ▪ Heart rate >90 beats/min Respiratory rate of >20 breaths/min for patients spontaneously ventilating or a ▪ PaCO2 <32 mm Hg White blood cell count >12,000 cells/mm3 or <4000 cells/mm3 or >10% immature ▪ (band) cells in the peripheral blood smear Sepsis Same criteria as for SIRS but with a clearly established focus of infection Severe Sepsis Sepsis associated with organ dysfunction and hypoperfusion Indicators of hypoperfusion: ▪ Systolic blood pressure <90 mm Hg ▪ >40 mm Hg fall from normal systolic blood pressure ▪ Lacticacidemia ▪ Oliguria ▪ Acute mental status changes Septic Shock Patients with severe sepsis who ▪ Are not responsive to intravenous fluid infusion for resuscitation ▪ Require inotropic or vasopressor agents to maintain systolic blood pressure

Prevention of infection:

Best Strategies for Infection Prevention and Control

 Hand Hygiene.  Environmental hygiene.  Screening and cohorting patients.  Vaccinations.  Surveillance.  Antibiotic stewardship  Care coordination  Following the evidence Strategies for infection prevention & control

(1) Hand hygiene: hand washing should be the cornerstone of reducing healthcare associated infections. This is the simplest approach to preventing the spread of infections and needs to be incorporated into the culture of the organization. Wash hands with warm water and soap vigorously for at least 20 seconds. Also, all staff members and visitors in the facility should be encouraged to wash their hands before drinking, eating, providing care and between caring for patients.

(2) Use gloves: health care professionals may not always wear gloves when interacting with patients. But, if any contact with blood or bodily fluids is possible, such as when changing sheets or emptying trash, gloves should be worn.

(3) Disinfect and keep surfaces clean: between patients, every room in a facility should be cleaned thoroughly with a us epa approved healthcare grade disinfectant. This helps to prevent accidental transmission of infections as new patients are admitted.

(4) Use personal protective equipment: appropriate personal protective equipment (PPE), such as gowns, gloves, masks and face shields, should be readily available to staff for usage.

(5) Provide infection control education: staff members need to know how to identify common infections and help prevent their spread. In addition, your organization should provide continued, recurring education on infection control. This includes training on blood borne pathogen and droplet-borne infections.

(6) Develop an infection prevention and control policy: the facility must establish and maintain an infection control program designed to provide a safe, sanitary and comfortable environment and to help prevent the development and transmission of disease and infection.

(7) Antibiotic stewardship: the misuse and overuse of antibiotics can put patients at a risk of contracting infections. Inappropriate antibiotic use may also result in patients becoming resistant to some drugs. If those patients contract an infection, it becomes harder to treat them and the risk of it spreading increases.

2. a) This is a case of analgesic induced peptic ulcer bleeding:

Aspirin and paracetamol (acetaminophen) are the most commonly used minor analgesics, but their effects on the differ widely. The effects of other non-steroidal anti-inflammatory drugs (NSAIDs), including phenylbutazone, are intermediate. Aspirin is significantly associated with major upper gastrointestinal haemorrhage, whereas paracetamol is not. Short term use of aspirin produces erythema, erosions and occasionally ulcers; paracetamol does not, while other NSAIDs do so to varying degrees.

Complications of chronic peptic ulcer:

Internal bleeding: If a peptic ulcer develops near the site of a blood vessel, it can damage the vessel and cause bleeding. Internal is one of the most common complications of a peptic ulcer occurring in between 15-20% of cases. Risk factors for bleeding include:

 Continued use of non-steroidal anti-inflammatory drugs (nsaids), and  Being 60 years of age, or over. Depending on the site and type of the blood vessel, this could cause moderate but long-term bleeding which can lead to anaemia (a condition where the body does not have enough oxygen-carrying red blood cells). Symptoms of anaemia include:

 Fatigue,  Breathlessness (dyspnoea),  Pale skin, and  Irregular heart beats.

Alternatively, the bleeding can be rapid and massive, causing you to:

 Vomit blood, and/or  Pass stools that are very dark or tar-like.

If the bleeding is moderate, it can usually be treated by giving you injections of proton pump inhibitors (PPIs). Research has found that lowering the amount of acid around the site of the bleeding makes the blood more likely to clot so that the bleeding will stop. Massive bleeding can be treated using blood transfusions to replace any blood loss. Surgery can be used to repair the blood vessels. Once you are in a stable condition, an endoscopy will be carried out to determine whether there is a high risk of the bleeding recurring. If your risk is thought to be high, you will be given a further course of eradication therapy and PPIs.

Perforation Perforation means that the ulcer has eaten through all of your lining. It occurs in an estimated 2-10% of all cases. Perforation is potentially very serious because bacteria that live in your stomach can move out of your stomach and infect the lining of your abdomen (peritoneum). This is known as peritonitis. Peritonitis is a medical emergency because tissue of the peritoneum is usually sterile (germ- free) so unlike other parts of the body, such as the skin, it does not have an inbuilt defence mechanism for fighting off infection. In peritonitis, an infection can rapidly spread into the blood (sepsis) before spreading to other organs. This carries the risk of multiple organ failure and, if left untreated, death. The most common symptom of peritonitis is the sudden onset of abdominal pain that then gets steadily worse. Peritonitis requires admission to hospital where you will be treated with injections of antibiotics to get rid of the infection. Surgery is then used to seal the hole in the stomach wall. Gastric obstruction In some cases, a peptic ulcer can produce inflammation (swelling) and /or scar tissue that can obstruct the normal passage of food through your digestive system. This is known as gastric obstruction. Gastric obstruction occurs in an estimated 5-8% of cases of peptic ulcers. Symptoms of gastric obstruction include:

 Repeated episodes of vomiting, with large amounts of vomit that contain undigested food,  A persistent feeling of bloating, or fullness,  Feeling very full after eating less food than usual, and  Unexplained weight loss.

If a diagnosis of gastric obstruction is suspected, an endoscopy will be used to determine the type and site of the obstruction. If the obstruction is due to inflammation, PPIs or H2-receptor antagonists can be used to reduce the inflammation. If the obstruction is due to scar tissue, surgery will be required to treat it. One option is to pass a small balloon through an endoscope and then inflate it in order to widen the site of the obstruction. In more severe cases of scarring, it may be necessary to surgically remove the affected section of stomach, before and reattaching the remainder of the stomach.

2. b) Diagnosis is breast abscess/ lactational mastitis.

The diagnosis of mastitis and breast abscess can usually be made based on a physical examination. Lactation mastitis usually affects only one breast and the symptoms can develop quickly The signs and symptoms usually appear suddenly and they include:

 Breast tenderness or warmth to the touch  General malaise or feeling ill  Swelling of the breast  Pain or a burning sensation continuously or while breast-feeding  Skin redness, often in a wedge-shaped pattern  Fever of 101 F (38.3 C) or greater  The affected breast can then start to appear lumpy and red. Some women may also experience flu-like symptoms such as:

 Aches  Shivering and chills  Feeling anxious or stressed  Fatigue[

A breast ultrasound may be performed. The ultrasound provides a clear image of the breast tissue and may be helpful in distinguishing between simple mastitis and abscess or in diagnosing an abscess deep in the breast. In cases of infectious mastitis, cultures may be needed in order to determine what type of organism is causing the infection. Cultures are helpful in deciding the specific type of antibiotics that will be used in curing the disease. These cultures may be taken either from the breast milk or of the material aspirated from an abscess. Mammograms or breast biopsies are normally performed on women who do not respond to treatment. In lactation mastitis, frequent emptying of both breasts by breastfeeding is essential. Also essential is adequate fluid supply for the mother and baby. For breastfeeding women with light mastitis, massage and application of heat prior to feeding can help as this may aid unblocking the ducts. However, in more severe cases of mastitis heat or massage could make the symptoms worse and cold compresses are better suited to contain the inflammation. Antibiotics In lactational mastitis, antibiotics are not needed in the overwhelming majority of cases and should be used only for bacterial infections. For people with nonsevere infections, dicloxacillin or cephalexin are recommended. For people with severe infections, vancomycin is recommended. The length of antibiotic treatment ranges anywhere from 5–14 days.

An abscess (or suspected abscess) in the breast may be treated by ultrasound-guided fine- needle aspiration (percutaneous aspiration) or by surgical incision and drainage; each of these approaches is performed under antibiotic coverage. One recommended treatment includes antibiotics, ultrasound evaluation and, if fluid is present, ultrasound-guided fine needle aspiration of the abscess with an 18 gauge needle, under saline lavage until clear. The exudate is then sent for microbiological analysis for identification of the pathogen and determination of its antibiotic sensitivity profile, which may in turn give an indication for changing the antibiotics. At follow-up, a mammography is performed if the condition has resolved; otherwise the ultrasound-guided fine-needle aspiration with lavage and microbiological analysis is repeated. If three to five aspirations still do not resolve the condition, percutaneous drainage in combination with placement of an indwelling catheter is indicated, and only if several attempts at ultrasound-guided drainage fail, surgical resection of the inflamed lactiferous ducts (preferably performed after the acute episode is over). 3. a) A parotid fistula is a communication between the skin and a salivary duct or gland, through which saliva is discharged

Causes:

I. and duct injuries following surgery of parotid gland and temporomandibular joint. II. Various other causes of parotid injury are rupture of parotid abscess, inadvertent incision of parotid abscess, complication of superficial Parotidectomy, gunshot wounds and trauma. Clinical features:

 Salivary extravasations into the tissues causing swelling over or adjacent to parotid gland.  Expanding neck mass and cutaneous fistula formation.  In glandular fistulas discharge is less and tends to heal spontaneously with conservative treatment, where as ductal fistulas continuously discharge saliva and spontaneous healing is very rare. Diagnosis:

 Diagnosis is largely made from clinical signs and history.  The fistula is often visible over the parotid region, or located upon clinical examination. Discharge from the fistula, which must be differentiated from a draining sinus, often results in a visibly wet face.  The salivary discharge from a fistula is usually golden or blood tinged saliva, which will be viscous and 'stringy' in appearance.  The presence of mucous can be confirmed by using periodic acid-schiff stain if necessary.  History of or current presence of a wound over the parotid regionis a very good indicator. Treatment options:

 Conservative management  Aspiration and pressure dressings  Anti-Sialogogues  Radiation therapy  Parasympathetic Denervation (Tympanic Denervation)  Cauterization of the Fistulous tract  Reconstruction of the duct  Superficial or Total Parotidectomy.

3. b) Also known as Tuberculous lymphadenitis.

Tuberculous lymphadenitis (or tuberculous adenitis) is a chronic specific granulomatous inflammation of the lymph node with caseation necrosis, caused by infection with Mycobacterium tuberculosis or Mycobacterium bovis. The characteristic morphological element is the tuberculous granuloma(caseating tubercule). This consists of giant multinucleated cells and (Langhans cells), surrounded by epithelioid cells aggregates, T celllymphocytes and fibroblasts. Granulomatous tubercules eventually develop central caseous necrosis and tend to become confluent, replacing the lymphoid tissue. Stages of Tubercular Lymphadenitis -  Lymphadenitis  Periadenitis  Cold abscess  'Collar stud' abscess  Sinus

Diagnosis:

 A high index of suspicion is needed for the diagnosis of mycobacterial cervical lymphadenitis.  A thorough history and physical examination, tuberculin test, staining for acid-fast bacilli, radiologic examination, and fine-needle aspiration cytology (FNAC) will help to arrive at an early diagnosis of mycobacterial lymphadenitis.  Smears: Smears can be obtained either from a draining sinus or by FNA. Ziehl-Neelsen staining of the smears may reveal mycobacteria in the fresh specimens. Chance of finding AFB is higher in patients with cold abscess. Combination of FNA with culture or a Mantoux test further increases the diagnostic yield in mycobacterial cervical lymphadenitis.  Culture: Culture of mycobacterium is diagnostic for mycobacterial cervical lymphadenitis. However, a negative culture result should not exclude the diagnosis of mycobacterial cervical lymphadenitis. The presence of 10–100 bacilli per cubic millimeter of the specimen is enough for a positive culture result.  Tuberculin Test: This intradermal test (Mantoux test) is used to show delayed type hypersensitivity reactions against mycobacterial antigen, in which the reagent is mostly protein purified derivative (PPD). The test becomes positive 2–10 weeks after the mycobacterial infection. o Positive reactions (>10-mm induration) can occur in M. tuberculosis infections. o Intermediate reactions (5- to 9-mm induration) can occur after BCG vaccination, M. tuberculosis infection or nontuberculous mycobacterial infections. . Negative reactions (< 4-mm induration) represent a lack of tuberculin sensitization. False-negative reactions can occur in about 20% of all persons with active tuberculosis. The test may be positive in different conditions, like other infections, metabolic disease, malnutrition, live virus vaccination, malignancy, immunosuppressive drugs, newborns, elderly people, stress, sarcoidosis and inadequate test application.  Polymerase chain reaction (PCR): It is a fast and useful technique for the demonstration of mycobacterial DNA fragments in patients with clinically suspected mycobacterial lymphadenitis.  Histopathology: Histopathologic examination is diagnostic of mycobacterial cervical lymphadenitis.  Radiology and imaging: Chest radiograph, ultrasound, CT and MRI of the neck can be performed in mycobacterial lymphadenitis. o Associated chest lesions as seen on chest radiography are very common in children but less common in adults. o Ultrasound of the neck can demonstrate singular or multiple hypoechoic and multiloculated cystic lesions that are surrounded with thick capsule. o On CT, the presence of conglomerated nodal masses with central luscency, a thick irregular rim of contrast enhancement and inner nodularity, a varying degree of homogeneous enhancement in smaller nodes, dermal and subcutaneous manifestations of inflammation, such as thickening of the overlying skin, engorgement of the lymphatics and thickening of the adjacent muscles, and a diffusely effaced fascial plane may suggest mycobacterial cervical lymphadenitis.

Treatment:

 Antituberculosis treatment is the mainstay in the management of TB lymphadenitis.  In addition surgical treatment is more useful in cases where abscess has been formed.

3. c) Volvulus is a broad term that describes the torsion of bowel around its mesentery. Torsion results in narrowing of the lumen at the point of rotation and compromise of the vessels that supply the torted gut.

4 types of volvulus:

 Gastric volvulus  midgut volvulus  cecal volvulus  sigmoid volvulus Sigmoid volvulus is the most common, accounting for 60% of cases of intestinal volvulus

Patients with volvulus are commonly elderly, debilitated, and bedridden. Often, the patient has a history of dementia or neuropsychiatric impairment. As a result, only a limited history is available. More than 60-70% of patients present with acute symptoms; the remainder present with subacute or chronic symptoms. A history of chronic constipation is common. The patient may describe previous episodes of abdominal pain, distention, and obstipation, which suggest repeated subclinical episodes of volvulus.

Laboratory Studies Laboratory tests include a complete blood count (CBC) with differential and a comprehensive metabolic profile. An elevated white blood cell (WBC) count and left shift indicate bowel ischemia, peritoneal infection, or systemic sepsis. Bowel obstruction may cause significant changes in electrolyte levels. Other diagnostic studies include plain abdominal radiography, computed tomography (CT), barium enema, and sigmoidoscopy or colonoscopy

Plain Abdominal Radiography Massive dilation of the loop arising from the pelvis and extending to the diaphragm is a typical finding of sigmoid volvulus. The walls of the loop are evident as three bright lines converging in the pelvis to create a beaklike appearance

Cecal volvulus produces large- and small-bowel obstruction. Radiographic findings reveal a markedly distended loop of bowel extending from the right lower quadrant upward to the left upper quadrant. The small bowel is distended, whereas the distal colon is decompressed. CT of Abdomen and Pelvis Computed tomography (CT) is not often needed, because the plain radiographic findings typically suffice for diagnosis of sigmoid volvulus. However, the radiographic findings for cecal volvulus may be less diagnostic. In such cases, CT can delineate the exact site of the torsion and reveal evidence of ischemia.

Surgery is the definitive treatment of sigmoid and cecal volvulus. The decisions regarding timing of surgery and choice of procedure depend on the clinical presentation.

3. d) Immune thrombocytopenia (ITP), previously known as idiopathic thrombocytopenic purpura, is defined as isolated thrombocytopenia with normal bone marrow and in the absence of other causes of thrombocytopenia. ITP has two distinct clinical syndromes, manifesting as an acute condition in children and a chronic condition in adults.

Pathophysiology ITP is primarily a disease of increased peripheral platelet destruction, with most patients having antibodies to specific platelet membrane glycoproteins. Relative marrow failure may contribute to this condition, since studies show that most patients have either normal or diminished platelet production. Acute ITP often follows an acute infection and has a spontaneous resolution within 2 months. Chronic ITP persists longer than 6 months without a specific cause.

Etiology ITP develops when platelets become coated with immunoglobulin G (IgG) autoantibodies to platelet membrane antigens, resulting in splenic sequestration and phagocytosis by mononuclear macrophages. The resulting shortened life span of platelets in the circulation, together with incomplete compensation by increased platelet production by bone marrow megakaryocytes, results in a decreased number of circulating platelets.

Clinical presentation: Onset of childhood ITP is typically abrupt, and often occurs several weeks after viral illness or live virus immunization. In adults, onset is typically gradual. Common signs include the following:  Purpura  Menorrhagia  Epistaxis  Gingival bleeding  Bruising tendency

Laboratory Studies Isolated thrombocytopenia on a complete blood cell count (CBC) is the key laboratory finding. The white blood cell (WBC) count and hemoglobin level typically are normal, unless severe hemorrhage has occurred. On peripheral smear, truly giant platelets suggest congenital thrombocytopenia. Coagulation study results are normal, and a bleeding time is not useful.

Imaging Studies A CT scan of the brain is warranted if concern exists regarding intracranial hemorrhage. Prehospital Care Prehospital care focuses on the ABCs (airway, breathing, circulation), which include providing oxygen if necessary, controlling severe hemorrhage, and initiating intravenous (IV) fluids to maintain hemodynamic stability. Airway control may be necessary for a large intracranial hemorrhage. Emergency medical services providers should be aware of the potential for serious bleeding complications in patients with immune thrombocytopenia (ITP). Treatment in pediatric patients American Society of Hematology (ASH) guidelines recommend that in children who have no bleeding, or only mild bleeding (ie, skin manifestations only, such as bruising and petechiae), regardless of the platelet count, management should be with observation alone. For pediatric patients requiring treatment, ASH suggests a short course of corticosteroids for first-line treatment rather than IVIG or anti-D immunoglobulin. For children or adolescents with ITP who have significant ongoing bleeding despite treatment with IVIG, RhIG, or conventional doses of corticosteroids, the ASH suggests a thrombopoietin receptor agonist (TPO-RA) rather than rituximab or splenectomy as second- line treatment. Rituximab may also be considered as an alternative to splenectomy. Treatment in adults A 2010 international consensus report advised that treatment for ITP is rarely indicated in adult patients with platelet counts above 50×109/L, in the absence of the following:  Bleeding due to platelet dysfunction or another hemostatic defect  Trauma  Surgery  Clearly identified comorbidities for bleeding  Mandated anticoagulation therapy  Profession or lifestyle that predisposes the patient to trauma In adults, treatment is recommended for a platelet count < 30×109/L. The ASH recommends that if treatment is needed, shorter courses of corticosteroids (≤6 weeks) are preferred over prolonged courses (>6 weeks including treatment and taper) as first-line treatment. The guidelines suggest either prednisone (0.5-2.0 mg/kg per day) or dexamethasone (40 mg per day for 4 days) as the type of corticosteroid for initial therapy. In adults with ITP lasting ≥3 months who are corticosteroid-dependent or have no response to corticosteroids, the ASH guidelines suggest either splenectomy or a TPO-RA. In adults who are going to be treated with aTPO-RA, either eltrombopag or romiplostim is suggested by the ASH guidelines. ASH also suggests rituximab over splenectomy. For adults with ITP, ASH guidelines recommend treating newly diagnosed patients whose platelet count is less than 30 × 109/L. Recommendations for first-line treatment include the followin:  Shorter courses of ≤ 6 weeks are preferred over prolonged courses (>6 weeks including treatment and taper) of corticosteroids  Either prednisone (0.5-2.0 mg/kg per day) or dexamethasone (40 mg per day for 4 days) as the type of corticosteroid for initial therapy.  IVIg may be used with corticosteroids when a more rapid increase in platelet count is required  Either IVIg or anti-D (in appropriate patients) be used if corticosteroids are contraindicated Recommended second-line treatments for cases that do not respond to corticosteroids, or recur afterward, are as follows, in order of preference:  A TPO-RA (eltrombopag or romiplostim)  Rituximab  Splenectomy

3.e) Introduction:Acute necrotizing pancreatitis is defined as necrosis of the pancreatic parenchyma with or without necrosis of the peripancreatic tissues.  It occurs as a complication in 20%–30% of patients with acute pancreatitis and historically has been associated with high rates of morbidity (34%–95%) and mortality (2%–39%).  Mortality within the first 2 weeks of onset is most often due to systemic inflammatory response syndrome, which is an exaggerated inflammatory response associated with systemic organ dysfunction, immunosuppression, and organ failure.  Subsequent mortality is typically the result of uncontrolled infection.  Recent developments in the overall medical management of necrotizing pancreatitis and the application of new endoscopic, interventional, and surgical techniques have led to improved outcomes .

Classification and Terminology in Necrotizing Pancreatitis:

The revised Atlanta classification system divides pancreatic necrosis into three morphologic subtypes, depending on whether it involves pancreatic parenchyma only, peripancreatic tissues only, or both pancreatic parenchyma and peripancreatic tissues.

Imaging:

 Abdominal Ultrasound

 CT scan: Areas of necrosis are seen as non-enhancing low attenuating regions within the pancreas best observed if a dual phase pancreatic study is performed. Foci of gas may also be present in more extreme cases; extraluminal gas is highly suggestive of superimposed infection (see: emphysematous pancreatitis).

Blood investigations:

 Pancreatic enzymes  Sodium  Potassium  Glucose  Cholesterol  Triglycerides

Necrotizing Pancreatitis: Management And Treatment The initial assessment of AP should involve determining disease severity and whether signs of organ failure are present. This acuity will dictate admission to the regular floor versus an intensive care unit. A patient with AP should undergo a workup for the etiology of pancreatitis with attention to any signs of biliary obstruction and or cholangitis that might warrant an urgent endoscopic retrograde cholangiopancreatography (ERCP). The first step in management of AP is aggressive intravenous fluid hydration to maintain intravascular volume, as well as pancreatic and systemic perfusion. Antibiotics have no prophylactic role in preventing infection and failed to show benefit in sterile pancreatic necrosis.

Nutritional support and enteral feeding Total parenteral nutrition (TPN) was the standard of care for patients with severe AP, however enteral nutrition beyond the ligament of Treitz was found to be more beneficial. Approaches to drainage of necrotic collections Two thirds of necrotic pancreatic collections are sterile and will resolve with conservative management, while the remainder will become infected and will require further intervention. Thus, the most important factor to guide further interventions is the presence of symptoms or infected necrosis. Indications for intervention in each stage of the necrotic collection are as follows:

▪ A sterile ANC with presence of symptoms, such as abdominal pain or mechanical obstruction: intervention should be delayed preferably for 4 weeks or longer.

▪ An infected pancreatic fluid collection: dictate drainage, endoscopic drainage should be preferred over radiology whenever possible due to better drainage and decreased risk of percutaneous fistula, and over surgery because of high morbidity and mortality.

▪ An asymptomatic WOPN and a sterile ANC do not require intervention regardless of size, because these may resolve spontaneously over time. Surgical

 Open necrosectomy used to be the standard approach for treating infected pancreatic necrosis.

Image guided percutaneous and endoscopic drainage are also done.

4. a) Sequestrum.

 Sequestrum is a devitalized avascular segment of bone, surrounded, by pus /infected granulation tissue and is more dense than surrounding bone.Because of avascularity, sequestrum does not decalcify , is more radio opaque and heavy, so sinks in water.  Its outer surface is usually jagged / irregular due to erosive process by proteolytic enzymes in granulation tissue.

It is a complication (sequela) of osteomyelitis. The pathological process is as follows:

 Infection in the bone leads to an increase in intramedullary pressure due to inflammatory exudates  The periosteum becomes stripped from the osteum, leading to vascular thrombosis  Bone necrosis follows due to lack of blood supply  Sequestra are formed  The sequestra are surrounded by sclerotic bone which is relatively avascular (without a blood supply). Within the bone itself, the haversian canals become blocked with scar tissue, and the bone becomes surrounded by thickened periosteum.  Due to the avascular nature of this bone, antibiotics which travel to sites of infection via the bloodstream poorly penetrate these tissues, hence the difficulty in treating chronic osteomyelitis.  At the same time as this, new bone is forming (known as involucrum). Openings in this involucrum allow debris and exudates (including pus) to pass from the sequestrum via sinus tracts to the skin.  Rarely, a sequestrum may turn out to be an osteoid osteoma, a rare tumor of the bone.

 Types of Sequestra: 1 Tubular ( Diaphyseal) – Pyogenic Osteomyelitis 2 Trapezoid - Pyogenic Osteomyelitis 3 Ring -At end of stumps, around Steinman pin, wires, Schanz screws

4 Flake/ Feathery - Tuberculous Osteomyelitis( in cavity ) 5 coarse sandy - Tuberculous Osteomyelitis ( out of cavity ) 6 Fine sandy - Viral Osteomyelitis 7 Black - Actinomycosis

4. b) Tuberculous spondylitis, also known as Pott disease, refers to vertebral body osteomyelitis and intervertebral discitis from tuberculosis (TB). The spine is the most frequent location of musculoskeletal tuberculosis, and commonly related symptoms are back pain and lower limb weakness/paraplegia.

Epidemiology

Tuberculous spondylitis is one of the more common infections of spine in countries where TB is prevalent. Unfortunately, the incidence of tuberculous spondylitis, as with other forms of TB, is on the rise, due to new multiple drug resistant strains.

Discitis/osteomyelitis represents approximately 50% of all musculoskeletal tuberculosis, and usually affects the lower thoracic and upper lumbar levels of the spine.

Clinical presentation

Patients usually present with back pain, lower limb weakness/paraplegia, and kyphotic deformity. Constitutional symptoms (fever and weight loss) are also common but not as pronounced as with bacterial discitis/osteomyelitis.

Pathology

The spine is involved due to hematogenous spread via the venous plexus of Batson 2. There is usually a slow collapse of one or usually more vertebral bodies, which spreads underneath the longitudinal ligaments. This results in an acute kyphotic or "gibbus" deformity. This angulation, coupled with epidural granulation tissue and bony fragments, can lead to cord compression. Unlike pyogenic infections, the discs can be preserved. In late-stage spinal TB, large paraspinal abscesses without severe pain or frank pus are common, leading to the expression "cold abscess".

Radiographic features Plain radiograph

The spread of infection is typically described as 'sub-ligamentous': beneath the anterior longitudinal ligament, usually sparing the posterior elements and often involving multiple levels.

Tuberculous spondylitis can be difficult to detect in early stages because of relative preservation of the disc space.

A reduction in vertebral height is often seen with the irregularity of the anterosuperior end plate being relatively early and subtle sign. Due to the subligamentous extension, there may be some irregularity of the anterior vertebral margin. This is a classical appearance with TB spondylitis.

Later, paraspinal collections can develop which can be remarkably large.

Ivory vertebrae can result with re-ossification. Other associated features may include:

 Gibbus deformity  Vertebra plana

As with other extrapulmonary TB, the chest film may be unrevealing (no pulmonary lesions seen in up to 50% of cases), with the source being a primary lung lesion that is clinically silent.

CT and MRI

Cross-sectional imaging is required to assess better the extent of involvement and particularly for the presence of an epidural component and cord compression. MRI is the modality of choice for this, with CT with contrast being a distant second.

Features include irregularity of both the endplate and anterior aspect of the vertebral bodies, with bone marrow oedema and enhancement seen on MRI:

 T1: hypointense marrow in adjacent vertebrae  T2: hyperintense marrow, disc, soft tissue infection  T1 C+ (Gd): marrow, subligamentous, discal, dural enhancement

The paraspinal collections are typically well circumscribed, with fluid centers and well- defined enhancing margins.

Pharmacologic Therapy:

Isoniazid and rifampin should be administered during the whole course of therapy. Additional drugs are administered during the first 2 months of therapy. These are generally chosen from among the first-line drugs, which include pyrazinamide, ethambutol, and streptomycin. The use of second-line drugs is indicated in cases of drug resistance. Treatment duration

Studies performed by the British Medical Research Council indicate that tuberculous spondylitis of the thoracolumbar spine should be treated with combination chemotherapy for 6-9 months.]

However, the research council’s studies did not include patients with multiple vertebral involvement, cervical lesions, or major neurologic involvement. Because of these limitations, many experts still recommend chemotherapy for 9-12 months.

For selected cases with surgical indication that allows complete debridement of the lesion, a combination of surgery and ultra-shortened course of therapy (4.5 mo), appears to show comparable outcomes of a combination of surgery and 9 months of drug therapy.

Surgical Indications and Contraindications

Indications

While most patients should respond to medical treatment, a surgical approach needs to be evaluated and considered. Indications for surgical treatment of Pott disease generally include the following:

 Neurologic deficit - Acute neurologic deterioration, paraparesis, and paraplegia  Spinal deformity with instability or pain  No response to medical therapy - Continuing progression of kyphosis or instability  Large paraspinal abscess  Nondiagnostic percutaneous needle biopsy sample

Resources and experience are key factors in the decision to use a surgical approach. The lesion site, extent of vertebral destruction, and presence of cord compression or spinal deformity determine the specific operative approach (kyphosis, paraplegia, tuberculous abscess).

Vertebral damage is considered significant if more than 50% of the vertebral body is collapsed or destroyed or a spinal deformity of more than 5° exists.

The most conventional approaches include anterior radical focal debridement and posterior stabilization with instrumentation. The specific advantages and limitations of surgical techniques vary. Individualization of the case is of greatest importance. Newer modalities and techniques are being reported, such as thoracoscopic decompression.

In Pott disease that involves the cervical spine, the following factors justify early surgical intervention:

 High frequency and severity of neurologic deficits  Severe abscess compression that may induce dysphagia or asphyxia  Instability of the cervical spine Contraindications

Vertebral collapse of a lesser magnitude is not considered an indication for surgery because, with appropriate treatment and therapy compliance, it is less likely to progress to a severe deformity.

4. c) Tennis elbow.

Introduction:

 Also known as lateral epicondylitis.  Tennis elbow is a lesion affecting the tendinous origin of the wrist extensor. Most commonly involves the extensor carpi radialis brevis (ECRB), occasionally, the anterior edges of the extensor communis, the extensor radialis longus, and rarely the origin of the extensor carpi ulnaris. We can say that the lesion may be tenoperiosteal, tendinous or musculotendinous.

Cause:

Overuse Recent studies show that tennis elbow is often due to damage to a specific forearm muscle. The extensor carpi radialis brevis (ECRB) muscle helps stabilize the wrist when the elbow is straight. This occurs during a tennis groundstroke, for example. When the ECRB is weakened from overuse, microscopic tears form in the tendon where it attaches to the lateral epicondyle. This leads to inflammation and pain.

The ECRB may also be at increased risk for damage because of its position. As the elbow bends and straightens, the muscle rubs against bony bumps. This can cause gradual wear and tear of the muscle over time.

Activities

Athletes are not the only people who get tennis elbow. Many people with tennis elbow participate in work or recreational activities that require repetitive and vigorous use of the forearm muscle.

Painters, plumbers, and carpenters are particularly prone to developing tennis elbow. Studies have shown that auto workers, cooks, and even butchers get tennis elbow more often than the rest of the population. It is thought that the repetition and weight lifting required in these occupations leads to injury.

Age Most people who get tennis elbow are between the ages of 30 and 50, although anyone can get tennis elbow if they have the risk factors. In racquet sports like tennis, improper stroke technique and improper equipment may be risk factors.

Unknown

Clinical Examination:  Pain at the common extensor origin  Often pain 5mm distal and slightly anterior to lateral humeral epicondyle  Pain with resisted wrist extension and digit extension.

Pathophysiology

 Micro tears at ECRB  Incomplete healing  Granulation tissue formation  Development of angiofibroblastic hyperplasia

Imaging:

 Plain film  MRI

Nonsurgical treatment:

 Activity modification  Counterforce brace  Wrist extension splint  Stretching  Ultrasound  Strengthening after acute phase  NSAIDs  Steroid injections

Surgical treatment:

 Debride degenerative tissue at ECRB origin  Remove traction spurs  Stimulate angiogenesis  Repair defect/ superficial extensor closure

4. d) Introduction:

Avascular necrosis is the death of bone tissue due to a lack of blood supply. Also called osteonecrosis, it can lead to tiny breaks in the bone and the bone's eventual collapse.

A broken bone or dislocated joint can interrupt the blood flow to a section of bone. Avascular necrosis is also associated with long-term use of high-dose steroid medications and excessive alcohol intake.

Anyone can be affected, but the condition is most common in people between the ages of 30 and 50.

Symptoms  Many people have no symptoms in the early stages of avascular necrosis. As the condition worsens, one might feel pain while putting weight on affected joint. Eventually, one can feel the pain even while lying down.

 Pain can be mild or severe and usually develops gradually. Pain associated with avascular necrosis of the hip might center on the groin, thigh or buttock. Besides the hip, the areas likely to be affected are the shoulder, knee, hand and foot.

 Some people develop avascular necrosis on both sides (bilaterally) — such as in both hips or in both knees.

Causes

 Joint or bone trauma.

 Fatty deposits in blood vessels.

 Certain diseases: Medical conditions, such as sickle cell anemia and Gaucher's disease, also can cause diminished blood flow to bone.

 For about 25 percent of people with avascular necrosis, the cause of interrupted blood flow is unknown.

Risk factors

 Trauma.

 Steroid use.

 Excessive alcohol use.

 Bisphosphonate use.

 Certain medical treatments.

Medical conditions associated with avascular necrosis include:

 Pancreatitis

 Diabetes

 Gaucher's disease

 HIV/AIDS

 Systemic lupus erythematosus

 Sickle cell anemia Complications

Untreated, avascular necrosis worsens with time. Eventually, the bone can collapse. Avascular necrosis also causes bone to lose its smooth shape, potentially leading to severe arthritis.

Prevention

 Limit alcohol.

 Keep cholesterol levels low.

 Monitor steroid use.

 Stop smoking.

4. e) Dupuytren’s contracture. Introduction:

Dupuytren's contracture is a thickening of the fibrous tissue layer underneath the skin of the palm and fingers. Although painless, the thickening and tightening (contracture) of this fibrous tissue can cause the fingers to curl (flex).

Aetiopathogenesis:

 Common in men older than 40 years; in persons of Northern European descent; and in persons who smoke,use alcohol, or have diabetes (3 to 33 %)

 Present with a small, pitted nodule (or multiple nodules) on the palm, which slowly progresses to contracture of the fingers

 Progresses' faster in <50 yr olds

 Smoking and alcohol use increase the chance that surgery will be needed

Clinical features:

 Found on the palm of the hand proximal to the metacarpo-phalangeal (MCP) joint. Can be bilateral.

 Patients usually have difficulty with tasks such as face washing, hair combing, and putting their hands in their pockets.

 Note the site of the nodule and the presence of contractures; bands; and skin pitting, tenderness, and dimpling.

Treatment:

 Nonsurgical Treatment o Steroid injection. If a lump is painful, an injection of corticosteroid - a powerful anti- inflammatory medication - may help relieve the pain. In some cases, it may prevent the progression of contracture. Several injections may be needed for a lasting effect. o Splints. Splinting does not prevent increased bend in the finger. Forceful stretching of the contracted finger will not help either, and may speed the progression of contracture.

 Surgical Treatment Surgery is recommended when your doctor has confirmed through measurements over time that the disease is progressing. Some patients turn to surgical treatment when hand function is limited; they have trouble grasping objects or putting their hands in their pockets.

 Surgical procedure. Surgery for Dupuytren's contracture divides or removes the thickened bands to help restore finger motion. Sometimes the wound is left open and allowed to heal gradually. Skin grafting may be needed.  Complications:Although rare, risks of surgery include injury to nerves and blood vessels, and infection. Permanent stiffness of the fingers may occur, although this is also rare.  Recovery. Some swelling and soreness are expected after surgery, but severe problems are rare.  Most people will be able to move their fingers better after surgery.  Recurrence. Approximately 20% of patients experience some degree of recurrence. This may require further surgery.

The West Bengal University of Health Sciences

M.B.B.S. 3rd Professional Part – II Examination, 2019

Subject: Surgery Time: 21/2 hrs.

Paper: I Marks: 60

Group –A 1. Define shock. Give the etiological classification of shock. Describe the pathogenesis and the management of septic shock. 2+3+5+5

Group –B 2.a) A 45 year old lady, known to be having USG evident cholelithiasis, admitted with the complaints of severe pain abdomen radiating to the back, out of proportion to any of the signs present and her laboratory values showed a significant serum hyperamylasemia (3 times the normal value). How would you proceed to investigate further, prognosticate and manage (principles only)? 5+5+5

Or b) A 54 year old gentleman presents in the OPD with a history of dull, vague epigastric pain with persistently and steadily progressive yellow discoloration of the eyes and urine for last 4 months with intermittent rise of temperature with chills and rigor for last 15 days. He also gives a history of recently developed anorexia and gross weight loss. On examination, his gall bladder is palpable as soft cystic swelling. How would you proceed to have a detailed work –up of the patient, prepare him for general anaesthesia for a major operation and give the outline of treatment (principles only including palliation)? 5+5+5

Group – C 3. Answer in brief on any three of the following: 3x5

a) Subphrenic abscess b) Complications of splenectomy c) Femoral hernia d) Adenomatous polyps of colon e) Tuberculous cervical lymphadenopathy. Group – D 4. Write short notes (any three): 3x5

a) Cubitus valgus b) Fracture healing c) Osteosarcoma d) Club foot e) Supracondylar fracture of humerus

Definition of shock: Shock may be defined as inadequate delivery of oxygen and nutrients to maintain normal tissue and cellular function. The resultant cellular injury is initially reversible; if the hypoperfusion is severe enough and prolonged, the cellular injury becomes irreversible.

Classification: a. Cardiogenic : Shock caused as a result of cardiac pump failure

Causes:

 Myocardial Infarction  Arrythmias (Atrial fibrillation, ventricular tachycardias, bradycardias, etc)  Mechanical abnormalities (valvular defects)  Extracardiac abnormalities (PE, pulm HTN, tension pneumothorax) b. Hypovolemic: Shock caused by decreased preload due to intravascular volume loss.

Causes:

 Hemorrhagic – trauma, GI bleed, hemorrhagic pancreatitis, fractures  Fluid loss induced – Diarrhea, vomiting, burns c. Distributive: Shock as a result of severely diminished systemic vascular resistance.

a. Septic: secondary to an overwhelming infection

b. Anaphylactic: secondary to an overwhelming infection

c. Neurogenic: secondary to a sudden loss of the autonomic nervous system function. d. Combined.

Pathogenesis of sepsis and septic shock:

Management:

Diagnosis:

 Before the initiation of antimicrobial therapy, at least two blood cultures should be obtained  At least one drawn percutaneously.  At least one drawn through each vascular access device if inserted longer than 48 hours  Other cultures such as urine, cerebrospinal fluid, wounds, respiratory secretions or other body fluids should be obtained as the clinical situation dictates  Other diagnostic studies such as imaging and sampling should be performed promptly to determine the source and causative organism of the infection may be limited by patient stability.

Treatment primarily consists of:

I. Volume resuscitation II. Early antibiotic administration III. Early goal directed therapy IV. Rapid source identification and control. V. Support of major organ dysfunction. VI. Sequestration of lipopolysaccharides.

 Antibiotics (early administration)  Hemodynamic support – (fluid resuscitation)  Restore tissue perfusion  Normalize cellular metabolism  Tight glycemic control  Vasopressor agents Dopamine, norepinephrine, dobutamine  Source control – Surgical debridement of infected, devitalized tissue – Catheter replacement  Supplemental oxygen (treatment of acute respiratory distress syndrome,ARDS)  Nutritional support  Anti-inflammatory agents : o Cortocosteroids o Ibuprofen, o Prostaglandin E1 o Pentoxifylline  Oxygen Scavengers :N-acetylcysteine , selenium  Drugs modifying coagulation – Anti-thrombin III  Drugs enhancing host defenses o Intravenous immunoglobulin (IVIG) o Interferon-gamma o GM-CSF o Immunonutrition  Other drugs o Growth hormone, antibiotics, fresh frozen plasma, anesthetic sedative and analgesic agents, catecholamines  Hemofiltration, plasma filtration, plasma exchange  Experimental therapies o Anti-endotoxin therapies – IVIG, BPI protein o IL-1Ra o Anti-TNF-alpha, soluble TNFR o PLA2 inhibitors, PAF inhibitors o iNOS inhibitors o Anti-coagulants (APC)

2. a) It is a case of Gall stone pancreatitis.

Prognostic indices

In predicting the prognosis, there are several scoring indices that have been used as predictors of survival. Two such scoring systems are the Ranson criteria and APACHE II (Acute Physiology, Age and Chronic Health Evaluation) indices. Most, but not all studies report that the Apache score may be more accurate.

Pracactice guidelines state:

"The two tests that are most helpful at admission in distinguishing mild from severe acute pancreatitis are APACHE-II score and serum hematocrit. It is recommended that APACHE-II scores be generated during the first 3 days of hospitalization and thereafter as needed to help in this distinction. It is also recommended that serum hematocrit be obtained at admission, 12 h after admission, and 24 h after admission to help gauge adequacy of fluid resuscitation."

"Immediate assessment should include clinical evaluation, particularly of any cardiovascular, respiratory, and renal compromise, body mass index, chest x ray, and APACHE II score".

Ranson criteria is a clinical prediction rule for predicting the severity of acute pancreatitis.

At admission Age in years > 70 years

White blood cell count > 18000 cells/mm3

Blood glucose > 220 mg/dL

Serum AST > 400 IU/L

Serum LDH > 250 IU/L

At 48 hours Calcium (serum calcium < 2.0 mmol/L (< 8.0 mg/dL)

Hematocrit fall > 10%

Oxygen (hypoxemia PO2) - NA

BUN increased by 1.8 or more mmol/L (5 or more mg/dL) after IV fluid hydration

Base deficit (negative base excess) > 5 mEq/L Sequestration of fluids > 4 L

The criteria for point assignment is that a certain breakpoint be met at anytime during that 48 hour period, so that in some situations it can be calculated shortly after admission. It is applicable to both gallstone and alcoholic pancreatitis.

Alternatively, pancreatitis can be diagnosed by meeting any of the following:

APACHE II score

Apache score of ≥ 8 Organ failure Substantial pancreatic necrosis (at least 30% glandular necrosis according to contrast-enhanced CT)

Interpretation If the score ≥ 3, severe pancreatitis likely. If the score < 3, severe pancreatitis is unlikely Or

Score 0 to 2 : 2% mortality Score 3 to 4 : 15% mortality Score 5 to 6 : 40% mortality Score 7 to 8 : 100% mortality

"Acute Physiology And Chronic Health Evaluation" (APACHE II) score > 8 points predicts 11% to 18% mortality. Balthazar scoring: Computed Tomography Severity Index (CTSI) is a grading system used to determine the severity of acute pancreatitis. The numerical CTSI has a maximum of ten points, and is the sum of the Balthazar grade points and pancreatic necrosis grade points: CT SEVERITY INDEX AND MODIFIED CTSI

CT SEVERITY MODIFIED CT SEVERITY CT SEVERITY INDEX MODIFIED CT SEVERITY INDEX INDEX (MORTELE,2004) (BALTHAZAR, 1990) INDEX (MORTELE, 2004) (BALTHAZAR,1 PROGNOSTIC POINTS PROGNOSTIC POINTS 990) INDICATOR INDICATOR PROGN PROGNOSTIC PANCREATIC PANCREATIC OSTIC POIN INDICATOR POINT INFLAMMATION INFLAMMATION INDICAT TS S NORMAL 0 NORMAL 0 OR PANCREAS PANCREAS PANCRE PANCREATIC ENLARGED 1 PANCREATIC ABN 2 ATIC NECROSIS PANCREAS +/- NECROS PERIPANCREATIC IS INFLAMMATION PANCREATIC 2 PANCREATIC OR 4 NONE 0 NONE 0 ABNORMALITIES PERIPANCREATIC WITH FLUID 2 <30% 2 PERIPANCREATIC COLLECTION/ FAT <30% INFLAMMATION NECROSIS 30- 4 >30% 4 SINGLE FLUID 3 COLLECTION 50% 2/MORE 4 6 EXTRAPANCREATIC 2 COLLECTION OR >50% COMPLICATIONS GAS

CTSI's staging of acute pancreatitis severity has been shown by a number of studies to provide more accurate assessment than APACHE II, Ranson, and C-reactive protein (CRP) level. However, a few studies indicate that CTSI is not significantly associated with the prognosis of hospitalization in patients with pancreatic necrosis, nor is it an accurate predictor of AP severity.

Glasgow Imrie criteria

3 or more positive criteria within 48h of admission = severe attack

Age > 55y

WBC > 15 000

Glucose > 7mmol/L

Blood urea > 7mmol/L

Albumin < 35g/L

Corrected calcium < 2mmol/L

PaO2 < 10kPa Marshall Scoring System

Most sensitive for evaluation of AP patients. • 50% of the patients with necrotising acute pancreatitis develop organ failure with severe acute pancreatitis. • 15% of edematous acute pancreatitis develop organ failure.

Score > 3 is associated with  Severe course,  Systemic complications and  Significant correlation with fatal outcome (Р = 0.007) .

Criteria for organ failure based on Marshall scoring system: Organ system Score 0 1 2 3 4 Respiratory >400 301-400 201-300 101-200 <100 (PaO2/FiO2)

Renal ≤1.5 >1.5 - ≤ 1.9 >1.9 - ≤ 3.5 >3.5 - ≤ > 5.0 (Serum 5.0 Creatinine md/dl) Cardiovascular >90 <90, fluid <90, fluid <90, <90, (systolic blood responsive unresponsiv pH<7.3 pH<7.2 pressure, mm e Hg)

BISAP: Bedside index for severity in Acute Pancreatitis 1. BUN > 25mg/dl.

2. Impaired mental status ( Glasgow Coma Score < 13) 3. SIRS • Incremental increases in the BISAP score (3

4. Age > 60 years or more) have been shown to correlate with

5. Pleural effusion detected on imaging. an increased risk of organ failure pancreatic necrosis and mortality

One point is assigned for each variable within 24 hours of presentation and added for a composite score of 0-5.

SOFA SCORE SOFA Score Variables 0 1 2 3 4

Respiratory > 400 ≤ 400 ≤ 300 ≤ 200 ( with respiratory ≤ 100 (PaO2/FiO2) support) (with respiratory support)

Coagulation > 150 ≤ 150 ≤ 100 ≤ 50 ≤ 20 ( Platelets x 103/µL)

Liver < 1.2 1.2 – 1.9 2.0 – 5.9 6.0 – 11.9 > 12.0 ( Bilirubin: mg/dl) Cardiovascular No Mean Arterial Dopamine ≤ 5 Dopamine > 5 Dopamine > 15 (Hypotension) hypotensi Pressure < 70 (microgram/kg/min) or (microgram/kg/min), (microgram/kg/min), on mm of Hg Dobutamine (any dose) Epinephrine ≤ 0.1 Epinephrine > 0.1 Adrenergic agents (microgram/kg/min), or (microgram/kg/min), or administered for atleast one Norepinephrine ≤ 0.0 Norepinephrine > 0.0 hour Adrenergic agents administered Adrenergic agents for atleast one hour administered for atleast one hour

Central nervous 15 13-14 10-12 6-9 <6 system (Glasgow coma scale)

Renal <1.2 1.2-1.9 2.0-3.4 3.5-4.9 or < 500 > 5.0 or < 200 (Creatinine – mg/dl or Urine output ml/day Treatment

 Pain control  Bowel rest

In the management of acute pancreatitis, the treatment is to stop feeding the patient, giving him or her nothing by mouth, giving intravenous fluids to prevent dehydration, and sufficient pain control. As the pancreas is stimulated to secrete enzymes by the presence of food in the stomach, having no food pass through the system allows the pancreas to rest.

 Nutritional support

Recently, there has been a shift in the management paradigm from TPN (total parenteral nutrition) to early, post-pyloric enteral feeding (in which a feeding tube is endoscopically or radiographically introduced to the third portion of the ). The advantage of enteral feeding is that it is more physiological, prevents gut mucosal atrophy, and is free from the side effects of TPN (such as fungemia). The additional advantages of post-pyloric feeding are the inverse relationship of pancreatic exocrine secretions and distance of nutrient delivery from the , as well as reduced risk of aspiration.

Disadvantages of a naso-enteric feeding tube include increased risk of sinusitis (especially if the tube remains in place greater than two weeks) and a still-present risk of accidentally intubating the bronchus even in intubated patients.

 Antibiotics:

Prophylactic Antibiotic Therapy

• Avoid prophylactic antibiotic doses – use ONLY for DEFINED INFECTIONS ( INFECTIVE NECROSIS or EXTRAPANCREATIC inf) • Infection : Source : Gut flora Organisms : Escherichia coli , Klebsiella pneumonia , Enterococcus sp.

INDICATIONS : 1. Infective necrosis 2. Sterile necrosis > 50% 3. Extrapancreatic infections

• Preferred antibiotics : 1. Carbapenem ( Imipenem+cilastatin) 2. Quinolones 3. Metronidazole 4. 3rd generation cephalosporines

 ERCP

Early ERCP (endoscopic retrograde cholangiopancreatography), performed within 24 to 72 hours of presentation, is known to reduce morbidity and mortality.

The indications for early ERCP are as follows :

• Concomitant cholangitis • Significant persistent biliary obstruction (bilirubin > 5 mg/ dl) • ERCP in severe biliary pancreatitis without biliary sepsis or obstruction

The disadvantages of ERCP are as follows:

ERCP precipitates pancreatitis, and can introduce infection to sterile pancreatitis

The inherent risks of ERCP i.e. bleeding

It is worth noting that ERCP itself can be a cause of pancreatitis.

Currently, cholecystectomy is recommended for patients with gallstone-induced pancreatitis. ERCP with endoscopic sphincterotomy (ES) within 24 to 48 hours is also suggested for the treatment of acute gallstone pancreatitis.

2. b) D/D: Carcinoma of head and periampullary region of the pancreas.

Investigations:

Blood Tests:

 Patients with pancreatic head lesions frequently have elevated bilirubin and alkaline phosphatase levels suggestive of obstructive jaundice.  Other routine laboratory studies are usually normal.  Haemoglobin may be low.  The two most widely used pancreatic cancer serum markers are the CEA and the Lewis blood group carbohydrate antigen CA 19-9. Both are frequently elevated in patients with advanced disease. Extremely high levels of either CA 19-9 or CEA usually indicate unresectable or metastatic disease.

Imaging Studies:

 The main imaging modalities used for patients with suspected pancreatic neoplasms include right upper quadrant ultrasonography, CT, MRI with or without magnetic resonance cholangiopancreatography (MRCP), endoscopic retrograde cholangiopancreatography (ERCP), and percutaneous transhepatic cholangiography (PTC).  Right upper quadrant ultrasonography is a commonly used initial test. It can detect gallstones, dilatation of the biliary tree, and pericholecystic fluid. This imaging modality can also pick up hepatic metastases, pancreatic masses, peripancreatic and hilar lymphadenopathy, and ascites.  The investigation of choice in the work-up of patients suspected of a pancreatic cancer is a multidetector spiral CT and is probably the single most useful diagnostic and staging modality.

 CT provides more complete and accurate imaging of the pancreatic head and surrounding structures.  It gives very important information about the immediately adjacent vascular structures such as the portal, superior mesenteric, and splenic veins, as well as the superior mesenteric artery and celiac axis.  The involvement of periampullary lymph nodes and retroperitoneal structures may be demonstrated.  Additionally, information about distant metastatic disease can be gleaned if metastatic deposits are seen in the liver or in the peritoneal cavity.  The presence of ascites is usually an ominous sign.

 MRCP is now being utilized to image the biliary tree and the pancreatic duct. It has the advantage of being completely noninvasive.

 The vascular structures can also be visualized with the use of the contrast agent gadolinium and the performance of a magnetic resonance angiogram (MRA).  Thus a single (long) session in a scanner can provide information about tumor size and extent (MRI), the intraductal anatomy of the biliary and pancreatic system (MRCP), and the status of the nearby vasculature (MRA).  The resulting scan has the potential to provide information about tumor size and extent, biliary and pancreatic ductal anatomy, and vascular involvement through a single, noninvasive procedure.

 ERCP sometimes is required to solidify the diagnosis of pancreatic cancer.

 The classic findings of a long, irregular stricture in a pancreatic duct with distal dilation or a "double duct sign" in which there is cutoff of both the pancreatic duct and distal bile duct at the level of the genu of the pancreatic duct are pathognomonic  ERCP may be of benefit in patients with biliary obstruction and cholangitis whereupon an endoscopic stent can be placed for decompression.  ERCP is most useful when there is pancreatic duct obstruction, but no mass is evident on either CT or MRI. In this situation, it is necessary to try to distinguish chronic pancreatitis from pancreatic cancer.

 EUS is one of the most common imaging procedures used to diagnose pancreatic cancer.

 It is often the best procedure to obtain samples of a tumor to make a definitive diagnosis of pancreatic cancer.  EUS may be able to find small pancreatic masses that have not been detected by computed tomography (CT) or magnetic resonance imaging (MRI) scans but suspected by the doctor as a result of symptoms and/or blood test results.  Studies show that EUS is equal to or better than CT scans for detection of early pancreatic cancer.  Biopsy: Biopsy to confirm the presence and identify the type of cancer is usually required before chemoradiation therapy of unresectable pancreatic tumors or neoadjuvant treatment of resectable tumors. Percutaneous biopsy, performed with either CT or ultrasound guidance, or transduodenal biopsy, performed with endoscopic ultrasound guidance, is routinely employed in these situations.

Preoperative preparation of a case of obstructive jaundice.

 Proper diagnosis and assessment.  Injection vitamin K 1M 10 mg for 5 days.  Fresh Frozen plasma‐often requires 6 bottles or more.  Blood transfusion in case of anaemia.  Oral neomycin, lactulose Mannitol 100‐200 ml BD IV to prevent hepato renal syndrome.  Hydration  Repeated monitoring by doing prothrombin time,electrolytes.  Antibiotics like third generation cephalosporins.  Calcium supplements as calcium chloride IV.

Treatment and prognosis  Most tumours are not resectable at diagnosis.  Surgery for stage I and II does offer the chance of cure, with however high morbidity (20-30%) and mortality (5%). Resection is performed with a Whipple operation.  Even when resection is possible, the majority of patients succumb to recurrence, with only a doubling of survival in operated patients from 5 to 10% at 5 years. Almost a quarter of patients are dead 12 months following diagnosis.

Palliative Nonsurgical Treatment of Pancreatic Cancers

 Establishing the diagnosis and relieving symptoms of jaundice, gastric outlet obstruction, and pain are the goals of palliative nonsurgical treatment.  Tissue diagnosis can usually be made by CT- or ultrasound-guided percutaneous fine- needle aspiration of either the tumor or its metastases.  Transduodenal fine-needle aspiration of the tumor with endoscopic ultrasound guidance and duct cytology obtained by brushings is an alternative method of establishing the diagnosis.  Decompression of the obstructed biliary tract can be achieved using either an endoscopic or a percutaneous-transhepatic approach.  Pancreatic tumors can extend into and obstruct the duodenum, leading to gastric outlet obstruction. This commonly occurs in the second portion of the duodenum in patients with pancreatic head cancers. Pancreatic body tumors can invade the third or fourth portion of the duodenum and also cause obstruction. Many of these patients can be palliated by endoscopic placement of expandable endoluminal metal stents into the duodenum.  Pain, which is a common symptom of pancreatic cancer, is usually caused by tumor invasion of the peripancreatic neural plexus. Most patients can be adequately treated with orally or transcutaneously administered analgesics.  Narcotic medications may be required. When or if this fails, percutaneous CT-guided or endoscopic ultrasound-guided celiac plexus block may be helpful.

Palliative Surgical Management of Pancreatic Cancer

 Most of the symptoms experienced by patients with unresectable pancreatic cancer can be relieved by nonsurgical means.  Surgical palliation is, for the most part, employed for patients who are undergoing laparotomy for anticipated resectable disease and found to be unresectable at the time of surgery.  In that situation, biliary tract decompression can be achieved by creating either a cholecystojejunostomy or a choledochojejunostomy.  Duodenal obstruction can be managed by creation of a side-to-side gastrojejunostomy in which an antecolic jejunal loop is anastomosed to the posterior wall of the gastric antrum.  Palliation of pain can be achieved, intraoperatively, by injecting alcohol into the celiac plexus, and some surgeons routinely perform operative celiac plexus block at the time of surgical palliation.  However, many patients with unresectable pancreatic cancer can be successfully managed with minimal or no narcotic analgesics, and when more severe pain occurs, results similar to those achieved by intraoperative chemical splanchnicectomy can be achieved using a percutaneous approach.

Chemoradiation Therapy

 Many different protocols for chemoradiation treatment of recurrent or unresectable pancreatic cancer have been described. The best results have been achieved using radiation therapy combined with either 5- fluorouracil or gemcitabine.

3. a) Answer. Subphrenic abscess:

Subphrenic abscess, also known as subdiaphragmatic or infradiaphragmatic abscess, is localized accumulation of pus in the subphrenic spaces.

Clinical presentation  Fever  Upper quadrant pain  Chills  Leukocytosis

Pathology Subphrenic abscess may be caused by:  Perforation of hollow viscus (for example, , colon)  Inflammatory process (pancreatitis, cholecystitis, hepatic abscess, etc.)  Trauma of internal organs (stab, gunshots, etc.)  as complication of surgical operations (splenectomy, cholecystectomy, gastrectomy, etc.), most commonly in 1-3 weeks after surgery  Idiopathic (no obvious reason)

Radiographic features Subphrenic abscess may be right-sided (between right hemidiaphragm and liver) or left- sided (between left hemidiaphragm and spleen), most often is right-sided.

In general, there is infradiaphragmatic localized collection of fluid with air bubbles or air- fluid level which has an enhancing wall. It is often combined with elevation of hemidiaphragm and pleural effusion.

Plain radiograph Often the only finding on plain radiograph is elevation of hemidiaphragm. Also infradiaphragmatic collection of air (if present), pleural effusion can be detected.

Ultrasound Nonhomogeneous fluid-density mass in the infradiaphragmatic spaces which may contains echogenic debris, gas.

CT CT is good modality for detection of subphrenic abscess, estimating the volume. It allows to evaluate condititon of adjacent organs and possible reason of abscess (e.g. perforation) and his complications.

Treatment and prognosis Without adequate treatment subphrenic abscess could lead to peritonitis, sepsis and death.

Treatment options include combination of antibacterial therapy and surgical drainage.

3. b) Answer. Complications of Splenectomy:

General complications:  Bleeding.  Wound infection.

Immediate complications specific to splenectomy:

 Haemorrhage resulting from a slipped ligature.  Haematemesis from gastric mucosal damage and gastric dilatation is uncommon.  Left basal atelectasis is common, and a pleural effusion may be present.  Adjacent structures at risk during the procedure include the stomach and pancreas. o A fistula may result from damage to the greater curvature of the stomach during ligation of the short gastric vessels. o Damage to the tail of the pancreas may result in pancreatitis, a localised abscess or a pancreatic fistula.  Postoperative thrombocytosis may arise and, if the blood platelet count exceeds 1 × 106/mL, prophylactic aspirin is recommended to prevent axillary or other venous thrombosis.  Post-splenectomy septicaemia may result from Streptococcus pneumoniae, Neisseria meningitides, Haemophilus influenzae and Escherichia coli. The risk is greater in the young patient, in splenectomised patients treated with chemoradiotherapy and in patients who have undergone splenectomy for thalassaemia, sickle cell disease and autoimmune anaemia or thrombocytopenia.  Opportunist post-splenectomy infection (OPSI) is a major concern. Published guidelines emphasise that most infections after splenectomy could be avoided through measures that include offering patients appropriate and timely immunisation, antibiotic prophylaxis, education and prompt treatment of infection.  All patients with compromised immune function should receive prophylaxis. Satisfactory oral prophylaxis can be obtained with penicillin, erythromycin or amoxicillin, or co-amoxiclav.  Suspected infection can be treated intravenously with these same antibiotics and cefotaxime, ceftriaxone or chloramphenicol in patients allergic to penicillin and cephalosporins.  If elective splenectomy is planned, consideration should be given to vaccinating against pneumococcus, meningococcus C (both repeated every five years) and H.influenza type B (Hib) (repeated every ten years).  Haemophilus influenzae type b vaccination is recommended irrespective of age.  Patients who have undergone splenectomy and are travelling to countries where malaria is present are strongly advised to use all physical anti-mosquito barriers, as well as anti-malarial therapy, since they are at increased risk of severe malaria.  Overwhelming post-splectomy sepsis due to Capnocytophaga canimorsus may result from dog, cat or other animal bites.  In the trauma victim, vaccination can be given in the postoperative period, and the resulting antibody levels will be protective in the majority of cases.

3. c) Answer. Femoral hernia:

Femoral canal: The major feature of the femoral canal is the femoral sheath. This sheath is a condensation of the deep fascia (fascia lata) of the thigh and contains, from lateral to medial, the femoral artery, femoral vein, and femoral canal. The femoral canal is a space medial to the vein that allows for venous expansion and contains a lymph node (node of Cloquet). Other features of the femoral triangle include the femoral nerve, which lies lateral to the sheath.

Wall of the Femoral canal:

 Anterior is the inguinal ligament  Posterior is the iliopsoas, pectineal, and long adductor muscles (floor).  Medial is lacunar ligament  Lateral is femoral vessle

Femoral Hernia: Hernia medial to femoral vessels under inguinal ligament Small femoral hernia may be unnoticed by the patient or disregarded for years perhaps until the day it strangulates. Adherence of the greater omentum sometimes causes a dragging pain. Rarely a large sac is present.

 Age: uncommon in children , most common in old age female .

 Sex; women > men (but still commonest hernia in women the inguinal hernia )

 The patient came with local symptoms

- Discomfort and pain

- Swelling in the groin

 General ; femoral hernia is more likely to be strangulated than the inguinal hernia

 Multiplicity ; often bilateral

Femoral hernia versus inguinal hernia:

Inguinal hernia Femoral hernia

1- more common in male 1- more common in females

2- pass through the inguinal canal 2- pass through the femoral canal

3- neck of the sac is above and medial the 3- neck of the sac is below and lateral the pubic tubercle pubic tubercle

4- less common to be strangulated 4- more common to be strangulated

5- can be treated without surgery 5- must be treated surgically

6- the two diagnostic signs of hernia + 6- the two diagnostic signs of hernia -

7- the sac mainly contain ; bowel 7- the sac mainly contains ; omentum

Diagnosis:

 The diagnosis is largely a clinical one, generally done by physical examination of the groin. However, in obese patients, imaging in the form of ultrasonography, CT or MRI may aid in the diagnosis. An abdominal x-ray showing small bowel obstruction in a female patient with a painful groin lump needs no further investigation.  Several other conditions have a similar presentation and must be considered when forming the diagnosis: inguinal hernia, an enlarged inguinal lymph node, aneurysm of the femoral artery, saphena varix, and an abscess of the psoas.

Management:

Femoral hernias, like most other hernias, usually need operative intervention. This should ideally be done as an elective (non-emergency) procedure. However, because of the high incidence of complications, femoral hernias often need emergency surgery. Surgery:  Laparoscopic surgery  Open surgery: Three approaches have been described for open surgery:

 Lockwood’s infra-inguinal approach  Lotheissen‘s trans-inguinal approach  McEvedy’s high approach The infra-inguinal approach is the preferred method for elective repair. The trans-inguinal approach involves dissecting through the inguinal canal and carries the risk of weakening the inguinal canal. McEvedy’s approach is preferred in the emergency setting when strangulation is suspected. This allows better access to and visualization of the bowel for possible resection. In any approach, care should be taken to avoid injury to the urinary bladder which is often a part of the medial part of the hernial sac. Repair is either performed by suturing the inguinal ligament to the pectineal ligament using strong non-absorbable sutures or by placing a mesh plug in the femoral ring. With either technique care should be taken to avoid any pressure on the femoral vein.

3.d) Answer. Adenomatous polyps of colon:

Introduction: Adenomatous polyps (adenomas) of the colon and are benign (noncancerous) growths, but may be precursor lesions to colorectal cancer. Polyps greater than one centimeter in diameter are associated with a greater risk of cancer. If polyps are not removed, they continue to grow and can become cancerous.

Definition: A colorectal polyp is any mass projecting into the lumen of the bowel above the surface of the intestinal epithelium.

Classification: Polyps arising from the intestinal mucosa are generally classified by their gross appearance as pedunculated (with a stalk) or sessile (flat, without a stalk). They are further classified by their histologic appearance as tubular adenoma (with branched tubular glands), villous adenoma (with long finger-like projections of the surface epithelium),or tubulovillous adenoma (with elements of both cellular patterns).

The most common benign polyp is the tubular adenoma, constituting about 65% to 80% of all polyps removed. About 10% to 25% of polyps are tubulovillous, and 5% to 10% are villous adenomas.

Tubular adenomas are most often pedunculated; villous adenomas are more commonly sessile. The degree of cellular atypia is variable across the span of polyps, but there is generally less atypia in tubular adenomas, and severe atypia or dysplasia (precancerous cellular change) is found more often in villous adenomas.

The incidence of invasive carcinoma being found in a polyp is dependent on the size and histologic type of the polyp.

There is less than a 5% incidence of carcinoma in an adenomatous polyp less than 1 cm in size, whereas there is a 50% chance that a villous adenoma greater than 2 cm in size will contain a cancer.

Management:

The treatment of an adenomatous or villous polyp is removal, usually by colonoscopy. The presence of any polypoid lesion is an indication for a complete colonoscopy and polypectomy, if feasible. Polyps on a stalk are often removed by a snare passed through the colonoscope, whereas sessile (flat) polyps present technical problems with this technique because of danger of perforation associated with the snare technique. Although it may be feasible to elevate the sessile polyp from the underlying muscularis with saline injection, permitting subsequent endoscopic excision, sessile lesions often require segmental colectomy for complete removal.

3. e) Answer. Tuberculous cervical lymphadenitis, also known as scrofula and King's evil, continues to be seen in endemic areas and in the industrialised world particularly among the immunocompromised.

Epidemiology

Tuberculous cervical lymphadenitis is the most common manifestation of extrapulmonary tuberculosis and is a very frequent cause of a peripheral lymphadenitis in the developing world. Additionally, in industrialised nations, there is a resurgence among intravenous drug users and the immunocompromised population, especially those due to HIV.

Most frequently children and young adults are affected (11-30 years of age) and there may be a slight female predilection. Clinical presentation: The presentation is, usually, with one or more cervical masses. Nodes may be hard or fluctuant, but unlike a suppurative bacterial lymphadenitis, they tend not to be particularly tender and only have limited inflammatory changes in the overlying skin. If undiagnosed or untreated, spontaneous discharge may eventually occur.

Cervical nodes are the most commonly affected nodes in tuberculous lymphadenitis, accounting for approximately 63% of cases, followed by mediastinal (27%) and axillary nodes (8%).

Within the neck certain lymph node groups are more frequently involved than others, with a predilection for nodes in the posterior triangle (51%) and deep upper cervical (48%). In the majority of cases, lyphadenitis is unilateral.

Pathology

Affected nodes demonstrate central caseation, characteristic of mycobacterial infections, which appears as a creamy to chalky off-white regions 3. It is believed that lymphadenitis most likely represents post-primary reactivation of Mycobacterium tuberculosis previously spread haematogeneously during primary infection. Mycobacterium avium intracellulare accounts for 10% of cases which is unlike tuberculosis tends to be unilateral and commonly seen without any chest infection.

Radiographic features

Imaging alone is often unable to categorically distinguish tuberculous lymphadenitis from other causes of cervical lymphadenopathy and necrotic/cystic lymphadenopathy. It is, therefore, important to interpret imaging findings with a knowledge of the patient's demographics. Interestingly less than 50% of patients with tuberculous cervical lymphadenitis demonstrate abnormalities on chest radiographs.

Ultrasound

Ultrasound is an excellent first-line investigation as it is not only able to assess cervical lymphadenopathy but also enables guided fine needle aspiration cytology. The combination of grey-scale imaging and FNAC as a sensitivity of 92% and specificity 97% in distinguishing benign from malignant nodal disease.

Grey scale features that suggest the diagnosis of tuberculous lymphadenitis above malignancy (the main differential-see below) include:

 nodal matting  surrounding soft tissue oedema (less marked than one would expect given the size of the collections)

Doppler examination is particularly useful in helping distinguish tuberculous infection from necrotic metastatic disease. Reactive nodes (including those in tuberculous lymphadenitis) demonstrate prominent vascularity, but mostly confined to the hilum, whereas malignant nodes demonstrate more peripheral/capsular vascularity CT: CT appearances of tuberculous lymphadenitis are variable, depending on the degree of caseation present in the node. Nodes may initially appear merely enlarged, often with attenuation similar to muscle. Eventually, central caseation develops and the nodes become centrally low density and eventually frankly cystic. They are, usually, matted together with only minor surrounding inflammatory changes.

MRI: MRI appearances are similar to those of CT, ranging from homogeneously enlarged nodes to cystic transformation with peripheral enhancement.

Treatment and prognosis

Treatment is with prolonged courses of multi-agent antimycobacterials and in some instances (after many months of medical management) surgical excision of residual nodal masses.

Percutaneous drainage should be avoided prior to medical management as it is liable to create fistulae.

It is important to note that during therapy new or existing cervical nodes may enlarge. This should not be mistaken for a failure of medical management, and such enlargement is often transient.

4. a) Cubitus valgus:

Cubitus valgus is a medical deformity in which the forearm is angled away from the body to a greater degree than normal when fully extended. A small degree of cubitus valgus (known as the carrying angle) is acceptable and occurs in the general population.  When present at birth, it can be an indication of Turner syndrome or Noonan syndrome.  It can also be acquired through fracture or other trauma.  The physiological cubitus valgus varies from 3° to 29°.  Women usually have a more pronounced Cubitus valgus than men.  The deformity can also occur as a complication of fracture of the lateral condyle of the humerus, which may lead to tardy/delayed ulnar nerve palsy. Treatment::  Hormone therapy  Osteotomy  Fixation

4. b) Fracture healing:

A fracture occurs when the continuity of a bone is broken and local blood supply is interrupted. If the overlying soft tissues are also injured, fracture healing may be delayed or disrupted, particularly in anatomic regions with decreased vascular networks such as the tibial diaphysis. Bone is unique in its ability to regenerate itself. Healing occurs via reactivation of embryologic processes resulting in the formation of bone not scar. Fracture healing can be described in three conceptual stages. An understanding of the timing and mechanisms associated with each stage is important in planning fracture treatment. The stages are:

Inflammation This begins immediately after bone injury with the formation of a local hematoma or fibrin clot. There is local cell death where vessel disruption has resulted in ischemia - usually at the very ends of the fractured bone. Over the course of the next few days, this area becomes infiltrated by inflammatory cells and is characterized by local swelling and warmth. The inflammatory cells release lysosomal enzymes and other mediators that attract pluripotent cells to the area; they also act to remove necrotic tissue. Fibroblasts, mesenchymal cells and osteoprogenitor cells appear and may transform nearby tissues. The fracture is tender and may be grossly mobile to physical examination at this stage. Inflammation is at its peak 48 hours after a fracture.

Repair The reparative phase begins a few days after the injury with the arrival of mesenchymal cells able to differentiate into fibroblasts, chondroblasts and osteoblasts. The repair phase persists for several months; it can be divided into two distinct phases: soft and hard callus formation.

1. "Soft callus" formation lasts for approximately six weeks from the time of injury. During this preliminary stage of repair, pain and swelling subside and bony fragments become united by fibrous and cartilagenous tissue. Woven bone is formed. While this creates some stability, the fracture may still angulate at this stage if not held with stable external support, such as a cast or external fixator, or internal support provided by plates, screws or intramedullary devices. 2. "Hard callus" formation - During this second stage of repair, woven bone is transformed into lamellar bone. This takes approximately three months.

Remodeling

Remodeling is the process by which bone is removed in tiny increments and then replaced by new bone. After a fracture, remodeling may continue for months or even years. The adult human skeleton continuously replaces itself at rate of 10-18% per year. The rate of remodeling is accelerated in children and during fracture repair. In addition to being an essential part of fracture healing, remodeling plays an important role in calcium homeostasis. During the remodeling phase the woven bone is converted to lamellar bone and the medullary canal is reconstituted. During this phase, bone responds to loading characteristics according to Wolff's law. Some angular deformity may correct during this stage in children with sufficient growth remaining (up to 5o per year of growth remaining).

Factors affecting fracture healing:

 Blood Supply. Soft tissue injury. Radiation. Chemical or thermal burns. ...  Excessive Movement. Inadequate immobilisation. Inadequate fixation or compliance.  Gap. Intact fellow bone. Interposed soft tissue. ...  Other. Nutrition (Vit C required for normal collagen) Drugs (corticosteriods inhibit osteoblast differentiation.

4. c) Osteosarcoma.

Introduction:

 20% of all primary bone tumor second-most common primary malignancy of bone  Incidence: 1 to 3 per million per year.  Male: female—1.6:1(except parosteal variety) • age: conventional—2nd decade Site aroud the knee jt.(arising mainly from metaphysis;intramedullary region) 52% --lower end of femur 20%-- upper end of tibia 9% -- upper end of humerus.

Predisposing factors:

• radiation • viral infection: plyoma virus/harvey virus • chemicals:beryllium 20-methyl cholanthrene

Clinical features:

Presenting features: - pain(night pain) -sometimes only tiredness & limp -palpable mass - skin conditions to be examined carefully • h/o trauma sometimes draws attention.

Associated features: • effusion & swelling of nearby joints • fever • pallor & cachexia • regional lymph node • features associated with pulmonary metastasis • pathological #

Distal neurovascular deficits and pressure symptoms ….may be associated with the tumour

Classification: • primary osteosarcoma • secondary osteosarcoma Pathology:

Macroscopy: typical osteosarcoma presents as a large illdefined lesion in the metaphyseal region of the involved bone. It typically destroys cortex and frequently extends inwards marrow cavity and outwards into the adjacent soft tissue.

Macroscopic tumour often elevates periosteum to produce codman’s triangle on radiograph. It also produces sunray appearance due to vessels which pass from the periosteum to the cortex & along which bone is laid down & some of the new bone may be reactionary.

Large ill-defined lesion in the metaphyseal region of long bone • leg of mutton’ appearance • stony-hard to soft and gritty in consistency • areas of haemorrhage& necrosis • colour: white : yellow : bluish white: fibroblastic osteoblasticcartilagenous • codman’s triangle ---due to subperiosteal new bone formation • sunray appearance ---due to bone deposition in sub- periosteal space along the vessels. Sunray appearence

Pathology:

Microscopy; Lichtensten’s criteria to identify osteosarcoma : 1)sarcomatousstroma 2)spindle cells. 3) direct formation of neoplastic osteoid and bone. Hallmark of osteosarcoma is the formation of osteoid by malignant mesenchymalcells .the neoplastic mesenchymal cells in between osteoid & cartilage elements may be spindle shaped and pleomorphic with bizarre hyperchromatic nuclei and frequent mitotic figures. giant cells may be present.

Radiologic investigations • plain radiograph(x-ray) •ct scan • mri scan • bone scan.

Radiology • arises in the metaphysial region of a long bone • outgrows from the medullary canal to extraskeletal region • displays representative features of a malignant lesion- permeative growth pattern/indistinct margins/cortical erosion • periosteal reaction with formation of codman’s triangle/sunburst appearance • wide variety of radiographic appearance like bone cyst

CT scan and MRI scan are not as instrumental as plain radiograph • bone scan is useful to detect metastasis

MRI: Excellent for describing lesions in the marrow cavity • helpful to determine the level of resection • useful for screening skip lesions • can detect medullary invasion in case of juxtacortical tumors • can detect epiphyseal involvement and penetration of physeal cartilage.

Management: multidisciplinary approach primary care physician orthopaedic surgeon radiation oncologist pathologist physiotherapist rehabilitation specialist social workers & others.

Treatment options: • chemotherapy • surgery • radiotherapy

Chemotherapy: • introduction of systemic chemotherapy has dramatically improved survival rates. • before the routine use of chemotherapy— treatment was immediate wide or radical amputation • 80% patients died of metastasis eventually, though metastasis was not evident on presentation.

• neo-adjuvant chemotherapy: ct administered before the surgical resection of primary tumour • adjuvant chemotherapy: ct administered postoperatively to treat presumed micro- metastasis • it shrinks the tumour mass , making it easier for operation • it decreases the spread of tumour cells during surgery, • t/t against potential micro-metastasis started immediately, (it also gives idea about responsiveness & effectiveness of the chemotherapeutic agent to the tumour)

Low grade osteosarcoma-- treated by surgery alone. high grade osteosarcoma-- treated by neo-adjuvant chemotherapy -surgery - adjuvant chemotherapy,

Common agents used doxorubicin, vincristine, methotrexate, cyclophosphamide &ifosfamide,dactinomycin

Radiation therapy: • role of radiotherapy is limited in the treatment of osteo-sarcoma --a relatively radio-resistant tumor. • radiation therapy can palliate pain from local recurrence and prevent need for amputation in patients who are presented with distant metastasis

Indications • post-operative -- where surgical margin is involved • palliation of pain from primary tumour in the presence of metastatic disease • radical treatment of inoperable sites (skull, vertebra, ilium, sacrum) • bilateral lung irradiation in pulmonary metastasis Method: • external beam radiation — by linear accelereter. • brachytherapy —limited role • iort – single dose,in specially prepared ot

Surgery is the mainstay of therapy • limb sacrificing surgery or • limb salvaging surgery. Principles of surgery choice between limb salvage surgery and amputation must be made on the basis of the expectations and desires of the individual patient and the family.

Limb salvage surgery has become an accepted standard of care for patients with skeletal malignancies including osteosarcoma • many patients who once would have had an amputation are now having their limb saved

Ttreatment of • pulmonary metastasis • local recurrence • secondary disease. 4. d) Supracondylar fracture of humerus. Distal Humerus Anatomy

 Medial epicondyle proximal to trochlea –

 Lateral epicondyle proximal to capitellum –

 Radial fossa – accommodates margin of radial head during flexion

 Coronoid fossa – accepts coronoid process of ulna during flexion

Introduction:

 It is # which involves the lower end of the humerus usually involving the thin portion of the humerus through o Olecranon fossa or o Just above the fossa or o Metaphysis  Most common elbow injuries in children.  Makes up approximately 60% of elbow injuries.  Becomes uncommon as the age increases.

General consideration:

Incidence of supracondylar #:

o Age: peak age: 5-7 yrs. Average age : 6.7 yrs o Sex: Boys = Girls (Latest Trends) o Side: Left > Right (Non dominant > dominant ) o Nerve injuries: 7% - Median> Radial > Ulnar o Vascular injuries: 1% o Cause of #  Fall from height 70% - children > 3 yrs  Fall from bed - children < 3 yrs  Non accidental injury (Child abuse) children < 15 months o Associated #s : Distal radius > Scaphoid > Proximal humerus > Monteggia o Clinical types:  Extension type: 98%  Flexion type : 2%  Open injuries : < 1%

Radiography:

 Radiographic views:  Antero posterior  Lateral  Oblique  Axial (jones view )

Clinical features:

 In most cases, children will not move the elbow if a fracture is present, although this may not be the case for non-displaced fractures.  Swelling about elbow is a constant feature, develop within first few hrs.

 S shaped deformity

 Distal humeral tenderness

 Anterior plucker sign +ve

 Nerve injury incidence is high, between 7 and 16 % (median, radial and ulnar nerve)

 Anterior interosseous nerve is most commonly injured nerve

 In many cases, assessment of nerve integrity is limited , because children can not always cooperate with the exam

 Carefully document pre manipulation exam, as post manipulation neurologic deficits can alter decision making

 Thorough documentation of all findings is important. A simple record of “neurovascular status is intact” is unacceptable.

 Individual assessment and recording of motor, sensory, and vascular function is essential

 Always palpate the arm and forearm for signs of compartment syndrome.

Treatment:

General principles:

 Splinting elbow in comfortable position -20-30degrees of flexion of elbow.  Careful physical examination & X-ray evaluation.  Tight bandaging/ excessive flexion or excessive extension should be avoided.  Associated life threatening complications (if any) to be attended first.

Type Treatment I  Simple immobilization with a post slab in 90deg. With a cuff and collar  Xray to be rpted at 5-7 days to document for any displacement  Slab kept for 3 weeks

II  Treatment – closed reduction under anaesthesia  Traction is applied followed by correction of rotational deformity  Extension deformity is corrected with pressure by thumb over the olecranon

III  Closed reduction & percutaneous K wire fixation  Open reduction & K wire fixation

Complications: Neurovascular complications (a) tear or entrapment of the brachial artery; (b) spasm of the artery and (c) compression of the artery relieved by manipulation of the fracture (d) compression of ulnar nerve. Causing Pink and Pulseless hand in supracondylar fracture. Thus there is loss of circulation of forearm, causing lack of reperfusion of tissues resulting in tissue death causing compartment syndrome. Therefore the complications of elbow dislocations include the following:

 Posttraumatic periarticular calcification.  Myositis ossificans or calcific tendinitis  Neurovascular injuries (8-21% of cases) — palsy to the anterior interosseus nerve at time of index injury is most common, followed by brachial artery injuries (5-13%). Injury to the ulnar nerve is reported with percutaneous pinning through the medial epicondyle.  Osteochondral defects, intra-articular loose bodies, and avascular necrosis of the capitulum  Instability

Other injuries:

 Most commonly brachial artery injury, and if left untreated could lead to Volkmann's contracture (permanent flexion contracture of the hand at the wrist, resulting in a claw- like deformity of the hand and fingers).

 Cubitus varus: Also known as gunstock deformity. This is more of a cosmetic problem, and does not affect the range of motion of the elbow joint.

 Malunion: The condyles of the distal humerus do not correctly join together. It may be misaligned anteriorly, posteriorly, medially or laterally in respect to the shaft of the humerus. A thicker ossification will also appear on the distal end. Remove it one year later to prevent additional ossification from forming.

4. e) Club foot.

Introduction:

Complicated, multifactorial deformity of primarily genetic origin

Three basic components:

(i) Ankle joint plantarflexed/equines

(ii) Subtalar joint inverted/varus

(iii) Forefoot adducted

Incidence:  Approx 1/1,000 live births  Usually sporadic  Bilateral deformities occur 50%

Etiology:

 Unknown  Defect in development of talus leads to Soft tissue changes in joints, or vice versa

Diagnosis/Evaluation:

 Distinguish mild/severe forms from other disease  AP/Lat standing or AP/stress dorsiflex lat films

Treatment:

 Non-surgical  Weekly serial manipulation and casting - must follow certain order of correction - success rate 15-80%  Physiotherapy - The aim is to stretch the ligaments and tendons into the correct position.  Strapping - Strips of adhesive strapping are passed around the foot, up the sides of the leg, and over the top of the knee, to hold the foot in a corrected position. This is usually done weekly, following some physiotherapy.  Plaster fixation - The surgeon manipulates the foot into position, and holds it in place with plaster. This needs to be repeated about every week for 3 to 6 months.  Ponseti Method - The treatment involves weekly stretching of the foot deformity in the clinic, followed by the application of long leg plaster casts. The cast is changed every 1 or 2 weeks. The physician may performs a tenotomy, an Achilles tendon lengthening using non-invasive surgery.  Splinting - There are different types of splint available that may be worn just at night, or for most of the time. With traditional nonoperative treatment, splintage begins at 2-3 days after birth.

The order of correction is as follows:

1. Forefoot adduction 2. Forefoot supination 3. Equinus

 Surgical: majority do well; calf and foot is smaller. Any approach should afford adequate exposure. Structures to be released or lengthened are the following: o Achilles tendon (ETA) o Tendon sheaths of the muscles crossing the subtalar joint o Posterior ankle capsule and deltoid ligament o Inferior tibiofibular ligament o Fibulocalcaneal ligament o Capsules of the talonavicular and subtalar joints o Division of associated ligaments around the subtalar joint o Plantar fascia and intrinsic muscles Surgical treatment should take into account the age of the patient.

o In children younger than 5 years, correction can be achieved with soft-tissue procedures. o Children older than 5 years require bony reshaping (eg, dorsolateral wedge excision of the calcaneocuboid joint [Dillwyn Evans procedure] or osteotomy of the calcaneum to correct varus). o Lateral wedge tarsectomy or triple fusion (arthrodesis) is required if the child is older than 10 years (salvage procedures).

4. e) Supracondylar fracture of humerus. Distal Humerus Anatomy

 Medial epicondyle proximal to trochlea –

 Lateral epicondyle proximal to capitellum –

 Radial fossa – accommodates margin of radial head during flexion

 Coronoid fossa – accepts coronoid process of ulna during flexion

Introduction:

 It is # which involves the lower end of the humerus usually involving the thin portion of the humerus through o Olecranon fossa or o Just above the fossa or o Metaphysis  Most common elbow injuries in children.  Makes up approximately 60% of elbow injuries.  Becomes uncommon as the age increases.

General consideration:

o Incidence of supracondylar #:

a) Age: peak age: 5-7 yrs

Average age : 6.7 yrs

b) Sex: Boys = Girls (Latest Trends)

c) Side: Left > Right

( Non dominant > dominant )

d) Nerve injurie:7% - Median> Radial > Ulnar

e) Vascular injuries:1%

g) Cause of #

 Fall from height 70% ----- children > 3 yrs  Fall from bed children < 3 yrs  Non accidental injury (Child abuse) children < 15 months

h) Associated #s

Distal radius > Scaphoid > Proximal humerus > Monteggia

i) Clinical types

Extension type: 98%

Flexion type : 2%

Open injuries : < 1%

Mechanism of injury:

 For Extension type of SC # humerus

Fall on outstretched hand Elbow hyper extended Fore arm – pronated or supinated

 For Flexion type of SC # humerus

Fall directly on the elbow rather than out stretched hand

Radiography:  Radiographic views:

Antero posterior

Lateral

Oblique

Axial ( jones view )

Clinical features:  In most cases, children will not move the elbow if a fracture is present, although this may not be the case for non-displaced fractures.

 Swelling about elbow is a constant feature, develop within first few hrs.

 S shaped deformity

 Distal humeral tenderness

 Anterior plucker sign +ve

 Nerve injury incidence is high, between 7 and 16 % (median, radial and ulnar nerve)

 Anterior interosseous nerve is most commonly injured nerve

 In many cases, assessment of nerve integrity is limited , because children can not always cooperate with the exam

 Carefully document pre manipulation exam, as post manipulation neurologic deficits can alter decision making

 Thorough documentation of all findings is important. A simple record of “neurovascular status is intact” is unacceptable.

 Individual assessment and recording of motor, sensory, and vascular function is essential

 Always palpate the arm and forearm for signs of compartment syndrome.

Treatment:

General principles:

 Splinting elbow in comfortable position -20-30degrees of flexion of elbow.  Careful physical examination & X-ray evaluation.  Tight bandaging/ excessive flexion or excessive extension should be avoided.  Associated life threatening complications (if any) to be attended first.

Type Treatment I  Simple immobilization with a post slab in 90deg. With a cuff and collar  Xray to be rpted at 5-7 days to document for any displacement  Slab kept for 3 weeks

II  Treatment – closed reduction under anaesthesia  Traction is applied followed by correction of rotational deformity  Extension deformity is corrected with pressure by thumb over the olecranon

III  Closed reduction & percutaneous K wire fixation  Open reduction & K wire fixation

Complications:

Neurovascular complications (a) tear or entrapment of the brachial artery; (b) spasm of the artery and (c) compression of the artery relieved by manipulation of the fracture (d) compression of ulnar nerve. Causing Pink and Pulseless hand in supracondylar fracture. Thus there is loss of circulation of forearm, causing lack of reperfusion of tissues resulting in tissue death causing compartment syndrome. Therefore the complications of elbow dislocations include the following:

 Posttraumatic periarticular calcification.  Myositis ossificans or calcific tendinitis  Neurovascular injuries (8-21% of cases) — palsy to the anterior interosseus nerve at time of index injury is most common, followed by brachial artery injuries (5-13%). Injury to the ulnar nerve is reported with percutaneous pinning through the medial epicondyle.  Osteochondral defects, intra-articular loose bodies, and avascular necrosis of the capitulum  Instability

Other injuries:

 Most commonly brachial artery injury, and if left untreated could lead to Volkmann's contracture (permanent flexion contracture of the hand at the wrist, resulting in a claw- like deformity of the hand and fingers).

 Cubitus varus: Also known as gunstock deformity. This is more of a cosmetic problem, and does not affect the range of motion of the elbow joint.

 Malunion: The condyles of the distal humerus do not correctly join together. It may be misaligned anteriorly, posteriorly, medially or laterally in respect to the shaft of the humerus. A thicker ossification will also appear on the distal end. Remove it one year later to prevent additional ossification from forming.

The West Bengal University of Health Sciences

M.B.B.S. 3rd Professional Part – II Examination, 2018

Subject: Surgery Time: 21/2 hrs.

Paper: I Marks: 60

Group - A

1. A 40 years old man presents with a non-healing ulcer over the lower part of his inferior extremity. What are the causes? How will you investigate the case? What are the surgical considerations of diabetic foot ulcer? (No operative details) 3+7+5

Group – B 2. a) A 25 years old female, recently married, presents with sudden pain over the right lower abdomen. How will you take up the case to come at a diagnosis? Outline the management of acute appendicitis. (No operative details) 10+5

Or

b) What are the causes of intra-abdominal lump in the region of epigastrium? Discuss the management of Hydatid cyst of liver. 5+10

Group – C 3. Answer in brief on any three of the following: 3x5 a) Diagnostic peritoneal lavage b) Colostomy c) Molecular subtypes of breast carcinoma d) Intussusception e) Amoebic liver abscess Group – D 4. Write short notes (any three): 3x5 a) Myositis ossificans b) Fracture patella c) Complications of Colles’ fracture d) Pathological fracture e) Giant cell tumour

Answers.

1. Causes of non healing ulcer in lower limb:

Some of the major causes are listed under:

 Vascular congestion  Peripheral vascular disease  Infection (mostly due to resistant bacteria)  Poor nutritional  Diabetes mellitus  Some systemic disease such as SLE (Systemic Lupus)  Unknown causes Types of Non-Healing Ulcers

There are a few types of Non-healing ulcers:

 Pressure ulcers  Venous stasis ulcer  Diabetic Ulcer  Arterial ulcer  Pyoderma gangrenosum  Marjolin's ulcer

Investigations

 General: • Blood : Routine examination of blood including a hemoglobin percent (low Hb% can decrease claudication distances and aggravate rest pain), blood sugar examination as diabetics have worse prognosis, are essential. • Erythrocyte sedimentation rate (ESR) is usually raised in Buerger‘s disease. • In patients with high suspicion of underlying connective tissue disorders, specific test like RA factor, LE cell phenomenon etc. may be carried out. Lipid profile is mandatory in elderly patients with atherosclerosis. • Urine examination for sugar. • Plain X-ray of the abdomen will show the presence of arterial calcification and flecks of calcium may outline an aneurysm. • ECG: an abnormality in ECG may influence the decision for surgery, in patients with lower limb disease.

 Tests of global Vascular Status: • Hand Held Doppler ultrasound: blood flow detection uses a continuous wave ultrasound signal, beamed at an artery and the reflected beam is picked up by a receiver. The changes of frequency in the reflected beam, as compared with the transmitted beam, are due to the―Doppler shift, resulting from passage of beam through moving blood. These frequency changes are converted to audio signals. This investigation may be used effectively in cases where a differential diagnosis of atherosclerosis is entertained showing the site of block and extent of distal run-off. A 8-10 MHz continuous waver Doppler ultrasound probe is used to assess the posterior tibial (PT), dorsalis pedis (DP) and peroneal arteries. A normal pedal pulse produces a Doppler signal with 2 or 3 distinct phases with a clear sharp sounding systolic peak. The reduction in the pitch of this signal and a lack of the phasic components is recongnised as abnormal finding. In low arterial flow conditions, it becomes difficult to differentiate between arterial from venous flow. In such a situation, application of compression in the distal part of the limb would tend to augment in case of a venous signal but will leave the arterial signal unaffected.

• Ankle Brachial systolic blood pressure index (ABPI): This measurement gives the quantitative assessment of the global limb arterial perfusion. Normally an average of 2 measurement for each of the arteries namely dorsalis pedis and posterior tibial is used to denote the ankle pressure. Similar procedure is done for brachial artery. The ratio of the highest recorded systolic pressure at the ankle to the brachial systolic pressure at the arm forms the ABPI. The ankle brachial index is interpreted in the manner as given in the table below. As a thumb rule, patients with ABPI of 0.8 or 0.9 have moderate peripheral vascular disease, those between 0.5 to 0.8 suffer from intermittent claudication and those with < 0.5 may suffer from rest pain, ulceration or gangrene.

Ankle Brachial Pressure Index

Segmental pressures, i.e. differences in arterial blood pressure between segments of limb can be detected to give indication of the sites of stenosis, specially as Buerger‘s is said to be a segmental disease.

• Toe Pressures Using Photoplethysmograph: These are used when the arterial disease is suspected between the ankle and the toes.

• Pole Test: This is used to determine the adequacy of lower limb blood flow in patients with incompressible vessels or who are unable to tolerate an ankle pressure cuff. It involves listening to the pedal pulse using the hand held Doppler probe while the patients leg is raised against a pole. The height at which the Doppler signal disappears is reflected by the markings in mm Hg in a caliberated pole directly. A maximum of 60-70 mm Hg only can be recorded since the test is limited by the height upto which the patient is able to lift the leg.

• Transcutaneous Oximtery (TcPO2): It is based on the principle that the partial pressure of the oxygen which diffuses through to the surface of the skin reflects the oxygen tension of the underlying tissues.

• Walk Test: The basis of this test is that measurement of ABPI before and after a patient has walked can expose less severe or compensated peripheral vascular disease. The walk can be standard or graded ( incline) using a tread mill. A reduction in ABPI of >20% indicates presence of severe arterial disease.

 Tests for disease localization: • Duplex imaging: gives accurate information on the size of artery, the flow rate, turbulence and the presence of stenosis. The combination of Doppler and color mapping allows easy recognition of stenotic sites. This has been achieved by the use of pulsed or continuous wave Doppler and the two- dimensional images produced by the B- scan made either singly or in combination. Peak systolic velocity at the site of stenosis is compared to that measured proximally to obtain a peak systolic velocity ratio (PVR) and relates to the degree of stenosis. A 2x increase in PVR at a stenosis corresponds to 50% reduction in diameter on arteriography. This investigation has virtually become the first line investigation to localize the level of block with a great deal of accuracy. • Intravascular Ultrasound: Minute ultrasound probes with 10MHz transducers mounted on tips of small 3-4 F catheters are placed directly in the lumen of the arteries over a guidewire and produce intravascular ultrasound images which give details of arterial walls, luminal contents and dimensions. • Arteriography: This is an invasive technique which though has become much safer in the recent years due to fine 3-4 F catheters, and remains the gold standard to provide a road map required for vascualt surgeons expecially before surgery is planned. • CT Angiography: The introduction of the helical (spiral) CT scanning and multidetector CT which uses 2 or 4 helicals to scan the patient, CT imaging has been revolutionized for vascular imaging wherein a single breathhold time is sufficient to generate the scans from the aortic arch to the groins with imaging quality as good as conventional angiography. • MR Angiography: Increasing usage of Magentic Resonance Imaging in the last decade and improvement in technology has seen a shift towards using this modality for assessment of vascular system.

Management of diabetic foot ulcer:

Successful diagnosis and treatment of patients with DFUs involves a holistic approach that includes:

 Optimal diabetes control  Effective local wound care  Infection control  Pressure relieving strategies  Restoring pulsatile blood flow.

Classification and severity of diabetic foot infections:

The essential components of management are:

 Treating underlying disease processes  Ensuring adequate blood supply  Local wound care, including infection control  Pressure offloading

Treating the underlying disease processes:

 One should identify the underlying cause of the DFU during the patient assessment and, where possible, correct or eliminate it.  Treating any severe ischaemia is critical to wound healing, regardless of other interventions.  It is recommended that all patients with critical limb ischaemia, including rest pain, ulceration and tissue loss, should be referred for consideration of arterial reconstruction.  Achieving optimal diabetic control. This should involve tight glycaemic control and managing risk factors such as high blood pressure, hyperlipidaemia and smoking.  Nutritional deficiencies should also be managed.  When possible and appropriate one should check other footwear worn at home and at work (eg slippers and work boots). Ensuring adequate blood supply:

 A patient with acute limb ischaemia is a clinical emergency and may be at great risk if not managed in a timely and effective way.  It is important to appreciate that, aside from critical limb ischaemia, decreased perfusion or impaired circulation may be an indicator for revascularisation in order to achieve and maintain healing and to avoid or delay a future amputation.

Optimising local wound care:

 Emphasis in wound care for DFUs should be on radical and repeated debridement, frequent inspection and bacterial control and careful moisture balance to prevent maceration.  Tissue debridement  Inflammation and infection control  Moisture balance (optimal dressing selection)  Epithelial edge advancement.  Tissue debridement: There are many methods of debridement used in the management of DFUs including surgical/sharp, larval, autolytic and, more recently, hydrosurgery and ultrasonic. Debridement may be a one-off procedure or it may need to be ongoing for maintenance of the wound bed. The requirement for further debridement should be determined at each dressing change

The benefits of debridement include:

 Removes necrotic/sloughy tissue and callus  Reduces pressure  Allows full inspection of the underlying tissues  Helps drainage of secretions or pus  Helps optimise the effectiveness of topical preparations  Stimulates healing.  It is important to debride the wound margins as well as the wound base to prevent the ‘edge effect’, whereby epithelium fails to migrate across a firm, level granulation base.

Autolytic debridement: This is a natural process that uses a moist wound dressing to soften and remove devitalised tissue. Care must be taken not to use a moisture donating dressing as this can predispose to maceration. In addition, the application of moisture-retentive dressings in the presence of ischaemia and/or dry gangrene is not

 Clinically uninfected wounds should not be treated with systemic antibiotic therapy. However, virtually all infected wounds require antibiotic therapy.  Superficial DFUs with skin infection (mild infection) – For mild infections in patients who have not recently received antibiotic treatment:  Start empiric oral antibiotic therapy targeted at Staphylococcus aureus and ß- haemolytic Streptococcus  Change to an alternate antibiotic if the culture results indicate a more appropriate antibiotic  Obtain another optimum specimen for culture if the wound does not respond to treatment. Role of topical antimicrobials: The increasing prevalence of antimicrobial resistance (eg meticillin-resistant Staph. aureus [MRSA]) or other complications (eg Clostridium difficile infection) has led to a rise in the use of topical antimicrobial treatments for increased wound bioburden. Antimicrobial agents that are used topically have the advantage of not driving resistance. Such agents provide high local concentrations, but do not penetrate intact skin or into deeper soft tissue. Topical antimicrobials may be beneficial in certain situations.

Pressure offloading:

 In patients with peripheral neuropathy, it is important to offload at-risk areas of the foot in order to redistribute pressures evenly.  Inadequate offloading leads to tissue damage and ulceration.  The gold standard is the total contact cast (TCC).  Using a TCC in patients with a unilateral uncomplicated plantar ulcer can reduce healing time by around six weeks.  Disadvantages of TCCs include:  Must be applied by fully trained and experienced practitioners  May cause skin irritation and further ulcers if applied inappropriately  Prevents daily inspection (signs of spreading infection may go unnoticed)  May disturb sleep  Makes bathing difficult  Patient may not tolerate it (especially in warm climates)  May prevent patient's ability to work  Relatively high cost/low availability.  In patients with ischaemic or neuroischaemic ulcers, the priority is to protect the margins of the foot (eg using Scotchcast boots or healing sandals). TCCs are contraindicated in patients with ischaemia because of the risk of inducing further DFUs. Removable devices (such as removable cast walkers, Scotchcast boots, healing sandals and crutches, walkers and wheelchairs) should be selected in these patients. Removable devices may also be more pragmatic choices for less motivated patients because they allow patients to bathe and sleep more comfortably.

Prevention:

 Multidisciplinary team approach  Patient education  Daily foot inspections by patient  Shoegear changes  Elective surgery to decrease risk.

2. a) This patient should be evaluated as follows:

History:

o History of anorexia, nausea, vomiting, fever (whether fever is associated with chill and rigor to exclude UTI). o History of diarrhea. o History of amenorrhea. o History of trauma. Physical examination:

o Pulse – tachycardia. o Temperature. o Anemia. o Tenderness and rebound tenderness.

Investigations:

o Blood for – complete blood count, urea,creatinine. o Urine for R/E and C/S o USG- whole abdomen.  Patients with acute appendicitis typically look ill and are lying still in bed.  Low-grade fever is common (<38°C).  Examination of the abdomen usually reveals diminished bowel sounds and focal tenderness with voluntary guarding.  The exact location of the tenderness is directly over the appendix, which is most commonly at McBurney's point (located one third of the distance along a line drawn from the anterior superior iliac spine to the umbilicus).  Peritoneal irritation can be elicited on physical examination by the findings of voluntary and involuntary guarding, percussion, or rebound tenderness.  Any movement, including coughing (Dunphy's sign), may cause increased pain.  Other findings may include pain in the right lower quadrant during palpation of the left lower quadrant (Rovsing's sign), pain on internal rotation of the hip (obturator sign, suggesting a pelvic appendix), and pain on extension of the right hip (iliopsoas sign, typical of a retrocecal appendix).  Rectal and pelvic examinations are most likely to be negative. However, if the appendix is located within the pelvis, tenderness on abdominal examination may be minimal, whereas anterior tenderness may be elicited during rectal examination as the pelvic peritoneum is manipulated.  Pelvic examination with cervical motion may also produce tenderness in this setting.

If the appendix perforates:

 Abdominal pain becomes intense and more diffuse, and abdominal muscular spasm increases, producing rigidity.  The heart rate rises, with an elevation of temperature above 39°C.  The patient may appear ill and require a brief period of fluid resuscitation and antibiotics before the induction of anesthesia.  Occasionally, pain may improve somewhat after rupture of the appendix, although a true pain-free interval is uncommon.

The Alvarado score is a clinical scoring system used in the diagnosis of appendicitis. The score has 6 clinical items and 2 laboratory measurements with a total 10 points. The modified Alvarado score is at present in use.

Appendicectomy is the treatment of choice.

2. b) Lump in epigastric region: Intra-abdominal swellings:- They occur in connection with the:  Liver and Subphrenic abscess-  Stomach and Duodenum- o Congenital pyloric stenosis- Babies about 2 to 4 weeks old when present with projectile vomiting after meals, the diagnosis becomes obvious. On examination visible peristalsis of the stomach is always seen. Sometime a definite lump may be felt at the pylorus of the stomach. o Subacute perforation of peptic ulcer. o Carcinoma of the stomach-  - Intussusception, diverticulitis, Hyperplastic tuberculosis and neoplasms are the causes of swellings which may originate from the transverse colon.  Omentum- In the tubercular peritonitis, the omentum is rolled up to form a transverse ridge in the epigastrium. Enlarged lymph nodes or adherent coils of intestine are also come across in this condition.  Pancreas- It hardly gives rise to a palpable swelling. The condition that forms lump in connection with this organ is the pseudopancreatic cyst. True cyst of the pancreas is extremely rare. The pseudocyst is a collection of fluid in the lesser sac of the peritoneal cavity resulting from acute pancreatitis or trauma. It forms a smooth rounded swelling with fluctuation test positive. X-ray with barium meal will show the exact position of the swelling which is situated behind the stomach and is best seen in the lateral X-ray.  Abdominal aorta- Aneurysm of this part of the aorta is not uncommon. It presents a swelling in the epigastrium with characteristic expansile pulsation. This pulsation should be differentiated from transmitted pulsation caused by a swelling, just in front of the aorta. Knee-elbow position is of great help in this respect. The swelling in front of the aorta will hang loose loose forward leaving contact with the aorta, hence losing its pulsatile property. Whereas an aneurysm of the aorta will be still pulsatile.  Lymph nodes- In addition to the usual causes of enlargement of lymph nodes, the followings are more important in this region: (i) tabes mesenterica, (ii) lymphosarcoma and (iii) secondary malignant growth from the neighbouring organs and also from the testis.  Retroperitoneal sarcoma and teratoma are the two conditions commonly seen in the posterior abdominal wall arising from the retroperitoneal tissue.

Management of Hydatid cyst of liver:

Investigations  The definitive diagnosis of liver echinococcosis requires a combination of imaging, serologic, and immunologic studies.  Routine laboratory tests are rarely abnormal occasionally eosinophilia may be present in the presence of cyst leakage, or may be normal. Serum alkaline phosphatase levels are raised in one third of patients. Serology and immunological tests:  Serological tests detect specific antibodies to the parasite and are the most commonly employed tools to diagnose past and recent infection with E. granulosus. Detection of IgG antibodies implies exposure to the parasite, while in active infection high titers of specific IgM and IgA antibodies are observed.  Detection of circulating hydatid antigen in the serum is of use in monitoring after surgery and pharmacotherapy and in prognosis.  Tests of humoral immunity are still widely used to confirm the diagnosis.  Indirect immunofluorescence assay (IFA) is the most sensitive test (95%) in patients with hepatic CHD. Imaging techniques:  Imaging modalities range from simple to complex and invasive. Ultrasonography (US) is the screening method of choice.  CT scan is an important preoperative diagnostic tool to determine vascular, biliary or extra hepatic extension, to recognize complications, such as rupture and infections, and therefore to assess respectability.  Magnetic resonance Imaging (MRI scan) - MRI delineates the cyst capsule better than CT scan, as a low intensity on both T1 and T2 weighted images. However, CT scan is better in demonstrating mural calcifications, cysts less than 3 cm may not show any specific features and small peritoneal cysts may be missed.  Endoscopic retrograde cholangiopancreatography (ERCP) remains an important tool in cases where a rupture into the biliary tree has occurred, allowing both the diagnosis of major biliary communication and clearance of the common bile duct (CBD) prior to surgery or intervention by the means of sphincterotomy.  Direct cholangiography: intra-operative cholangiography is performed through a cystic drain or a T-Tube in a suspected intrabiliary rupture and bile duct obstruction. This method is used to detect post-operative complications following surgery.  However, diagnostic tests such as CT and MRI are mandatory in liver hydatidosis because they allow thorough knowledge regarding lesion size, location, and relations to intrahepatic vascular and biliary structures, providing useful information for effective treatment and decrease in post-operative morbidity. Plain Radiographs: Plain radiographs of the abdomen and chest may reveal a thin rim of calcification delineating a cyst, or an elevated hemi diaphragm.

Doppler ultrasonography is indicated to show the reports of hydatid cyst with vascular axes (portal vein, hepatic veins, and inferior vena cava). However, in the types I and IV, we have to consider differential diagnosis.

Gharbi Classification on Ultrasonography features of Hydatid Cyst.

Type V cysts determined by ultrasound to be calcified and have been assumed to be dead cysts and do not require surgery. Intra-operative Ultrasonography is an important investigation during surgery for hydatid cyst of the liver.

 WHO introduced a standardized classification of Ultrasonography images of cystic echinococcosis, to obtain comparable results in patients worldwide and to link disease status with each morphological type of Hydatid cysts.

 CL o unilocular anechoic cystic lesion without any internal echoes and septations  CE 1 o uniformly anechoic cyst with fine echoes settled in it representing hydatid sand  CE 2 o cyst with multiple septations giving it multivesicular appearance or rosette appearance or honeycomb appearance with unilocular mother cyst o this stage is the active stage of the cyst  CE 3 o unilocular cyst with daughter cysts with detached laminated membranes appearing as water lily sign o this is the transitional stage of the cyst  CE 4 o mixed hypo- and hyperechoic contents with absent daughter cysts; these contents give an appearance of a ball of wool (ball of wool sign) indicating the degenerative nature of the cyst  CE 5 o arch-like thick partially or completely calcified wall o this stage of cyst is inactive and infertile

Treatment: General considerations:  Surgery remains the gold standard treatment for hydatid liver disease. The aim of surgical intervention is to inactivate the parasite, to evacuate the cyst along with resection of the germinal layer, to prevent peritoneal spillage of scolices and to obliterate the residual cavity. It can be performed successfully in up to 90% of patients if a cyst does not have a risky localisation.  The choice of an optimal treatment should be carefully assessed in each case.  The principles of hydatid surgery are o Total removal of all infective components of the cysts; o The avoidance of spillage of cyst contents at time of surgery; o Management of communication between cyst and adjacent structures; o Management of the residual cavity; o Minimize risks of operation. All the surgical procedures can be divided into two large groups, a conservative group and a radical one. The conservative technique communication between cyst and adjacent structures; The conservative technique: Conservative procedures are safe and technically simple, and are useful in the management of uncomplicated hydatid cysts. Marsupialization was the most common used procedure because it is quick and safe. However, their main disadvantage is the high frequency of postoperative complications, the most common being bile leak from a cyst-biliary communication, bilomas and bile peritonitis. Radical surgical procedures: Radical surgical procedures include cystectomy, pericystectomy, lobectomy and hepatectomy. Radical procedures have lower rate of complications and recurrences but many authors consider them inappropriate, claiming that intraoperative risks are too high for a benign disease. Laparoscopic management of hydatid cysts The rapid development of laparoscopic techniques has encouraged surgeons to replicate principles of conventional hydatid surgery using a minimally invasive approach Laparoscopic procedure has some advantages compared to open surgery. In fact this approach to liver hydatid cyst offers a lower morbidity outcome and a shorter hospital stay and it is also associated with a faster surgery. In addition to that advantage, Laparoscopic procedure gives a better visual control of the cyst cavity under magnification which allows a better detection of biliary fistula. This approach is possible only in selected cases. The Criteria to exclude laparoscopic treatment of hydatid cyst of liver are o Rupture of the cyst in biliary tract o Central localization of the cyst o Cysts dimension > 15 cm o Number of cysts > 3 o Thickened or calcified walls o Opening of bile ducts that leak bile Percutaneous treatment of hydatid cyst PAIR (puncture, aspiration, injection, and reaspiration) is a percutaneous treatment technique for hydatid disease. The World Health Organization currently supports PAIR as an effective alternative to surgery, although its use is limited. The World Health Organization guidelines for indications and contraindications of PAIR are as follows: Indications for PAIR o Nonechoic lesion greater than or equal to 5 cm in diameter o Cysts with daughter cysts and/or with membrane detachment o Multiple cysts if accessible to puncture o Infected cysts o Patients who refuse surgery. o Patients who relapse after surgery. o Patients in whom surgery is contraindicated Patients who fail to respond to chemotherapy alone o Children over 3 years. o Pregnant women Contraindications for PAIR o Non cooperative patients o Inaccessible or risky location of the liver cyst o Cyst in spine, brain, and/or heart o Inactive or calcified lesion o Cyst communicating with the biliary tree Patients should be followed clinically after PAIR treatment. Recurrence is increased in more complicated cysts, including those with multiple daughter cysts. Endoscopic management of hydatid cyst: The ERCP is effective in diagnosing biliary tree involvement from the cyst. The Endoscopic management is useful in presence of intrabiliary rupture, which requires exploration and drainage of the biliary tract and also after surgery in presence of residual hydatid material (membranes and daughter cyst) left in biliary tree. During the endoscopic exploration the biliary tree is cleared of any hydatid material with a balloon catheter or a dormia basket. Chemotherapy for hydatid disease of liver: Medical treatment of hydatid liver cysts, is based on benzoimidazole carbamates, such as mebendazole and albendazole. Albendazole seems to be more effective owing to better penetration and absorption.

3. a) Diagnostic peritoneal lavage (DPL) or diagnostic peritoneal aspiration (DPA) is a surgical diagnostic procedure to determine if there is free floating fluid (most often blood) in the abdominal cavity.

Indications and Contraindications for Diagnostic Peritoneal Lavage Indications Equivocal physical examination Unexplained shock or hypotension Altered sensorium (closed head injury, drugs, etc.) General anesthesia for extra-abdominal procedures Cord injury Contraindications Clear indication for exploratory laparotomy Relative contraindications: Previous exploratory laparotomy

Pregnancy Obesity

Procedure:

After the application of local anesthesia, a vertical skin incision is made one third of the distance from the umbilicus to the pubic symphysis. The linea alba is divided and the peritoneum entered after it has been picked up to prevent bowel perforation. A catheter is inserted towards the pelvis and aspiration of material attempted using a syringe. If no blood is aspirated, 1 litre of warm 0.9% saline is infused and after a few (usually 5) minutes this is drained and sent for analysis.

Interpretation of results:

If any of the following are found then the DPL is positive of trauma and operative exploration is warranted.

 10 cc/blood  100,000 RBCs/mm3  500 WBCs/mm3  Presence of bile, bacteria or food particle  Amylase level > 175 IU/ml  Bilirubin level > 0.01 mg/DL  ALP > 2 IU/L

3. b) Answer. Colostomy refers to a surgical procedure where a portion of the is brought through the abdominal wall to carry stool out of the body.

It may be permanent or temporary.

Permanent colostomy: done in APR( Abdomino perineal resection)

Temporary colostomy : done in multiple fistula in ano.

Indications:

. Cancer of colon and rectum . Diverticular disease, . Crohn’s disease . Ulcerative colitis . Trauma. Types of colostomy:

 Loop colostomy: This type of colostomy is usually used in emergencies and is a temporary and large stoma. A loop of the bowel is pulled out onto the abdomen and held in place with an external device. The bowel is then sutured to the abdomen and two openings are created in the one stoma: one for stool and the other for mucus.  End colostomy: A stoma is created from one end of the bowel. The other portion of the bowel is either removed or sewn shut (Hartmann's procedure).  Double barrel colostomy: The bowel is severed and both ends are brought out onto the abdomen. Only the proximal stoma is functioning.

Complications: . Necrosis of stoma . Retraction of stoma . Prolapsed of stroma . Stenosis or narrowing . Parastomal hernia

3. c) Newer approaches look at ER, PR, HER2 together, along with the tumor grade, to categorize breast cancer into several conceptual molecular classes that have different prognoses and may have different responses to specific therapies.  DNA microarrays have assisted this approach.

Molecular subtypes are as follow:

 Luminal A breast cancer is hormone-receptor positive (estrogen-receptor and/or progesterone-receptor positive), HER2 negative, and has low levels of the protein Ki-67, which helps control how fast cancer cells grow. Luminal A cancers are low-grade, tend to grow slowly and have the best prognosis.  Luminal B breast cancer is hormone-receptor positive (estrogen-receptor and/or progesterone-receptor positive), and either HER2 positive or HER2 negative with high levels of Ki-67. Luminal B cancers generally grow slightly faster than luminal A cancers and their prognosis is slightly worse.  Triple-negative/basal-like breast cancer is hormone-receptor negative (estrogen- receptor and progesterone-receptor negative) and HER2 negative. This type of cancer is more common in women with BRCA1 gene mutations. Researchers aren’t sure why, but this type of cancer also is more common among younger and African-American women.  HER2-enriched breast cancer is hormone-receptor negative (estrogen-receptor and progesterone-receptor negative) and HER2 positive. HER2-enriched cancers tend to grow faster than luminal cancers and can have a worse prognosis, but they are often successfully treated with targeted therapies aimed at the HER2 protein, such as Herceptin (chemical name: trastuzumab), Perjeta (chemical name: pertuzumab), Tykerb (chemical name: lapatinib), and Kadcyla (chemical name: T-DM1 or ado-trastuzumab emtansine).  Normal-like breast cancer is similar to luminal A disease: hormone-receptor positive (estrogen-receptor and/or progesterone-receptor positive), HER2 negative, and has low levels of the protein Ki-67, which helps control how fast cancer cells grow. Still, while normal-like breast cancer has a good prognosis, its prognosis is slightly worse than luminal A cancer’s prognosis.

• 3. d) Intussusceptions. Introduction:

 Incidence approximately 2 per 1000 live births.  Peak age of presentation at 3-10 months.  M:F = 2:1.  Fewer than 10% have a clear focal pathological cause that starts the intussusception (Apex: older children are more likely to have an apex).

Types: o Ileoileal o Ileocolic – most common. o Ileoileocolic. o Colocolic o Multiple o Retrograde o Others

Clinicopathological features:

 Invagination/telescoping of the proximal bowel (called the intussusceptum, e.g. terminal /ileo-caecal valve) into the distal bowel (called the intussuscepiens, e.g. caecum/).  May be due to enlargement of lymphatic patches of Peyer (idiopathic)  Pathology at the apex may be: o Meckel's diverticulum; o polyp; o lymphoma.

Clinical features: Classic triad of features is: o Abdominal pain (associated with pallor, screaming, and restlessness); o Palpable sausage-shaped mass (mid-abdominal or right upper quadrant); o Passage of red-currant jelly stool. (Rectal examination may reveal bloody mucus and the lead point may rarely be palpable.)  Typically the infant is relatively settled between bouts of pain.  Signs of shock (lethargy, poor feeding, hypotonia) require urgent fluid resuscitation.  Features of obstruction (distension and vomiting) may occur.

Diagnosis and investigations:

 Ultrasound (diagnostic text of choice): intussusception in cross-section (doughnut or target sign).  Plain X-ray: may show soft tissue mass, small bowel obstruction, free air indicating perforation.  Air (or rarely gastrograffin) contrast enema is diagnostic and may be therapeutic.

Treatment:

 Immediate IV fluid resuscitation to correct fluid losses and to restore fluid, electrolyte, and acid/base balance.  Maintenance fluid replacement and replacement of continued losses (vomiting or nasogastric losses). Reduction only attempted once fluid balance restored.  Analgesia and sedation (morphine 0.2mg/kg) will aid process of reduction.

Methods of reduction:

Radiological reduction

 Air enema therapeutic in 75% of cases.  Usually performed in radiology department under screening control.  Surgeon should be present.  Evidence of irreducible obstruction or perforation mandates immediate halt.  Partial or incomplete reduction may warrant repeat attempt after 4-6 hours.  Informed consent includes risk of peforation.

Surgical reduction:

 Laparotomy indicated without enema if evidence of peritonitis or perforation.  Manual reduction by retrograde squeezing and gentle proximal traction.  Resection and anastomosis if bowel viability is in doubt (~10% require resection).  Post-reduction septic shock may occur with release of bacterial products from viable but damaged bowel segment.  Most recover rapidly with resumption of oral feeding in 24-48h and discharge home in 4-5 days.

Complications:

 Recurrence rate is 5-7% in non-operative cases, and about 3% for operative reduction.  Morbidity is low but delayed diagnosis, inadequate resuscitation, and failure to recognize ischaemic or perforated bowel account for 1% mortality.

3. e) Amoebic liver abscess.

Introduction: Amebic liver abscess is caused by the parasitic protozoan Entamoeba histolytica.

Aetiology:

Two species of ameba infect humans. E. dispar is associated with an asymptomatic carrier state and not with disease. E. histolytica is responsible for all forms of invasive disease. The life cycle involves cysts, invasive trophozoites, and fecally contaminated food or water to initiate the infection. Fecal-oral transmission occurs; the cyst passes through the stomach into the intestine unscathed, andthen pancreatic enzymes start to digest the outer cyst wall. The trophozoite is then released into the intestine and multiplies there. Normally, no invasion occurs, and the patient develops amebic dysentery alone or becomes an asymptomatic carrier. In a small number of cases, the trophozoite invades through the intestinal mucosa, travels through the mesenteric lymphatics and veins, and begins to accumulate in the hepatic parenchyma, forming an abscess cavity. Liquefied hepatic parenchyma with blood and debris gives a characteristic "anchovy paste" appearance to the abscess.

SYMPTOM Pain Fever Nausea and vomiting Anorexia Weight loss Malaise Diarrhea Cough or pleurisy Pruritus SIGN Hepatomegaly Right upper quadrant tenderness Pleural effusion or rub Right upper quadrant mass Ascites Jaundice

LABORATORY DATA Increased alkaline phosphatase WBC count >10,000/mm3

Hematocrit <36% Albumin <3 g/dL Bilirubin >2 mg/dL

Diagnosis • Blood culture • Haemoglobin Estimation • Stools Examination (Tropazoites and Cysts) • Radiography • Aspiration • Medical untrasonography and CT Scannig • Sigmoidoscopy • Liver fubction tests • Serological Tests

Distinguishing Clinical Characteristics of Patients with Hepatic Abscesses:

Amebic Pyogenic Age <50 years Age >50 years Male:female ratio 10:1 Male:female ratio 1:1 Hispanic descent No ethnic predisposition Recent travel to endemic area Malignancy Pulmonary dysfunction High fevers Amebic Pyogenic Abdominal pain Pruritus Diarrhea Jaundice Abdominal tenderness Septic shock Hepatomegaly Palpable mass

Management:

• Metronidazole 800mg TDS for 5-10 days • Aspiration • Repeated Imaging Of Liver Complications:

 Complications from amebic abscesses occur secondary to rupture of the abscess into the peritoneum, pleural cavity, or pericardium.  Extrahepatic sites also have been described in the lung, brain, skin, and genitourinary tract, presumably from hematogenous spread.  Ruptured amebic liver abscesses are associated with mortality rates upto 50%.

4. a) Myositis ossificans.

Myositis ossificans (MO) is a benign process characterised by heterotopic ossificationusually within large muscles. Its importance stems in large part from its ability to mimic more aggressive pathological processes.

Epidemiology:  Most cases of myositis ossificans occur as a result of trauma, and thus the main demographic is young adults.  Another group which are particularly prone to myositis ossificans are paraplegics, usually without evidence of trauma.

Clinical presentation  Typically presents as a painful, tender, enlarging mass, which in 80% of cases is located in large muscles of the extremities, often following recognised local trauma, although a definite traumatic event is not always recalled.  In the case of paraplegics, recognised episodes of trauma are often absent, and the disease occurs particularly around the knees and hips.

Pathology Myositis ossificans is essentially metaplasia of the intramuscular connective tissue resulting in extraosseous bone formation (without inflammation).

It has a zonal organization:

 Peripheral well organized mature lamellar bone  Intermediate osteoid region  Central immature non-ossified cellular (fibroblasts) focus Unfortunately the histologically of myositis ossificans can appear similar toosteosarcoma, and thus can actually lead to inappropriate management.

Radiographic features The typical radiographic appearance of myositis ossificans is circumferential calcification with a lucent centre, and a radiolucent cleft (string sign) that separates the lesion from the cortex of the adjacent bone.

Radiograph Calcification usually begins to become apparent on plain radiographs within 2-6 weeks, and the lesion reaches the classic well circumscribed peripherally calcified appearance by 2 months. Over the following 4 or so months, they typically become smaller and denser.

Cleft between it and the subjacent bone may be difficult to see on plain radiographs.

CT CT appearances are similar to those of plain radiography, demonstrating mineralisation proceeding from the outer margins towards the center. The cleft between it and the subjacent bone is usually visible.

The peripheral rim of mineralisation is usually visible within 4-6 weeks.

MRI MRI appearances change with the age of the lesion.

Early features can be misleading because the peripheral calcification is not well seen, and oedema in the soft tissues may extend beyond the often inapparent calcific rim.

Nuclear medicine Non-specific increased uptake on flow and blood pool images is seen early on in the development of the lesion, and gradually decreases as the lesion matures.Increased uptake on delayed phase is typical.

Treatment and prognosis Myositis ossificans is benign and there is no compelling evidence that malignant degeneration ever occurs. As such treatment is reserved for symptomatic lesions, and surgical resection is usually curative.

Differential diagnosis Imaging differential considerations include:

 Parosteal osteosarcoma: calcifies in centre and continues towards the periphery  Soft tissue sarcomas including o Malignant fibrous histiocytoma o Synovial sarcoma

4. b) Fracture of patella. Classification:  Displaced  Non-displaced o 1 to 2 mm articular step-off o 3 mm fragment separation  Transverse  Longitudinal  Comminuted

Mechanism of injury:

 Direct blow to patella  Indirect force from violent quadriceps contraction  Expected healing time: 8-12 weeks  Rehabilitation duration: 12-15 weeks

Clinical features:

 Pain  Swelling and redness over knee  Restriction of movement

Treatment goals:

 Alignment o Open repair of the retinaculum if extensor lag is present  Stability o Best achieved by restoring bony congruity with hardware to rigidly fix fracture

Treatment:

 Operative vs Nonoperative:  If able to straight leg raise and less than 2mm of step off, non-operative.  Disrupted mechanism – operative  If question, have patient lay on side to fire extensor  Patella Fractures: Surgical Options:  Fixation  Partial Patellectomy  Total Patellectomy  Ilizarov  Fixation of Patella Fractures:  Tension band, modified tension band  Cannulated screws  Suture repair  Cannulated Screw Tension Band with wire  Cannulated Screw Tension Band with cable  Plates  Tips for Modified Tension Band  Avoid prominent hardware with modified tension band  Bend both end of the wires to avoid migration  Helpful to use a suture for cerclage rather than wire  Tools for Using a Cannulated Screw Tension Band with Cable:  mm cable with preloaded crimp  4.0 mm partially threaded cannulated screws

4. c) Complications of Colle’s fracture:

4. d) Pathological fracture.

Definition:

A pathological facture is one in which a bone is broken, through an area, weakened by pre- existing disease, by a degree of stress, that would have left the normal bone intact. In other words, a fracture involving “abnormal bone” is a pathological fracture. Etiology:

 Development disorders of bone : o Congenital defects of bone tissue : Osteogenesis imperfecta Osteopetrosis o Disorder of cartilage growth : Achondroplasia Diaphysealaclasis (multiple exostosis) Dyschondroplasia (Ollier’s disease) o Nutritional and vitamin deficiencies: Scurvy, Rickets, Osteomalacia  Hormonal imbalance : o Hyperparathyroidism ,Cushing’s syndrome. o Pathological fracture from cortisone treatment Frohlich’s syndrome (hypopituitarism)  Atrophic conditions of bone : Disuse osteoporosis Senile osteoporosis  Pathological fracture through infected bone : Osteomyelitis  Cystic disorders and fibrous dysplasia of bone : o Unicameral bone cyst o Aneurysmal bone cyst o Non – osteogenic fibroma of bone o Monostotic and polyostotic fibrous dysplasia o Paget’s disease of bone  Primary and secondary tumors of bone: o Primary benign tumours: Chondroma, Benign chondroblastoma, Chondromyxoid fibroma, Haemangioma of bone ,Giant cell tumour of bone ,Disappearing bone disease. o Malignant tumours : Osteosarcoma,Chondrosarcoma ,Fibrosarcoma,Malignant fibrous histiocytoma, Malignant round cell tumour, Multiple myelomatosis o Metastatic tumours of bone – lungs, thyroid, kidney, GI tract, prostrate  Marrow cell disorder: Histiocytos Gaucher’s disease  Parasitic disease of the bone : Hydatid disease  Neurotrophic dystrophies of the bone : Tabes dorsalis ,Syringomyelia, Diabetic neuropathy  Iatrogenic pathological fracture : o Through screw hole stress protection phenomenon o Through biopsy o After removal of infected bone o Through a donor site for a bone graft

Pathological fracture is suspected when fracture occurs:

 Spontaneously  After minor trauma  Unusual fracture pattern  History of recent several fractures  Older patient  History of primary malignancy

Investigations:

 Radiography:

Plain –X-ray:

 Study the Fracture  Lesion location: Usually eccentric, Cortical involvement, Diaphyseo-metaphyseal junction  Don’t ignore the perifracture  Densities within the lesion: Bone formation changes suggests – Osteosarcoma;Calcification suggests – Chondrosarcoma  Other lesions  Reaction (periosteal / endosteal) should be examined.  Alteration in density and  Zone of transition architecture  Extra osseous masses or  Moth eaten or permeative pattern of bone reaction abnormalities

Laboratory studies:

General investigations Search for occult primary carcinoma

 Complete haemogram o Breast - Examination  Peripheral smear Mammography  Serum glucose o Lung – Chest X-ray  Serum albumin o Kidney – Ultrasonography  Serum calcium, Phosphate o Thyroid – Digital palpation o  Alkaline phosphatase Prostate – Serum PSA, Digital  LFT prostate examination o  Urine sugar and albumin Myeloma – Bone marrow  Bence-Jones proteins examination; Bence-Jones proteins; Serum and urine  Serum electrophoresis electrophoresis  Tumor markers: Ca 125, Ca 19.9, CEA o Skeletal Survey - X-ray skull,

spine and pelvis. o Other organs

MRI:

 Marrow disease  Epidural and nerve root compression can be detected  Localize the disease

Bone scan

PET scan

Treatment:

o Management of pain o Avoiding the fracture o Bone stabilization o Conservative measures

Systemic Therapy:

 Hormone therapy : Ca breast and prostate  Chemotherapy  Bisphosphonates : Zolindronic acid  Targeted therapy : Denosumab

Radiotherapy:

 External-beam radiotherapy  Stereotactic Body Radiotherapy(SBRT)  Radiopharmaceuticals : Unsealed source therapy with bone-seeking radionueclides

Surgical management:

 Indication:  Palliative  Fracture: Ambulation / Pain relief  Impending fracture.

4. e) Giant cell tumour:

Introduction:

Giant cell tumours (GCTs) of bone, also known as osteoclastomas, are relatively common bone tumours, usually benign which are arising from metaphysis and typically extending into the epiphysis of the long bones.

Epidemiology

 GCTs are common, comprising 18-23% of benign bone neoplasms and 4-9.5% of all primary bone neoplasms.

 They almost invariably (97-99%) occur when the growth plate has closed and are therefore typically seen in early adulthood

 80% of cases reported between the ages of 20 and 50, with a peak incidence between 20 and 30.

 There is overall a mild female predilection, especially when located in the spine, however malignant transformation is far more common in men (M:F of ~3:1).

Clinical presentation Presentation is not specific, typically presents insidiously and relates to bone pain, soft tissue mass, compression of adjacent structures or acutely with a pathological fracture.

Pathology o Giant cell tumours are believed to result from an over-expression in RANK/RANKL signalling pathway with resultant over-proliferation of osteoclasts. o These tumours contain numerous thin walled vascular channels predisposing to areas of haemorrhage and presumably related to the relatively frequent co-existence of aneurysmal bone cysts (ABC) found in 14% of cases. o Macroscopically, giant cell tumours are variable in appearance, depending on amount of haemorrhage, presence of co-existent ABC, and degree of fibrosis. o Microscopically they are characterised by prominent and diffuse osteoclastic giant cells and mononuclear cells ( round, oval, or polygonal and may resemble normal histiocytes).GCTs are low grade tumours even in radiologically aggressive appearing lesions.

o Approximately 5-10% of GCTs are malignant. Sarcomatous transformation is seen, especially in radiotherapy treated inoperable tumours.

o Although rare (3%) lung metastasis are possible.

Location They typically occur as single lesions. Although any bone can be affected, the most common sites are:

o Around the knee: distal femur and proximal tibia: 50-65% o Distal radius: 10-12% o Sacrum: 4-9% o Vertebral body Multiple locations: ≈ 1% (multiple lesions usually occur in association with Paget disease)

Radiographic features Classic appearance Four radiographic features are characteristic when a GCT is located in a long bone:

o Occurs only with a closed growth plate o Abuts articular surface: 84-99% come within 1 cm of the articular surface o Well defined with non-sclerotic margin (though < 5% may show some sclerosis) o Eccentric: if large this may be difficult to assess Plain film and CT General radiographic features include:

 Narrow zone of transition: a broader zone of transition is seen in more aggressive gcts  No surrounding sclerosis: 80-85%  Overlying cortex is thinned, expanded or deficient  Periosteal reaction is only seen in 10-30% of cases  Soft tissue mass is not infrequent  Pathological fracture may be present  No matrix calcification/mineralization MRI: can differentiate from aneurismal bone cyst Scintigraphy ( bone scan): Most GCT demonstrate increased uptake on delayed images, especially around the periphery, with a central photopenic region (doughnut sign). Increased blood pool activity is also seen, and can be seen in adjacent bones due to generalised regional hyperaemia (contiguous bone activity). Angiography: If performed, usually in the setting of pre-operative embolisation, angiography usually demonstrates a hypervascular tumour (2/3rd of cases) with the rest being hypo or avascular. Treatment and prognosis  Classically, treatment is with curettage and packing with bone chips or polymethylmethacrylate (PMMA

 Local recurrence is from the periphery of the lesion and has historically occurred in up to 40-60% of cases.

 Newer intraoperative adjuncts such as thermal or chemical treatment of the resection margins have lowered the recurrence rate to 2.5-10%.

 Early work on monoclonal antibodies, as an adjuvent treatment, has been impressive in causing high rates of tumour necrosis.

 Wide local excision is associated with a lower recurrence rate, but has greater morbidity.

Differential diagnosis: There is a relatively wide differential similar to that of a lytic bony lesion:  Chondroblastoma  Multiloculated aneurysmal bone cyst (abc): younger age group, but may co-exist with GCT; fluid-fluid levels  Non-ossifying fibroma: usually younger age group.  Brown tumour: in the setting of hyperparathyroidism  Enchondroma: only really a consideration in lesions of small bones of the hand and foot  Chondrosarcoma: typically older age group  Metastases and multiple myeloma

The West Bengal University of Health Sciences

M.B.B.S. 3rd Professional Part – II Examination, 2017

Subject: Surgery Time: 21/2 hrs.

Paper: I Marks: 60

Group – A 1. Describe the signs, symptoms, prevention and treatment of Tetanus. 15

Group – B 2. a) Discuss the investigations of a 50 years old male presented with obstructive jaundice and palpable gall bladder. Give outline of management of the patient. How will you prepare liver for operation if needed? 5+5+5

or b) Discuss the pathophysiology of acute intestinal obstructions. How will you manage a case of intussusceptions? 5+10

Group – C

3. Write short notes on (any three) of the following: 3x5 a) Sentinel lymph node biopsy. b) Alvarado Score. c) Parotid abscess. d) Mesenteric cyst. e) Gastrinoma (ZESyndrome) Group –D

4. Answer in brief on any three of the following. 3x5 a) Volkman’s ischaemic contracture. b) Pathological fracture. c) Congenital talipes equinovarus. d) Radial nerve injury due to fracture. e) Perthes disease.

1. Signs and symptoms:Tetanus symptoms usually emerge about 7-10 days after initial infection; however, this can vary from 4 days to about 3 weeks, and, in some cases, it may take months.

In general, the further the injury site is from the central nervous system, the longer the incubation period. Patients with shorter incubation times tend to have more severe symptoms.

Muscle symptoms - spasms and muscular rigidity (muscles become stiff). Stiffness usually starts with the chewing muscles, hence the name lockjaw. Muscle spasms then spread to the neck and throat, causing dysphagia (difficulty swallowing). Patients often go on to have spasms in their facial muscles.

Breathing difficulties may result from neck and chest muscle stiffness. With some patients, abdominal and limb muscles are also affected.

In severe cases, the spine will arch backward as the back muscles are affected - this is more common when children are infected.

Most patients with tetanus will also have the following symptoms:

 Bloody stools (feces)  Diarrhea  Fever  Headache  Sensitivity to touch  Sore throat  Sweating  Tachycardia (rapid heartbeat)

Complications: If the patient does not receive treatment, the risk of life-threatening complications is higher - mortality rates vary from 40-76 percent - symptoms may include:

 Fractures - sometimes, in severe cases, the muscle spasms and convulsions may lead to bone fractures.  Aspiration pneumonia - if secretions or contents of the stomach are inhaled, a lower respiratory tract infection can develop, leading to pneumonia.  Laryngospasm - the larynx (voice box) goes into a spasm which can last up to a minute and cause breathing difficulties. In severe cases, the patient can suffocate.  Tetanic seizures - if infection spreads to the brain, the patient can have epileptic-like fits (seizures).  Pulmonary embolism - a blood vessel in the lung can become blocked and affect breathing and circulation. The patient will urgently need oxygen therapy and anti- clotting medication.  Severe kidney failure (acute renal failure) - severe muscle spasms can result in the destruction of skeletal muscle which can cause myoglobin - a muscle protein - to leak into the urine. This can cause acute renal failure (severe kidney failure).

Treatments:

Wounds should be thoroughly cleaned to prevent infection.

Any cut or wound must be thoroughly cleaned to prevent infection. A tetanus-prone wound should be treated by a medical professional immediately.

What is a tetanus-prone wound? According to the National Health Service (NHS), United Kingdom, it is defined as:

 A wound or burn that requires surgical intervention that is delayed for over 6 hours  A wound or burn that has a considerable amount of removed (devitalized) tissue  Any puncture-type injury that has been in contact with manure or soil  Serious fractures where the bone is exposed to infection (compound fractures)  Wounds or burns in patients with systemic sepsis

Any patient with a wound listed above should receive TIG (tetanus immunoglobulin) as soon as possible, even if they have been vaccinated. Tetanus immunoglobulin contains antibodies that kill Clostridium tetani. It is injected into a vein and provides immediate short-term protection against tetanus.

TIG is just short-term and does not replace the long-term effects of vaccination. Experts say that TIG injections can be safely administered to pregnant and breastfeeding mothers.

Antibiotics - doctors may prescribe penicillin or metronidazole for tetanus treatment. These antibiotics prevent the bacterium from multiplying and producing the neurotoxin that causes muscle spasms and stiffness. Patients who are allergic to penicillin or metronidazole may be given tetracycline instead.

In treating muscle spasms and stiffness, patients may be prescribed:

 Anticonvulsants - such as diazepam (Valium) - relaxes the muscles to prevent spasms, reduces anxiety, and works as a sedative.  Muscle relaxants - such as baclofen - suppresses nerve signals from the brain to the spinal cord, resulting in less muscle tension.  Neuromuscular blocking agents - these medications block the signals from nerves to muscle fibers and are useful in controlling muscle spasms. They include pancuronium and vecuronium.

Prevention

 Exclusion of people with tetanus from childcare, preschool, school and work is not necessary.  Protection against tetanus is provided by a tetanus-containing vaccine which is given in combination with other vaccines as part of routine childhood immunisation. The first dose of tetanus vaccine, in combination with other vaccines, is now recommended to be given at 6 weeks of age. A tetanus booster is recommended at 3½ years of age and between 12 and 15 years of age. A booster is also recommended at age 50 years and prior to travelling overseas, if one has not been given in the previous 10 years.  For adolescents and adults, the combined diphtheria-tetanus-pertussis (dTpa) vaccine is preferred, if not given previously, as it will also provide protection against whooping cough (pertussis).  Boosters every 10 years are no longer recommended.

2. a) D/D: Carcinoma of head and periampullary region of the pancreas.

Investigations:

Blood Tests:  Patients with pancreatic head lesions frequently have elevated bilirubin and alkaline phosphatase levels suggestive of obstructive jaundice.  Other routine laboratory studies are usually normal.  Haemoglobin may be low.  The two most widely used pancreatic cancer serum markers are the CEA and the Lewis blood group carbohydrate antigen CA 19-9. Both are frequently elevated in patients with advanced disease. Extremely high levels of either CA 19-9 or CEA usually indicate unresectable or metastatic disease.

Imaging Studies:

 The main imaging modalities used for patients with suspected pancreatic neoplasms include right upper quadrant ultrasonography, CT, MRI with or without magnetic resonance cholangiopancreatography (MRCP), endoscopic retrograde cholangiopancreatography (ERCP), and percutaneous transhepatic cholangiography (PTC).  Right upper quadrant ultrasonography is a commonly used initial test. It can detect gallstones, dilatation of the biliary tree, and pericholecystic fluid. This imaging modality can also pick up hepatic metastases, pancreatic masses, peripancreatic and hilar lymphadenopathy, and ascites.  The investigation of choice in the work-up of patients suspected of a pancreatic cancer is a multidetector spiral CT and is probably the single most useful diagnostic and staging modality.

 CT provides more complete and accurate imaging of the pancreatic head and surrounding structures.  It gives very important information about the immediately adjacent vascular structures such as the portal, superior mesenteric, and splenic veins, as well as the superior mesenteric artery and celiac axis.  The involvement of periampullary lymph nodes and retroperitoneal structures may be demonstrated.  Additionally, information about distant metastatic disease can be gleaned if metastatic deposits are seen in the liver or in the peritoneal cavity.  The presence of ascites is usually an ominous sign.

 MRCP is now being utilized to image the biliary tree and the pancreatic duct. It has the advantage of being completely noninvasive.

 The vascular structures can also be visualized with the use of the contrast agent gadolinium and the performance of a magnetic resonance angiogram (MRA).  Thus a single (long) session in a scanner can provide information about tumor size and extent (MRI), the intraductal anatomy of the biliary and pancreatic system (MRCP), and the status of the nearby vasculature (MRA).  The resulting scan has the potential to provide information about tumor size and extent, biliary and pancreatic ductal anatomy, and vascular involvement through a single, noninvasive procedure.

 ERCP sometimes is required to solidify the diagnosis of pancreatic cancer.  The classic findings of a long, irregular stricture in a pancreatic duct with distal dilation or a "double duct sign" in which there is cutoff of both the pancreatic duct and distal bile duct at the level of the genu of the pancreatic duct are pathognomonic  ERCP may be of benefit in patients with biliary obstruction and cholangitis whereupon an endoscopic stent can be placed for decompression.  ERCP is most useful when there is pancreatic duct obstruction, but no mass is evident on either CT or MRI. In this situation, it is necessary to try to distinguish chronic pancreatitis from pancreatic cancer.

 EUS is one of the most common imaging procedures used to diagnose pancreatic cancer.

 It is often the best procedure to obtain samples of a tumor to make a definitive diagnosis of pancreatic cancer.  EUS may be able to find small pancreatic masses that have not been detected by computed tomography (CT) or magnetic resonance imaging (MRI) scans but suspected by the doctor as a result of symptoms and/or blood test results.  Studies show that EUS is equal to or better than CT scans for detection of early pancreatic cancer.

 Biopsy: Biopsy to confirm the presence and identify the type of cancer is usually required before chemoradiation therapy of unresectable pancreatic tumors or neoadjuvant treatment of resectable tumors. Percutaneous biopsy, performed with either CT or ultrasound guidance, or transduodenal biopsy, performed with endoscopic ultrasound guidance, is routinely employed in these situations.

Treatment and prognosis  Most tumours are not resectable at diagnosis.  Surgery for stage I and II does offer the chance of cure, with however high morbidity (20-30%) and mortality (5%). Resection is performed with a Whipple operation.  Even when resection is possible, the majority of patients succumb to recurrence, with only a doubling of survival in operated patients from 5 to 10% at 5 years. Almost a quarter of patients are dead 12 months following diagnosis.

Palliative Nonsurgical Treatment of Pancreatic Cancers

 Establishing the diagnosis and relieving symptoms of jaundice, gastric outlet obstruction, and pain are the goals of palliative nonsurgical treatment.  Tissue diagnosis can usually be made by CT- or ultrasound-guided percutaneous fine- needle aspiration of either the tumor or its metastases.  Transduodenal fine-needle aspiration of the tumor with endoscopic ultrasound guidance and duct cytology obtained by brushings is an alternative method of establishing the diagnosis.  Decompression of the obstructed biliary tract can be achieved using either an endoscopic or a percutaneous-transhepatic approach.  Pancreatic tumors can extend into and obstruct the duodenum, leading to gastric outlet obstruction. This commonly occurs in the second portion of the duodenum in patients with pancreatic head cancers. Pancreatic body tumors can invade the third or fourth portion of the duodenum and also cause obstruction. Many of these patients can be palliated by endoscopic placement of expandable endoluminal metal stents into the duodenum.  . Pain, which is a common symptom of pancreatic cancer, is usually caused by tumor invasion of the peripancreatic neural plexus. Most patients can be adequately treated with orally or transcutaneously administered analgesics.  Narcotic medications may be required. When or if this fails, percutaneous CT-guided or endoscopic ultrasound-guided celiac plexus block may be helpful.

Palliative Surgical Management of Pancreatic Cancer

 Most of the symptoms experienced by patients with unresectable pancreatic cancer can be relieved by nonsurgical means.  Surgical palliation is, for the most part, employed for patients who are undergoing laparotomy for anticipated resectable disease and found to be unresectable at the time of surgery.  In that situation, biliary tract decompression can be achieved by creating either a cholecystojejunostomy or a choledochojejunostomy.  Duodenal obstruction can be managed by creation of a side-to-side gastrojejunostomy in which an antecolic jejunal loop is anastomosed to the posterior wall of the gastric antrum.  Palliation of pain can be achieved, intraoperatively, by injecting alcohol into the celiac plexus, and some surgeons routinely perform operative celiac plexus block at the time of surgical palliation.  However, many patients with unresectable pancreatic cancer can be successfully managed with minimal or no narcotic analgesics, and when more severe pain occurs, results similar to those achieved by intraoperative chemical splanchnicectomy can be achieved using a percutaneous approach.

Chemoradiation Therapy

 Many different protocols for chemoradiation treatment of recurrent or unresectable pancreatic cancer have been described.  The best results have been achieved using radiation therapy combined with either 5- fluorouracil or gemcitabine.

Preoperative preparation of a case of obstructive jaundice.

 Proper diagnosis and assessment.  Injection vitamin K 1M 10 mg for 5 days.  Fresh Frozen plasma‐often requires 6 bottles or more.  Blood transfusion in case of anaemia.  Oral neomycin, lactulose Mannitol 100‐200 ml BD IV to prevent hepato renal syndrome.  Hydration  Repeated monitoring by doing prothrombin time,electrolytes.  Antibiotics like third generation cephalosporins.  Calcium supplements as calcium chloride IV.

2. b) Pathophysiology of acute intestinal obstruction.

Bowel obstruction occurs when the normal flow of intraluminal contents is interrupted. The cause of the obstruction may be external to the bowel (extrinsic), within the wall of the bowel (intrinsic), or due to a luminal defect that prevents the passage of gastrointestinal contents. Obstruction of the can be partial or complete. A type of complete obstruction, a closed-loop obstruction, occurs when the intestine is obstructed at two locations, creating a segment with no proximal or distal outlet. Closed-loop obstruction can rapidly progress to bowel strangulation.

Normal physiology — The main function of the small intestine is to digest and absorb nutrients. Microvilli and (ie, valvulae conniventes, plicae circulares or valves of Kerkring) increase the surface area available for absorption and cause the intestinal contents to twist while flowing through the small intestine. These circular folds can be seen in radiographic studies. The small bowel is relatively free of microbes, whereas the large intestine is populated with commensal bacteria that aid digestion, synthesize a number of vitamins, and break down bilirubin.

Obstructive physiology — Obstruction leads to progressive dilation of the intestine proximal to the blockage, while distal to the blockage the bowel will decompress as luminal contents pass. Swallowed air and gas from bacterial fermentation can accumulate, adding to bowel distention. As the process continues, the bowel wall becomes edematous, normal absorptive function is lost, and fluid is sequestered into the bowel lumen. There may also be transudative loss of fluid from the intestinal lumen into the peritoneal cavity. With proximal bowel obstruction, ongoing emesis leads to additional loss of fluid containing Na, K, H, and Cl, and metabolic alkalosis. These fluid losses can result in hypovolemia. Bacterial overgrowth can also occur in the proximal small bowel, which is normally nearly sterile, and emesis can become feculent.

If bowel dilation is excessive, the intramural vessels of the small intestine become compromised and perfusion to the wall of the intestine is reduced. If perfusion to a segment of intestine is inadequate to meet the metabolic needs of the tissue, ischemia will occur, which will eventually lead to necrosis and perforation unless the process is interrupted. Alternatively, ischemic necrosis of the bowel can be related to twisting of the bowel and/or its mesentery around an adhesive band or to lax intestinal attachments.

Managing a case of intussusceptions:

Diagnosis and investigations: o Ultrasound (diagnostic text of choice): intussusception in cross-section (doughnut or target sign). o Plain X-ray: may show soft tissue mass, small bowel obstruction, free air indicating perforation. o Air (or rarely gastrograffin) contrast enema is diagnostic and may be therapeutic. Treatment: o Immediate IV fluid resuscitation to correct fluid losses and to restore fluid, electrolyte, and acid/base balance. o Maintenance fluid replacement and replacement of continued losses (vomiting or nasogastric losses). Reduction only attempted once fluid balance restored. o Analgesia and sedation (morphine 0.2mg/kg) will aid process of reduction.

Methods of reduction: Radiological reduction o Air enema therapeutic in 75% of cases. o Usually performed in radiology department under screening control. o Surgeon should be present. o Evidence of irreducible obstruction or perforation mandates immediate halt. o Partial or incomplete reduction may warrant repeat attempt after 4-6 hours. o Informed consent includes risk of peforation. Surgical reduction: o Laparotomy indicated without enema if evidence of peritonitis or perforation. o Manual reduction by retrograde squeezing and gentle proximal traction. o Resection and anastomosis if bowel viability is in doubt (~10% require resection). o Post-reduction septic shock may occur with release of bacterial products from viable but damaged bowel segment. o Most recover rapidly with resumption of oral feeding in 24-48h and discharge home in 4-5 days.

3. Answers. a) Sentinel lymph node biopsy.

Sentinel node biopsy is a surgical procedure used to determine if cancer has spread beyond a primary tumor into your lymphatic system. Sentinel node biopsy is used most commonly in evaluating breast cancer and melanoma.

The sentinel nodes are the first few lymph nodes into which a tumor drains. Sentinel node biopsy involves injecting a tracer material that helps the surgeon locate the sentinel nodes during surgery. The sentinel nodes are removed and analyzed in a laboratory. If the sentinel nodes are free of cancer, then cancer isn't likely to have spread and removing additional lymph nodes is unnecessary.

If, after sentinel node biopsy, evaluation of the sentinel nodes reveals cancer, then you'll likely need additional lymph nodes removed for your doctor to determine how far the cancer has spread.

Sentinel node biopsy is recommended for people with certain types of cancer in order to determine whether the cancer cells have migrated into the lymphatic system.

Sentinel node biopsy is routinely used for people with:

 Breast cancer  Melanoma

Sentinel node biopsy is sometimes used in other types of cancer, such as:

 Colon cancer  Esophageal cancer  Head and neck cancer  Non-small cell lung cancer  Stomach cancer  Thyroid cancer

Sentinel node biopsy is generally a safe procedure. But as with any surgery, sentinel node biopsy carries a risk of complications, including:

 Bleeding  Pain or bruising at the biopsy site  Infection  Allergic reaction to the dye used for the procedure  Lymphedema — a condition in which your lymph vessels are unable to adequately drain lymph fluid from an area of your body, causing fluid buildup and swelling b) Alvarado Score.

Introduction:

The Alvarado score is a clinical scoring system used in the diagnosis of appendicitis. The score has 6 clinical items and 2 laboratory measurements with a total 10 points. It was introduced in 1986 and although meant for pregnant females, it has been extensively validated in the non-pregnant pregnant population. The modified Alvarado score is at present in use.

c) Parotid abscess.

Parotid abscess

A parotid abscess is a buildup of pus and infected material in the parotid gland, the largest of the salivary glands surrounding the jaw.

Patients at risk:  This condition most commonly develops as a consequence of poor oral care, usually when people are recovering from a medical condition and are less capable of following a basic oral hygiene routine.  Immunocompromised patients.  Diabetics.

Bacteriology: Staphylococcus aureus, Klebsiella pneumoniae, Escherichia coli and Alpha- haemolytic streptococci.

Symptoms:  Symptoms of a parotid abscess include significant pain and swelling of one or both parotid glands. The area behind the jaw can feel tight and hot, and the skin may be red and shiny. Diagnosis:  A careful physical examination to rule out conditions with similar symptoms, like mumps.  Examination of the mouth may also show leakage of pus from the parotid gland into the oral cavity and the patient may have foul-smelling breath as a result of the infection.

Treatment: Treatment usually involves drainage, medications to treat infection, and monitoring for signs of recurrence.

d) Mesenteric cyst:

Mesenteric cysts are a very rare cause of abdominal pain, and have a wide range of underlying causes. Mesenteric cysts most commonly occur in the small-bowel mesentery on the mesenteric side of the bowel.

Mesenteric and omental cysts can be either simple or multiple and either unilocular or multilocular, and they may contain hemorrhagic, serous, chylous, or infected fluid. [7] The fluid is serous in ileal and colonic cysts and is chylous in jejunal cysts. They can range in size from a few millimeters to 40 cm in diameter.

Epidemiology

Mesenteric cysts are rare, with a reported incidence of 0.5-1 per 100,000 admissions.

Clinical presentation: Patients usually present with abdominal pain and/or mass, although they can be an asymptomatic, incidental finding.

Aetiology

 lymphatic: simple lymphatic cyst and lymphangioma  mesothelial: simple mesothelial cyst, benign cystic mesothelioma, and malignant cystic mesothelioma  enteric: enteric cyst and enteric duplication cyst  urogenital  mature cystic teratoma (dermoid cysts)  non-pancreatic pseudocysts (infectious and traumatic cysts)

Classification of mesenteric cysts:

Radiographic features

Mesenteric cysts can occur anywhere in the mesentery, from the duodenum to the rectum, and may extend into the retroperitoneum. Mesenteric cysts are cystic mesenteric lesions that can be further characterized on the wall thickness (thin or thick walled) and their loculation (unilocular or multilocular).

 Lymphangioma is a benign lesion of vascular origin. Most lymphangiomas are located in the neck, but 5% of lymphangiomas are abdominal. Lymphangioma has enhancing septa. Unlike in cystic peritoneal metastases, ascites is not a feature of lymphangioma. Lymphangioma is often closely associated with the small bowel. At surgery it is usually very difficult to separate the tumor from the bowel and in many cases the bowel also has to be resected.

 Enteric duplication cyst is a cyst with a wall that has all three layers of the bowel wall, i.e. mucosa, and muscularis propria. They may occur anywhere in the mesentery, so either adjacent to or away from the bowel.

The diagnosis should be made by use of all standard methods of abdominal tumor diagnosis, with ultrasonography (US) and computed tomography (CT), and especially nuclear magnetic resonance providing most information of the growth size and localization. Total cystectomy is the therapeutic method of choice. Open method has been preferred, although reports on successful cystectomy by the laparoscopic method have already appeared in the literature.

Intraoperative differentiation between lymphatic cyst and lymphangioma is of utmost importance, and can only be achieved by pathohistologic examination of the cyst wall. If intraoperative biopsy cannot be performed or the finding is uncertain, each cyst should be extirpated in toto due to the above mentioned risk associated with cystic lymphangioma. e) Gastrinoma:

• 75% sporadic

• 25% MEN-1: ZES usually occurs after the onset of hyperparathyroidism in MEN-1 and occurs 50% of the time.

Gastrinomas:

• 2nd most common islet cell tumor

• 50% in duodenum

• Hypergastrinemia with increased acid output: Hypergastrinemiais responsible for hypertrophy of the ,including the gastric enterochromaffin cells, which in turn increases the number of parietal cells. This increase causes a greater maximal gastric acid output. A secondary action of hypergastrinemia is the stimulation of acid secretion resulting in increased basal acid secretion. • Passaro's Triangle. In 70% to 90% of patients, the primary gastrinoma is found in Passaro's triangle, an area defined by a triangle with points located at the junction of the cystic Location duct and common bile duct, the • 1. Gastrinoma triangle second and third portion of the 90% duodenum, and the neck and body of the pancreas. Investigations : • 2. Somatostatin receptor scintigraphy • 3. Ultrasound, CT, MRI unreliable • 4. Endoscopic ultrasound adjunct at surgery

Presentation

• 1. Abdominal pain • 1. Ulcers in atypical • 2. Ulcer on EGD locations. • 3. Secretory diarrhea • 2. Multiple ulcers • 4. calcium, PTH or • 3. Ulcers that fail to respond pituitary tumor with to conventional treatment MEN-1 • 4. Ulcers that recur after • 5. Recurrence of ulcer conventional treatment despite adequate • 5. Peptic ulcer disease in treatment. association with diarrhea • 6. Ulcers in association with hyperparathyroidism.

Clinical Presentation: Many patients with ZES present with abdominal pain, peptic ulcer disease, and severe esophagitis. Aspects of Peptic Ulcer Disease That Raise Suspicion for Zollinger-Ellison Syndrome

1. Ulcers in atypical locations.

2. Multiple ulcers

3. Ulcers that fail to respond to conventional treatment

4. Ulcers that recur after conventional treatment

5. Peptic ulcer disease in association with diarrhea

6. Ulcers in association with hyperparathyroidism

Investigations:

The diagnosis of ZES is made by measuring the serum gastrin level. It is important that patients stop taking proton pump inhibitors for this test. In most patients with gastrinomas, the level is greater than 1000 pg/ml, but it can be in the 150 to 1000 pg/ml range or even normal.

In equivocal cases, when the gastrin level is not markedly elevated, a secretin stimulation test is helpful. A rise of more than 200 pg/ml confirms the diagnosis of gastrinoma.

Tumor Localization

The next step is to localize the gastrinoma(s) and to determine if metastases are present. This is best accomplished preoperatively if possible. In 70% to 90% of patients, the primary gastrinoma is found in Passaro's triangle; an area defined by a triangle with points located at the junction of the cystic duct and common bile duct, the second and third portion of the duodenum, and the neck and body of the pancreas.

The test of choice is somatostatin receptor scintigraphy in combination with computed tomography (CT).

Endoscopic ultrasound (EUS) is another new modality that assists in the preoperative localization of gastrinomas. It is particularly helpful in localizing tumors in the pancreatic head or duodenal wall where gastrinomas are usually less than 1 cm. A combination of octreoscan and EUS detects more than 90% of gastrinomas.

Surgery

Pharmacologic control of acid secretion has rendered total gastrectomy unnecessary. Omeprazole therapy is so effective that every patient with ZES is a candidate for a tumor removal operation until proved otherwise because of systemic illness or widespread metastases. Although gastrinomas have a high rate of malignancy, efforts at surgical cure are clearly justified. Every attempt is made to localize the tumor before surgery, and CT and MRI are effective with larger tumors and especially with hepatic metastases. Gastric secretion is controlled during the perioperative period with either oral or parenteral proton pump inhibitors.

Intraoperative ultrasonography plus palpation is effective in localizing 90% to 98% of pancreatic gastrinomas.

Tumors within the pancreas are enucleated if at all possible.

Treatment of metastatic disease has undergone serial changes but is still unsatisfactory. Radiation therapy and chemotherapy are largely ineffective. The combination of doxorubicin, streptozotocin, and 5-fluorouracil has a low, temporary response rate, but it is highly toxic and has no impact on survival.

4. Answers. a) Volkman’s ischaemic contracture. Definition: Volkmann's ischaemic contracture is a permanent flexion contracture of the hand at the wrist, resulting in a claw-like deformity of the hand and fingers. It is more common in children. Passive extension of fingers is restricted and painful.

On examination the fingers are white or blue and cold and the radial pulse is absent.

Causes:  Any fracture in elbow region or upper arm may lead to Volkmann's ischemic contracture but commonly caused due to supracondylar fracture of the humerus.  Volkmann's contracture results from acute ischaemia/necrosis of the muscle fibres of the flexor group of muscles of the forearm, especially M. flexor digitorum profundus and M. flexor pollicis longus which becomes fibrotic and short.  It is caused by obstruction on the brachial artery near the elbow, possibly from improper use of a tourniquet, improper use of a plaster cast, or compartment syndrome.  It is also caused by fracture of forearm bones which cause profuse bleeding from major blood vessels of forearm. Classification:  Holden Classification: o Level I—Injury is proximal to the ischemia and later contracture, as in a brachial artery injury . o Level II—Ischemia is directly under the injury (pressure)  Tsuge classification is based on the three levels of severity of Volkmann’s Contracture and is outlined below: o Mild—resulting contracture of 2-3 fingers only o Moderate—all fingers are flexed with thumb flexed in palm, wrist in flexion and partial loss of sensation in the hand o Severe—all muscles that flex and extend wrist and fingers are involved. Treatment:  Surgery to release the fixed tissues may help with the deformity and function of the hand.  All splints, plaster and bandages that might be obstructing the circulation should be removed and fascial compartment should be opened (fasciotomy) in the incipient stage. Prevention:

 Requires restoration of blood flow;  Reduction of compartmental pressure Management:

 Proper initial splinting of hand in the function position;  Release of forearm flexors:  Muscle slide  Tendon lengthening.  Test for intrinsic tightness  Free functional muscle transfer b) Pathological fracture. Definition: A pathological facture is one in which a bone is broken, through an area, weakened by pre-existing disease, by a degree of stress, that would have left the normal bone intact. In other words, a fracture involving “abnormal bone” is a pathological fracture. Etiology:

 Development disorders of bone : o Congenital defects of bone tissue : Osteogenesis imperfecta Osteopetrosis o Disorder of cartilage growth : Achondroplasia Diaphysealaclasis (multiple exostosis) Dyschondroplasia (Ollier’s disease) o Nutritional and vitamin deficiencies: Scurvy, Rickets, Osteomalacia  Hormonal imbalance : o Hyperparathyroidism ,Cushing’s syndrome. o Pathological fracture from cortisone treatment Frohlich’s syndrome (hypopituitarism)  Atrophic conditions of bone : Disuse osteoporosis Senile osteoporosis  Pathological fracture through infected bone : Osteomyelitis  Cystic disorders and fibrous dysplasia of bone : o Unicameral bone cyst o Aneurysmal bone cyst o Non – osteogenic fibroma of bone o Monostotic and polyostotic fibrous dysplasia o Paget’s disease of bone  Primary and secondary tumors of bone: o Primary benign tumours: Chondroma, Benign chondroblastoma, Chondromyxoid fibroma, Haemangioma of bone ,Giant cell tumour of bone,Disappearing bone disease. o Malignant tumours : Osteosarcoma,Chondrosarcoma ,Fibrosarcoma,Malignant fibrous histiocytoma, Malignant round cell tumour, Multiple myelomatosis o Metastatic tumours of bone – lungs, thyroid, kidney, GI tract, prostrate  Marrow cell disorder: Histiocytos Gaucher’s disease  Parasitic disease of the bone : Hydatid disease  Neurotrophic dystrophies of the bone : Tabes dorsalis ,Syringomyelia, Diabetic neuropathy  Iatrogenic pathological fracture : o Through screw hole stress protection phenomenon o Through biopsy o After removal of infected bone o Through a donor site for a bone graft

Pathological fracture is suspected when fracture occurs:

 Spontaneously  After minor trauma  Unusual fracture pattern  History of recent several fractures  Older patient  History of primary malignancy

Investigations:

 Radiography:

Plain –X-ray:

 Study the Fracture  Lesion location: Usually eccentric, Cortical involvement, Diaphyseo-metaphyseal junction  Don’t ignore the perifracture  Densities within the lesion: Bone formation changes suggests – Osteosarcoma;Calcification suggests – Chondrosarcoma  Other lesions  Reaction (periosteal / endosteal) should be examined.  Alteration in density and  Zone of transition architecture  Extra osseous masses or  Moth eaten or permeative pattern of bone reaction abnormalities

Laboratory studies:

General investigations Search for occult primary carcinoma

 Complete haemogram o Breast - Examination Mammography  Peripheral smear o Lung – Chest X-ray  Serum glucose o Kidney – Ultrasonography  Serum albumin o Thyroid – Digital palpation  Serum calcium, Phosphate o Prostate – Serum PSA, Digital prostate  Alkaline phosphatase examination o  LFT Myeloma – Bone marrow examination;  Urine sugar and albumin Bence-Jones proteins; Serum and urine  Bence-Jones proteins electrophoresis o Skeletal Survey - X-ray skull, spine and  Serum electrophoresis pelvis.  Tumor markers: Ca 125, Ca 19.9, CEA o Other organs

 MRI:  Marrow disease  Epidural and nerve root compression can be detected  Localize the disease  Bone scan  PET scan

Treatment:

o Management of pain o Avoiding the fracture o Bone stabilization o Conservative measures

Systemic Therapy:

 Hormone therapy : Ca breast and prostate  Chemotherapy  Bisphosphonates : Zolindronic acid  Targeted therapy : Denosumab

Radiotherapy :

 External-beam radiotherapy  Stereotactic Body Radiotherapy(SBRT)  Radiopharmaceuticals : Unsealed source therapy with bone-seeking radionueclides

Surgical management:

 Indication: Palliative o Fracture: o Ambulation / Pain relief o Impending fracture c) Congenital talipes equinovarus:

Congenital talipes equinovarus (CTEV) is considered the most common anomaly affecting the feet diagnosed on antenatal ultrasound.

Terminology

While some use CTEV and club foot (CF) synonymously, in certain publications term clubfoot is considered a more general descriptive term that describes three distinct abnormalities:

 talipes equinovarus (adduction of the forefoot, inversion of the heel and plantar flexion of the forefoot and ankle)  talipes calcaneovalgus (dorsal flexion of the forefoot with the plantar surface facing laterally)  metatarsus varus (inversion and adduction of the forefoot alone)

Epidemiology

 It carries an estimated incidence of 0.5-5% of live births.  The condition is bilateral in more than half of cases.

Pathology

 The deformity involves both ankle and subtalar joints. The malalignment is fixed by joint, ligamentous and tendinous contractures.  There may be a familial predilection in ~15% of cases . If detected antenatally, a careful search for other associated anomalies are recommended.

Associations:

 chromosomal anomalies o 18q deletion syndrome o trisomy 18 o Wolf-Hirschhorn syndrome  other syndromic conditions o Freeman Sheldon syndrome o Meckel Gruber syndrome o Roberts syndrome  renal anomalies o prune belly syndrome o renal agenesis  connective tissue disorders o Marfan syndrome o Ehlers-Danlos syndrome  spinal anomalies o caudal regression syndrome o diastematomyelia o spina bifida  skeletal dysplasias o diastrophic dysplasia

Radiographic features

Plain radiograph

Assessment requires weightbearing DP and lateral radiographs. Where weightbearing is not possible, it should be simulated.

Talipes equinovarus consists of four elements:

 hindfoot equinus: lateral talocalcaneal angle less than 35degree  hindfoot varus: talocalcaneal angle less than 20degree  metatarsus adductus: adduction and varus deformity of the forefoot; talus to first metatarsal angle greater than 15 degree.  talonavicular subluxation: medial subluxation of the navicular on the talus

Ultrasound:

 Approximately 10-20% of individuals with prenatal diagnosed clubfoot may have a normal foot or positional foot deformity requiring minimal treatment.  Sonographic features may vary dependent on severity. Both the tibia and fibula may be on the same image as the medially deviated foot and the foot may additionally appear plantar flexed.

Treatment and prognosis:

 Many (at least 50%) require orthopaedic intervention. The Ponseti method is a manipulative technique that corrects congenital clubfoot without invasive surgery.  Therapeutic options range from casting and manipulation, through to surgical release.  The most common surgical complication is overcorrection resulting in a "rockerbottom" flat foot deformity.  Overall prognosis will often depend on other associated condition. Isolated condition may be treated successfully. d) Radial nerve injury due to fracture.  The Injury  Primary nerve injury Injury mechanism o Fall: most common mechanism o High energy: MVA, fall from height o Iatrogenic: surgical treatment of humerus/humeral ORIF o Compression/contusion  Secondary nerve injury (after reduction or manipulation of humeral shaft fracture) Nerve may become incarcerated in the fracture after a reduction Incidence:  9‐12% of fractures of the humerus (about 1 in 10) Fracture patterns: More common fracture patterns: . Spiral (OTA A1) . Transverse (OTA A3) . Open fractures . Comminuted patterns less common (OTA C1,2,3) Fractures from the middle to the distal third of humerus most common for causing radial nerve palsy. Transverse and spiral fractures the most common types associated with nerve injury. High incidence of nerve entrapment or transection following open humeral fractures.

Nerve trauma Neuropraxia v. axonomesis o Stretch or shear injury to the nerve: Most common o Nerve entrapment in the fracture Rarely complete neruotomesis Can occur from sharp bone ends lacerating nerve (high energy, open fracture) The Anatomy Radial nerve in contact with the humerus Mid‐humerus, spiral groove Junction of middle third and distal third (pierces lateral intermuscular septum) Radial nerve most at risk in these locations Surface anatomy: Radial nerve palpable along lateral border of the humerus Radial nerve at level of the lateral epicondyle to form posterior interosseous nerve Radial nerve innervates the brachioradialis muscle, 2cm proximal to lateral epicondyle. Nerve Injury and Recovery Assessment Diagnosis Clinical o Motor assessment: Brachioradialis and ECRL are the first muscles to recover, extensor indicis proprius is the last o Sensory assessment: Recovery of the superficial radial nerve sensation often delayed Advancing Tinel’s sign Imaging: Ultrasound Can detect entrapment or radial nerve transection Electrophysiology studies EMGs findings indicative of nerve injury o Fibrillation potentials o Insertional activity o Positive sharp waves, turns, polyphasic signal o Reduced recruitment Timing of recovery: Reinnervation occurs at 1mm/day with one month delay Recovery would be expected ~ 4 mo. from injury, under “ideal” circumstances

Non‐surgical Treatment: o Monitoring for recovery of the radial nerve o Radial nerve palsy splinting, 8 hours per day to prevent contracture o Stretching exercises of the wrist and digits, daily o Electrical stimulation of extensor musculature of forearm.

Surgical Treatment o Recommended at 6 months if no clinical or EMG evidence of recovery o Neurolysis if nerve/neuroma is intact and conducts o Resection of damaged/non‐conducting nerve segment/sural nerve graft reconstruction Primary repair: desirable, usually not feasible Tendon transfers . Pronator teres to extensor carpi radialis brevis . Flexor carpi radialis to extensor digitorum communis to index, long, ring and small . Palmaris longus tendon to rerouted extensor pollicis longus. d) Perthes disease.

Legg-Calve-Perthes disease (LCPD) is the name given to idiopathic osteonecrosis or avascular necrosis of the capital femoral epiphysis of the femoral head. In 15-20% of patients with LCPD involvement is bilateral. in the US one in 1200 children younger than 15 years is affected by LCPD.

Epidemiology:

 the disease is most commonly seen at the age of 4-8 years.  Males are affected 4-5 times more often than females.  Caucasians are affected more frequently than persons of other races.

Symptoms of LCPD usually are present for several weeks or longer before the disease is detected. Often the child does not complain. Typical complants are hip or groin pain, limping, no history of trauma.

Etiology of LCPD is unclear, it is accepted that the blood supply to the capital femoral epiphysis is interrupted causing bone infarction. Revascularisation starts and subsequently new bone formation is started. At this point a percentage of patients develops LCPD.

The Catterall classification is based on radiographic appearances and is as follows:

 Stage I: histologic and clinical diagnosis without radiographic findings  Stage II: sclerosis with or without cystic changes with preservation of the contour and surface of femoral head  Stage III: loss of structural integrity of the epiphysis  Stage IV: loss of structural integrity of the acetabulum in addition

The Salter-Thomson classification reduces the Catterall groups to 2:

 Group A includes Catterall groups I and II. Less than 50% of the head is involved in this group.  Group B includes Catterall groups III and IV and more than 50% of the epiphysis is involved.

Investigations: X-rays of the hip may suggest and/or verify the diagnosis. X-rays usually demonstrate a flattened, and later fragmented, femoral head. A bone scan or MRI may be useful in making the diagnosis in those cases where X-rays are inconclusive. Usually, plain radiographic changes are delayed 6 weeks or more from clinical onset, so bone scintigraphy and MRI are done for early diagnosis. MRI results are more accurate, i.e. 97 to 99% against 88 to 93% in plain radiography. If MRI or bone scans are necessary, a positive diagnosis relies upon patchy areas of vascularity to the capital femoral epiphysis (the developing femoral head).

Treatment:

Therapy: limited running and jumping, and medications as needed.

Other nonsurgical treatments include:

 Physical therapy  Crutches.  Traction.  Casts.

Surgery: Most of the orthopedic treatments for Legg-Calve-Perthes disease are aimed at improving the shape of the hip joint to prevent arthritis later in life.

 Contracture release. Children who have Legg-Calve-Perthes often prefer to hold their leg across the body. This tends to shorten nearby muscles and tendons, which may cause the hip to pull inward (contracture). Surgery to lengthen these tissues may help restore the hip's flexibility.  Joint realignment. For children older than 6 to 8, realignment of the joint has been shown to restore a more normal shape to the hip joint. This involves making surgical cuts in the femur or pelvis to realign the joints. The bones are held in place with a plate while the bone heals.  Removal of excess bone or loose bodies. In older children with painful, restricted motion, trimming extra bone around the femoral head or repairing damaged cartilage may ease motion and relieve pain. Loose bits of bone or torn flaps of cartilage can be removed.  Joint replacement. Children who have had Legg-Calve-Perthes sometimes require hip replacement surgery later in life. These surgeries can be complicated because of a higher risk of bone fracture and nerve damage.

The West Bengal University of Health Sciences

M.B.B.S. 3rd Professional Part – II Examination, 2016

Subject: Surgery Time: 21/2 hrs.

Paper: I Marks: 60

Group – A 1) Classify hemorrhage. Discuss briefly the management of hemorrhagic shock. Mention complications of blood transfusion. 5+5+5 = 15

Answer. Types of haemorrgage:

Recognition of types of haemorrhage:

Arterial/ venous/capillary:

 Arterial haemorrhage: Arterial haemorrhage is recognised as bright red blood, spurting as a jet which rises and falls in time with the pulse. In protracted bleeding, and when quantities of intravenous fluids other than blood are given, it can become watery in appearance.  Venous haemorrhage: Venous haemorrhage is a darker red, a steady and copious flow. The colour darkens still further from excessive oxygen desaturation when blood loss is severe, or in respiratory depression or obstruction. Blood loss is particularly rapid when large veins are opened, e.g. common femoral or jugular. Venous bleeding can be under increased pressure as in asphyxia, or from ruptured varicose veins  Capillary haemorrhage: Capillary haemorrhage is bright red, often rapid, ooze. If continuing for many hours, blood loss can become serious, as in haemophilia.

Primary/reactionary/secondary:

 Primary haemorrhage: Primary haemorrhage occurs at the time of injury or operation. Reactionary haemorrhage  Reactionary haemorrhage: may follow primary haemorrhage within 24 hours (usually 4 6 hours) and is mainly due to rolling ( slipping ) of a ligature, dislodgement of a clot or cessation of reflex vasospasm. —  Secondary haemorrhage: Secondary‘ haemorrhage’ occurs after 7 14 days, and is due to infection and sloughing of part of the wall of an artery. Predisposing factors are pressure of a drainage tube, a fragment of bone, a ligature in an infected area or— cancer. It is also a complication of arterial surgery and amputations

External/internal:

 External haemorrhage: External haemorrhage is visible, revealed haemorrhage.  Internal haemorrhage: Internal haemorrhage is invisible, concealed haemorrhage. Internal bleeding may be concealed as in ruptured spleen or liver, fractured femur, ruptured ectopic gestation or in cerebral haemorrhage. Concealed haemorrhage may become revealed as in haematemesis or melaena from a bleeding peptic ulcer, as in haematuria from a ruptured kidney, or via the vagina in accidental uterine haemorrhage of pregnancy.

Classes of haemorrhage

Methods of determining acute blood loss:

Methods of determining acute blood loss:

Assessment and management of blood loss must be related to the pre-existing circulating blood volume, which can be derived from the patient s weight: 85 ml/kg; 75 ml/kg. ’ •Measuring Infant 80— blood loss • AdultBlood 65— clot: The size of a clenched fist is roughly equal to 500 ml. Swelling in closed fractures: Moderate swelling in closed fracture of the tibia equals 500 1500 ml blood loss. Moderate swelling in a fractured shaft of femur equals 500 2000 ml blood loss. • Swab weighing. In the operating theatre, blood loss can be measured by weighing the swabs— after use and subtracting the dry weight. The resulting total obtained (1 g = 1 ml)— is added to the volume of blood collected in the suction or drainage bottles. In extensive wounds and operations, the blood loss is grossly underestimated, due to evaporation of water from the swabs before weighing each batch.Prompt transfer of discarded swabs into polythene bags reduces this source of error. Blood, plasma and water are also lost from the vascular system because of evaporation from open wounds, into the tissues, sweating and expired water via the lungs. Indeed, for operations such as radical mastectomy or partial gastrectomy it may be necessary to multiply the swab weighing total by a factor of 1.5. For prolonged surgery via larger wounds, as in abdominothoracic or abdominoperineal operations, the total measured may need to be multiplied by 2.  Haemoglobin level This is estimated in g/100 ml (g/dl), normal values being 12 16 g/100 ml (12 16 g/dl). There is no immediate change in haemorrhage, but after some hours the level falls by influx of interstitial fluid info the vascular compartment in order to restore the blood —volume. —  Measurement of central venous pressure  Continuous tissue oxygen tension measurement

Managing internal bleeding:

ABC s

High’ concentration oxygen Assist ventilations

Control external bleeding

Stabilize fractures

RICE resuscitation, investigations, clinical examination, evaluation

Transport– rapidly to appropriate facility Control of external bleeding:

Pressure Dressing: Use bandage to secure dressing in place

Tourniquets:

o Final resort when all else fails o Used for amputations o 3-4 wide (blood pressure cuffs) o Write TK and time of application on forehead of patient o Notify” other personnel o Once applied,“ ” DO NOT REMOVE

Treatment of haemorrgagic shock:

The primary treatment of hemorrhagic shock is to control the source of bleeding as soon as possible and to replace fluid. o In controlled hemorrhagic shock (CHS), where the source of bleeding has been occluded, fluid replacement is aimed toward normalization of hemodynamic parameters. In uncontrolled hemorrhagic shock (UCHS), in which the bleeding has temporarily stopped because of hypotension, vasoconstriction, and clot formation, fluid treatment is aimed at restoration of radial pulse or restoration of sensorium or obtaining a blood pressure of 80 mm Hg by aliquots of 250 mL of lactated Ringer's solution (hypotensive resuscitation). o When evacuation time is shorter than 1 hour (usually urban trauma), immediate evacuation to a surgical facility is indicated after airway and breathing (A, B) have been secured ("scoop and run"). Precious time is not wasted by introducing an intravenous line. When expected evacuation time exceeds 1 hour, an intravenous line is introduced and fluid treatment is started before evacuation. The resuscitation should occur before, or concurrently with, any diagnostic studies. o Crystalloid is the first fluid of choice for resuscitation. Immediately administer 2 L of isotonic sodium chloride solution or lactated Ringer s solution in response to shock from blood loss. Fluid administration should continue until the patient's hemodynamics become stabilized. Because crystalloids quickly leak from the vascular’ space, each liter of fluid expands the blood volume by 20-30%; therefore, 3 L of fluid need to be administered to raise the intravascular volume by 1 L. o Alternatively, colloids restore volume in a 1:1 ratio. Currently available colloids include human albumin, hydroxy-ethyl starch products (mixed in either 0.9% isotonic sodium chloride solution or lactated Ringer s solution), or hypertonic saline-dextran combinations. The sole product that is avoided routinely in large-volume (>1500 mL/d) restoration is the hydroxy-ethyl starch product mixed’ in 0.9% isotonic sodium chloride solution because it has been associated with the induction of coagulopathy. The other products have not been so implicated. o In patients with hemorrhagic shock, hypertonic saline has the theoretical benefit of increasing intravascular volume with only small amounts of fluid. The combination of dextran and hypertonic saline may be beneficial in situations where infusion of large volumes of fluid may be harmful, such as in elderly persons with impaired cardiac activity. Additional trials will be required before this combination is accepted as standard of care. o PRBCs should be transfused if the patient remains unstable after 2000 mL of crystalloid resuscitation. For acute situations, O-negative noncrossmatched blood should be administered. Administer 2 U rapidly, and note the response. For patients with active bleeding, several units of blood may be necessary. o There are recognized risks associated with the transfusion of large quantities of PRBCs. As a result, other modalities are being investigated. One such modality is hemoglobin-based oxygen carriers (HBOC). Clinical application has been limited by its toxic effect profile. o If at all possible, blood and crystalloid infusions should be delivered through a fluid warmer. A blood sample for type and cross should be drawn, preferably before blood transfusions are begun. Start type-specific blood when available. Patients who require large amounts of transfusion inevitably will become coagulopathic. FFP generally is infused when the patient shows signs of coagulopathy, usually after 6-8 U of PRBCs. Platelets become depleted with large blood transfusions. Platelet transfusion is also recommended when a coagulopathy develops. Complications of Blood Transfusion: Complications from a single transfusion include:  Incompatibility haemolytic transfusion reaction  Febrile transfusion reaction  Allergic reaction  Infection o Bacterial infection (usually due to faulty storage) o Hepatitis o HIV o Malaria  Air embolism  Thrombophlebitis  Transfusion-related acute lung injury (usually from FFP).

Complications from massive transfusion include:  Coagulopathy  Hypocalcaemia  Hyperkalaemia  Hypokalaemia  Hypothermia.

Categories and Management of Adverse Transfusion Reactions:

INCIDENC MANIFESTA DIAGNOSTIC THERAPEUTIC/PROP E ETIOLOGY TION TESTING HYLACTIC APPROACH

Acute (<24 Hours) Transfusion Reactions—Immunologic

Hemolytic 1 : 38,000- Red cell Chills, fever, Clerical Keep urine output

1 : 70,000 incompatibili hemoglobin check >100 mL/hr ty uria, Analgesics (may hypotension DAT , renal need morphine) failure with Visual Pressors for

oliguria, DIC inspection hypotension (oozing (free Hemostatic from IV hemoglobin) sites), back components Repeat (platelets, pain, pain along the patient ABO, cryoprecipitate, infusion pre- and plasma) for vein, anxiety post- bleeding transfusion sample Further tests as indicated to define possible incompatibili ty Further tests to detect hemolysis (LDH, bilirubin, etc.)

INCIDENC MANIFESTA DIAGNOSTIC THERAPEUTIC/PROP E ETIOLOGY TION TESTING HYLACTIC APPROACH

Fever/chill, RBCs: Antibod Fever, Rule out Antipyretic nonhemolyti 1 : y to chills/rigors, hemolysis premedication c 200-1 donor headache, (DAT, (acetaminophen,

: 17 WBCs vomiting inspect for no aspirin)

(0.5% hemoglobine Accumul Leukocyte- -6%) mia, repeat ated reduced blood

patient ABO) Platel cytokine ets: s in Rule out 1:100 platelet bacterial -1:3 unit contaminatio (1%- n 38%)

WBC

antibody

screen

Urticarial 1 : 100-1 : Antibody to Urticaria, No testing needed Antihistamine, 33 (1%- donor plasma pruritus, treatment or

3%) proteins flushing premedication (PO or IV) May start unit slowly after

antihistamine if symptoms resolve

Anaphylactic 1 : 20,000- Antibody to Hypotension Rule out Trendelenburg

1 : 50,000 donor plasma , urticaria, hemolysis position proteins bronchospas (DAT, Fluids (includes IgA, m inspect for

haptoglobin, (respiratory hemoglobine Epinephrine C4) distress, mia, repeat (adult dose: 0.3- wheezing), patient ABO) 0.5 mL of 1 : 1000

local edema, solution SC or IM;

anxiety Anti-IgA in severe cases, 1 :

10,000 IV) IgA, Antihistamines, quantitative

corticosteroids, 2-agonists IgA- deficient blood componentsβ

Transfusion- 1 : 5000-1 : WBC Hypoxemia, Rule out Supportive care related acute 190,000 antibodies in respiratory hemolysis until recovery lung injury donor failure, (DAT, Defer implicated (occasionally hypotension inspect for in recipient), , fever, INCIDENC MANIFESTA DIAGNOSTIC THERAPEUTIC/PROP E ETIOLOGY TION TESTING HYLACTIC APPROACH

other WBC- bilateral hemoglobine donors activating pulmonary mia, repeat agents in edema patient ABO) components WBC antibody screen in donor and

recipient. If

positive, antigen typing may be indicated WBC crossmatch

Chest x-ray

Acute (<24 Hours) Transfusion Reaction—Nonimmunologic

Transfusion- Varies by Bacterial Fever, chills, Broad-spectrum Gram stain associated component contaminatio hypotension antibiotics (until sepsis n sensitivity testing Culture of completed) component Treat Patient complications culture (e.g., shock) Rule out hemolysis (DAT,

inspect for

hemoglobine mia, repeat patient ABO)

Hypotension Dependent Inhibited Flushing, Rule out Withdraw ACE associated on clinical metabolism hypotension hemolysis (DAT, inhibition with ACE setting of bradykinin inspect for inhibition with infusion hemoglobinemia, Avoid albumin volume of bradykinin repeat patient (negatively ABO) replacement for charged plasmapheresis filters) or Avoid bedside activators of leukocyte prekallikrein filtration

INCIDENC MANIFESTA DIAGNOSTIC THERAPEUTIC/PROP E ETIOLOGY TION TESTING HYLACTIC APPROACH

Circulatory <1% Volume Dyspnea, Chest x-ray Upright posture overload overload orthopnea, cough, Oxygen tachycardia, IV diuretic

hypertensio (furosemide) n, headache Phlebotomy (250-

mL increments)

Nonimmune Rare Physical or Hemoglobin Rule out Identify and eliminate hemolysis chemical uria, patient cause destruction hemoglobin hemolysis of blood emia (DAT, (heating, inspect for freezing, hemoglobine hemolytic mia, repeat drug or patient ABO) solution added to Test unit for blood) hemolysis

Air embolus Rare Air in Sudden X-ray for Place patient on left infusion line shortness of intravascular air side with legs elevated breath, above chest and head acute cyanosis, pain, cough, hypotension , cardiac dysrhythmia

Hypocalcemi Dependent Rapid citrate Paresthesia, Ionized Slow calcium a (ionized on clinical infusion tetany, calcium infusion while calcium) setting (massive dysrhythmia monitoring Prolonged transfusion of ionized calcium QT interval citrated levels in severe on blood, cases delayed electrocardio metabolism gram PO calcium

of citrate, supplement for apheresis mild symptoms procedures) during apheresis procedures

Hypothermia Dependent Rapid Cardiac Central body Use blood warmer on clinical infusion of dysrhythmia temperature setting cold blood Delayed (>24 Hours) Transfusion Reactions—Immunologic INCIDENC MANIFESTA DIAGNOSTIC THERAPEUTIC/PROP E ETIOLOGY TION TESTING HYLACTIC APPROACH

Alloimmuniz 1 : 100 Immune Positive Antibody Avoid ation, RBC (1%) response to blood group screen unnecessary antigens foreign antibody transfusions antigens on screening DAT Leukocyte- RBCs, WBCs, test or platelets reduced blood

(HLA)

Alloimmuniz 1:10 Immune Platelet Platelet Avoid ation, HLA (10%) response to refractorine antibody unnecessary antigens foreign ss, delayed screen transfusions antigens on hemolytic Lymphocytot Leukocyte- RBCs, WBCs, reaction, or platelets hemolytic oxicity test reduced blood

(HLA) disease of the newborn

Hemolytic 1:5000-1 : Anamnestic Fever, Antibody Identify antibody 11,000 immune decreasing screen response to hemoglobin, Transfuse compatible red red cell new positive DAT antigens antibody cells as needed

screening Tests for test, mild hemolysis jaundice (visual inspection for hemoglobine mia, LDH, bilirubin, urinary hemosiderin as clinically indicated)

Graft-versus- Rare Donor Erythroder Skin biopsy Corticosteroids, host disease lymphocytes ma, cytotoxic agents engraft in maculopapu HLA typing

recipient and lar rash, Irradiation of mount attack anorexia, blood components on host nausea, for patients at risk tissues vomiting, (including related diarrhea, donors and HLA- hepatitis, selected pancytopeni components)

a, fever

Post- Rare Recipient Thrombocyt Platelet antibody IGIV INCIDENC MANIFESTA DIAGNOSTIC THERAPEUTIC/PROP E ETIOLOGY TION TESTING HYLACTIC APPROACH

transfusion platelet openic screen and HPA-1 negative

purpura antibodies purpura, identification platelets (apparent bleeding 8- – alloantibody, 10 days Plasmapheresis

usually anti after HPA-1) transfusion destroy – autologous platelets

Immunomod Unknown Incompletely Increased None specific Avoid ulation understood renal graft unnecessary interaction of survival, transfusions donor WBC infection Autologous or plasma rate, factors with postresectio transfusion recipient n tumor Leukocyte- immune recurrence reduced red cells system rate and platelets (controversi al) Delayed (>24 Hours) Transfusion Reactions—Nonimmunologic

Iron Typically Multiple Diabetes, Serum

overload after >100 transfusions cirrhosis, ferritin RBC units with obligate cardiomyop Liver iron load in athy transfusion- enzymes dependent Endocrine patient function tests

ACE, angiotensin-converting enzyme; DAT, direct antiglobulin test; DIC, disseminated intravascular coagulation; HPA-1, human platelet antigen-1; IGIV, intravenous immunoglobulin; IM, intramuscular; IV, intravenous; LDH, lactate dehydrogenase; PO, per os; RBC, red blood cell; SC, subcutaneous; WBC, white blood cell.

Risk of Infection Associated With Blood Transfusion in the United States INFECTIOUS AGENT OR OUTCOME Viruses HIV-1 and -2 HTLV-I and -II HAV HBV INFECTIOUS AGENT OR OUTCOME HCV B19 parvovirus Bacteria RBCs Platelets (screened by Gram stain, pH, or glucose concentration) Platelets (screened by early aerobic culture) Parasites Babesiosis and malaria Trypanosoma cruzi

HAV, hepatitis A virus; HBC, hepatitis B virus; HCV, hepatitis C virus; HIV, human immunodeficiency virus; HTLV, human T-cell leukemia virus; RBC, red blood cell.

Group – B 2. Answer any of the following questions:

a) A 50 years old man presents with alternate constipation and diarrhea. He has a lump in left iliac fossa. How would you investigate and diagnose the case? Outline the treatment of such a case. 6+4+5 = 15

Answer. See the answer of question 2.a of Group – B of Supplementary Paper – I of 2012.

b) A 45 years old man presented with a recently discovered lump in the epigastrium with rapidly developing anorexia, asthenia, anemia and increasing vomiting. How would you investigate to arrive at the diagnosis? Outline the management of the case. 8+7 = 15

Answer. This is a case of Gastric outlet obstruction due to carcinoma stomach.

Differential diagnosis:

 Parietal swelling: o Epigastric hernia- A small round swelling exactly in the midline anywhere between the xiphisternum and umbilicus. In the first stage, it is sacless herniation of the extraperitoneal fat through a weak spot in the linea alba. There is no symptom at this stage. In the second stage, a pouch of peritoneum is drawn after it. In the last stage, a small tag of omentum gets into the sac and becomes adherent to it. At this stage the patient complains of dragging pain, discomfort or pain after food, not unlike those in peptic ulcer. o Lipoma.  Intra-abdominal swellings:- They occur in connection with the: o Liver and Subphrenic abscess o Hepatocellular carcinoma o Stomach and Duodenum  Congenital pyloric stenosis- Babies about 2 to 4 weeks old when present with projectile vomiting after meals, the diagnosis becomes obvious. On examination visible peristalsis of the stomach is always seen. Sometime a definite lump may be felt at the pylorus of the stomach.  Gastric outlet obstruction due to peptic ulcer  Carcinoma of the stomach o Transverse colon-  Intussusception: In intussusception there will be emptiness at the right iliac fossa. The patient complains of colicky pain, a lump in the epigastrium and "red current jelly" in the stool.  Diverticulitis: In inflammatory conditions a tender and irregular mass may be felt.  Hyperplastic tuberculosis  Neoplasms: In carcinoma the swelling is the presenting symptom. The swelling is irregular, hard and may be mobile above downwards and very slightly in the sideways or may be fixed. Anaemia, anorexia and occult blood in the stool are the features which helps in the diagnosis. Barium enema X-ray reveals constant 'filling defect' of the colon.  Omentum: In the tubercular peritonitis, the omentum is rolled up to form a transverse ridge in the epigastrium. Enlarged lymph nodes or adherent coils of intestine are also come across in this condition.  Pancreas: It hardly gives rise to a palpable swelling. The condition that forms lump in connection with this organ is the pseudocyst. True cyst of the pancreas is extremely rare. The pseudocyst is a collection of fluid in the lesser sac of the peritoneal cavity resulting from acute pancreatitis or trauma. It forms a smooth rounded swelling with fluctuation test positive. X- ray with barium meal will show the exact position of the swelling which is situated behind the stomach and is best seen in the lateral X-ray.  Abdominal aorta: Aneurysm of this part of the aorta is not uncommon. It presents a swelling in the epigastrium with characteristic expansile pulsation.  Lymph nodes: In addition to the usual causes of enlargement of lymph nodes, the followings are more important in this region: (i) tabes mesenterica, (ii) lymphosarcoma and (iii) secondary malignant growth from the neighbouring organs and also from the testis.  Retroperitoneal sarcoma and teratoma are the two conditions commonly seen in the posterior abdominal wall arising from the retroperitoneal tissue.

Investigations:

Laboratory investigations:

 Complete haemogram: In case of malignancy anaemia may be present.  Amylase and lipase: may be raised in pseudocyst pancreas  Tumour markers: AFP, Beta HCG, LDH for testicular cancer  Liver and kidney function tests

Imaging studies:

 Early USG and CECT abdomen: they can tell about origin, extent and number of abdominal lump. CECT can help in the diagnosis of pseudocyst pancreas, any retroperitoneal mass, diverticulitis and abdominal aortic aneurysm.  Barium enema: Used for colonic growth  CT and MRI: They can detect colonic mass, aortic aneurysms and lymphnode.  CT and MR Angiography: They can detect any vascular lesion.

Upper GI endoscopy: Used to detect lesions of stomach and duodenum.

See the answer of question 2.a of Group – B of Paper – I of 2014.

Management:

Investigations: The most confirmatory investigation is endoscopy of upper gastrointestinal tract.

Laboratory:

 Individuals with gastric outlet obstruction are often hypochloremic, hypokalemic, and alkalotic due to loss of hydrogen chloride and potassium. High urea and creatinine levels may also be observed if the patient is dehydrated.

Abdominal X-ray

 A gastric fluid level may be seen which would support the diagnosis.

Barium meal and follow through

 May show an enlarged stomach and pyloroduodenal stenosis.

Gastroscopy

 May help with cause and can be used therapeutically.

Treatment:

Prepare the patient after correction of electrolyte imbalance Preparation of the patient:  Rehydration with i/v isotonic saline with potassium supplementation. Replacing the sodium chloride and water allows the kidney to correct the acid base abnormality.  Following rehydration it may become obvious that the patient is also anaemic, the haemoglobin being spuriously high on presentation. –  The stomach should be emptied using a Wide-bore gastric tube. Pass an orogastric tube and lavage the stomach until it is completely emptied  Then endoscopy and contrast radiology  Biopsy of the area around the pylorus is essential to exclude malignancy  The patient should also have an anti-secretory agent, initially given intravenously to ensure absorption.  Build up nutrition.

Surgical principle:

 Surgery is the principal modality of treatment of gastric c.a. with curative intent  Stage of the disease, evidence of spread, age and build (general condition) all should be considered  Components of curative gastric cancer treatment consist following – . Extent of gastric resection . Extent of lymphadenectomy – . Splenectomy and Distal Pancreatectomy

Extent of Gastric Resection:  Extent of resection determined by obtaining a margin free of microscopic disease.  A line of resection at least 5-6 cm. from tumour mass necessary for low local recurrence.  In distal third cancer distal stomach with 2 cm of 1st part of duodenum removed and reconstruction by various methods.  In middle third cancer subtotal/total gastrectomy done. A minimum of 2 cm gastric remnant required for good reconstruction, so proximal margin of palpable tumour mass must be at least 7-8 cm from esophago-gastric junction for subtotal gastrectomy.  In proximal third cancer radical total gastrectomy with jejunal pouch esophageal anastomosis recommended.

Surgical resection:

• Resection of tumour • Grossly negative margin of at least 5 cms • Partial gastrectomy • Confirmed on frozen section • En block resection of adjacent involved organs

Extent of Gastrectomy:

 Radical subtotal gastrectomy (Distal tumour)  Total gastrectomy (Proximal tumour)

Early gastric cancer (T1 or 2, N0/1) Advanced gastric cancer (T3 or more or any of N2)

Suitable for attempted curative resection if the patient is Surgical intervention very unlikely to be curative. medically fit enough. May be undertaken for palliative treatment. Surgery is simple or radical gastrectomy. Local ablation for symptom control occasionally possible. Local resection or ablation has an uncertain place in Palliative chemotherapy occasionally effective for treatment. disseminated disease.

Extent of lymphadenectomy:

N1 – 3 to 6 N2 – 1, 2, 7, 8 & 11 N3 – 9, 10 & 12 . N1 nodes are w/in 3cm of the tumor . N2 along hepatic & splenic arteries . N3 more distant nodes. Adjuvant chemo and radiotherapy: For stages II and III adenocarcinoma

 Chemotherapy:

. 5-fluorouracil, leucovorin, cisplatin, doxorubicin and methotrexate

. Can not prolong survival in unresectable, metastatic or recurrent diseases

 Radiation (4500cGy):

. Effective in palliation for pain and bleeding

Group – C 3. Write short notes on (any three): 3x5 = 15 a) Hydatid cyst of liver. b) Pancreatic pseudocyst. c) Femoral hernia. d) Liver abscess. e) Marjolin’s ulcer. Answer. a) Hydatid cyst of liver.

Introduction: Hydatid disease in people is mainly caused by infection with the larval stage of the dog tapeworm Echinococcus granulosus. It is an important pathogenic, zoonotic and parasitic infection (acquired from animals) of humans, following ingestion of tapeworm eggs excreted in the faeces of infected dogs. Hydatid disease is a major endemic health problem in certain areas of the world. Pathology - Hydatid cyst structure A primary cyst in the liver is composed of three layers: 1. (pericyst): consisting of compressed liver parenchyma and fibrous tissue induced by the expanding parasitic cyst. 2. Laminated membrane (ectocyst): is elastic white covering, easily separable from the adventitia. 3. Germinal epithelium (endocyst) is a single layer of cells lining the inner aspects of the cyst and is the only living component, being responsible for the formation of the other layers as well as the hydatid fluid and brood capsules within– the cyst. In some primary cysts, laminated membranes may eventually disintegrate and the brood capsules are freed and grow into daughter cysts. Sometimes the germinal Epithelium daughter cysts, which if left untreated may cause recurrence.

Clinical features: o After infection with Echinococcus granulosus, humans are usually asymptomatic for a long time. The growth of the cyst in the liver is variable, ranging from 1 mm to 5 mm in diameter per year. o Most primary infections consist of a single cyst, but up to 20%-40% of infected people have multiple cysts. The symptoms depend not only on the size and number of cysts, but also on the mass effect within the organ and upon surrounding structures. Non complicated cysts: o Hydatid cyst of the liver is frequently silent and only diagnosed incidentally during abdominal investigation for other pathology. The clinical signs appear gradually with the increase volume of the cyst. o The most common symptom, when it occurs, is right upper quadrant or epigastric pain and the most common findings on examination are an enlarged liver and a palpable mass. o Pressure effects are initially vague. They may include non-specific pain, cough, low-grade fever, and the sensation of abdominal fullness. o As the mass grows, the symptoms become more specific because the mass impinges on or obstructs specific organs. Complicated cysts: o Patients may also present with complications of the cyst such as biliary communication, intraperitoneal rupture (spontaneous or post-traumatic) and, rarely, intrathoracic or intrapericardial rupture. o Cyst rupture can be associated with anaphylaxis secondary to the highly antigenic content of the cyst fluid or may be silent and present with multiple intraperitoneal cysts. o With secondary infection, tender hepatomegaly, chills, and spiking temperatures occurs. o Urticaria and erythema occur in cases of generalized anaphylactic reaction. o With biliary rupture the classic triad of jaundice, biliary colic and urticaria occurs. Diagnosis:  The diagnosis is most easily set by ultrasound or other imaging techniques such as CTscan or MRI, combined with case history. Serology tests such as ELISA or immunoblotting can be used in addition, being 80-100% sensitive for liver cysts but only 50-56% for lungs and other organs. False positive reactions may occur in persons with other tapeworm infections, cancer, or chronic immune disorders.  A great part of the patients treated for hydatid disease get their diagnosis incidentally, seeking medical care for other reasons. Investigations  The definitive diagnosis of liver echinococcosis requires a combination of imaging, serologic, and immunologic studies.  Routine laboratory tests are rarely abnormal occasionally eosinophilia may be present in the presence of cyst leakage, or may be normal. Serum alkaline phosphatase levels are raised in one third of patients. Serology and immunological tests:  Serological tests detect specific antibodies to the parasite and are the most commonly employed tools to diagnose past and recent infection with E. granulosus. Detection of IgG antibodies implies exposure to the parasite, while in active infection high titers of specific IgM and IgA antibodies are observed.  Detection of circulating hydatid antigen in the serum is of use in monitoring after surgery and pharmacotherapy and in prognosis.  Tests of humoral immunity are still widely used to confirm the diagnosis.  Indirect immunofluorescence assay (IFA) is the most sensitive test (95%) in patients with hepatic CHD. Imaging techniques:  Imaging modalities range from simple to complex and invasive. Ultrasonography (US) is the screening method of choice.  CT scan is an important preoperative diagnostic tool to determine vascular, biliary or extra hepatic extension, to recognize complications, such as rupture and infections, and therefore to assess respectability.  Magnetic resonance Imaging (MRI scan) - MRI delineates the cyst capsule better than CT scan, as a low intensity on both T1 and T2 weighted images. However, CT scan is better in demonstrating mural calcifications, cysts less than 3 cm may not show any specific features and small peritoneal cysts may be missed.  Endoscopic retrograde cholangiopancreatography (ERCP) remains an important tool in cases where a rupture into the biliary tree has occurred, allowing both the diagnosis of major biliary communication and clearance of the common bile duct (CBD) prior to surgery or intervention by the means of sphincterotomy.  Direct cholangiography: intra-operative cholangiography is performed through a cystic drain or a T-Tube in a suspected intrabiliary rupture and bile duct obstruction. This method is used to detect post-operative complications following surgery.  However, diagnostic tests such as CT and MRI are mandatory in liver hydatidosis because they allow thorough knowledge regarding lesion size, location, and relations to intrahepatic vascular and biliary structures, providing useful information for effective treatment and decrease in post- operative morbidity. Plain Radiographs: Plain radiographs of the abdomen and chest may reveal a thin rim of calcification delineating a cyst, or an elevated hemi diaphragm.

Doppler ultrasonography is indicated to show the reports of hydatid cyst with vascular axes (portal vein, hepatic veins, and inferior vena cava). However, in the types I and IV, we have to consider differential diagnosis.

Gharbi Classification on Ultrasonography features of Hydatid Cyst.

Type V cysts determined by ultrasound to be calcified and have been assumed to be dead cysts and do not require surgery. Intra-operative Ultrasonography is an important investigation during surgery for hydatid cyst of the liver.

 WHO introduced a standardized classification of Ultrasonography images of cystic echinococcosis, to obtain comparable results in patients worldwide and to link disease status with each morphological type of Hydatid cysts.

 CL o unilocular anechoic cystic lesion without any internal echoes and septations  CE 1 o uniformly anechoic cyst with fine echoes settled in it representing hydatid sand  CE 2 o cyst with multiple septations giving it multivesicular appearance or rosette appearance or honeycomb appearance with unilocular mother cyst o this stage is the active stage of the cyst  CE 3 o unilocular cyst with daughter cysts with detached laminated membranes appearing as water lily sign o this is the transitional stage of the cyst  CE 4 o mixed hypo- and hyperechoic contents with absent daughter cysts; these contents give an appearance of a ball of wool (ball of wool sign) indicating the degenerative nature of the cyst  CE 5 o arch-like thick partially or completely calcified wall o this stage of cyst is inactive and infertile

Treatment: General considerations:  Surgery remains the gold standard treatment for hydatid liver disease. The aim of surgical intervention is to inactivate the parasite, to evacuate the cyst along with resection of the germinal layer, to prevent peritoneal spillage of scolices and to obliterate the residual cavity. It can be performed successfully in up to 90% of patients if a cyst does not have a risky localisation.  The choice of an optimal treatment should be carefully assessed in each case.  The principles of hydatid surgery are o Total removal of all infective components of the cysts; o The avoidance of spillage of cyst contents at time of surgery; o Management of communication between cyst and adjacent structures; o Management of the residual cavity; o Minimize risks of operation. All the surgical procedures can be divided into two large groups, a conservative group and a radical one. The conservative technique communication between cyst and adjacent structures; The conservative technique: Conservative procedures are safe and technically simple, and are useful in the management of uncomplicated hydatid cysts. Marsupialization was the most common used procedure because it is quick and safe. However, their main disadvantage is the high frequency of postoperative complications, the most common being bile leak from a cyst-biliary communication, bilomas and bile peritonitis. Radical surgical procedures: Radical surgical procedures include cystectomy, pericystectomy, lobectomy and hepatectomy. Radical procedures have lower rate of complications and recurrences but many authors consider them inappropriate, claiming that intraoperative risks are too high for a benign disease. Laparoscopic management of hydatid cysts The rapid development of laparoscopic techniques has encouraged surgeons to replicate principles of conventional hydatid surgery using a minimally invasive approach Laparoscopic procedure has some advantages compared to open surgery. In fact this approach to liver hydatid cyst offers a lower morbidity outcome and a shorter hospital stay and it is also associated with a faster surgery. In addition to that advantage, Laparoscopic procedure gives a better visual control of the cyst cavity under magnification which allows a better detection of biliary fistula. This approach is possible only in selected cases. The Criteria to exclude laparoscopic treatment of hydatid cyst of liver are o Rupture of the cyst in biliary tract o Central localization of the cyst o Cysts dimension > 15 cm o Number of cysts > 3 o Thickened or calcified walls o Opening of bile ducts that leak bile Percutaneous treatment of hydatid cyst PAIR (puncture, aspiration, injection, and reaspiration) is a percutaneous treatment technique for hydatid disease. The World Health Organization currently supports PAIR as an effective alternative to surgery, although its use is limited. The World Health Organization guidelines for indications and contraindications of PAIR are as follows: 1. Indications for PAIR o Nonechoic lesion greater than or equal to 5 cm in diameter o Cysts with daughter cysts and/or with membrane detachment o Multiple cysts if accessible to puncture o Infected cysts o Patients who refuse surgery. o Patients who relapse after surgery. o Patients in whom surgery is contraindicated Patients who fail to respond to chemotherapy alone o Children over 3 years. o Pregnant women 2. Contraindications for PAIR o Non cooperative patients o Inaccessible or risky location of the liver cyst o Cyst in spine, brain, and/or heart o Inactive or calcified lesion o Cyst communicating with the biliary tree Patients should be followed clinically after PAIR treatment. Recurrence is increased in more complicated cysts, including those with multiple daughter cysts. Endoscopic management of hydatid cyst: The ERCP is effective in diagnosing biliary tree involvement from the cyst. The Endoscopic management is useful in presence of intrabiliary rupture, which requires exploration and drainage of the biliary tract and also after surgery in presence of residual hydatid material (membranes and daughter cyst) left in biliary tree. During the endoscopic exploration the biliary tree is cleared of any hydatid material with a balloon catheter or a dormia basket. Chemotherapy for hydatid disease of liver: Medical treatment of hydatid liver cysts, is based on benzoimidazole carbamates, such as mebendazole and albendazole. Albendazole seems to be more effective owing to better penetration and absorption. b) Pseudocyst pancreas: A pseudocyst is defined as a peripancreatic fluid collection contained by a wall of fibrous granulation tissue that does not have an epithelial lining. This is in contrast to cystic neoplasms of the pancreas, which are characterised by an epithelial lining.

From history and clinical examination:

Symptoms: Signs:

o Abdominal pain > 3 weeks (80 – 90%) o Tenderness o Nausea / vomiting o Abdominal fullness o Early satiety o Bloating, indigestion

 Clinically suspect a pseudocyst o Episode of pancreatitis fails to resolve o Amylase levels persistantly high o Persistant abdominal pain o Epigastric mass palpated after pancreatitis

Investigation:

USG:

It will be able to tell about the organ involved

Upper GI endoscopy:

It will rule out any gastric neoplasm or peptic ulcer disease.

Serum amylase:

Serum amylase level will be persistently high.

Lipase:

Serum lipase level may be high also.

Management after diagnosis:

 Computerized tomography: This is the gold standard for initial assessment and follow-up Large Pancreatic Pseudocyst  Magnetic resonance cholangiopancreatography (MRCP): to establish the relationship of the pseudocyst to the pancreatic ducts.

 50% resolve spontaneously

Treatment: The principal indications for treating pancreatic pseudocysts are to relieve symptoms and to prevent complications. In the absence of symptoms or evidence of enlargement, expectant management usually is reasonable. An enlarging asymptomatic pseudocyst that has been present for 6 weeks usually is treated. This relatively conservative approach is based on the low risk of complications. A natural-history study from India indicates that asymptomatic pseudocysts less than 7.5 cm in diameter and without internal debris will resolve spontaneously at an average of 5 months. Complications:  Infection  S/S – Fever, worsening abd pain, systemic signs of sepsis  CT – Thickening of fibrous wall or air within the cavity  GI obstruction  Perforation  Hemorrhage  Thrombosis – SV (most common)  Pseudoaneurysm formation – Splenic artery (most common), GDA, PDA

Indications for drainage Intervention

 Presence of symptoms (> 6 wks)  Percutaneous drainage  Enlargement of pseudocyst ( > 6 cm)  Endoscopic drainage  Complications  Surgical drainage  Suspicion of malignancy

Percutaneous drainage:

 Continuous drainage until output < 50 ml/day + amylase activity ↓  Failure rate 16%  Recurrence rates 7%  Complications  Conversion into an infected pseudocyst (10%)  Catheter-site cellulitis  Damage to adjacent organs  Pancreatico-cutaneous fistula  GI hemorrhage

Endoscopic drainage:  Transenteric drainage

 Cystogastrostomy

 Cystoduodenostomy

 Transpapillary drainage

 40-70% of pseudocysts communicate with pancreatic duct

 ERCP with sphincterotomy, balloon dilatation of pancreatic duct strictures, and stent placement beyond strictures

Surgical options:

 Excision

 Tail of gland & along with proximal strictures – distal pancreatectomy & splenectomy

 Head of gland with strictures of pancreatic or bile ducts – pancreaticoduodenectomy

 External drainage  Internal drainage

 Cystogastrostomy

 Cystojejunostomy

 Permanent resolution confirmed in between 91%–97% of patients  Cystoduodenostomy

 Can be complicated by duodenal fistula and bleeding at anastomotic site c) Femoral hernia.

Femoral canal: The major feature of the femoral canal is the femoral sheath. This sheath is a condensation of the deep fascia (fascia lata) of the thigh and contains, from lateral to medial, the femoral artery, femoral vein, and femoral canal. The femoral canal is a space medial to the vein that allows for venous expansion and contains a lymph node (node of Cloquet). Other features of the femoral triangle include the femoral nerve, which lies lateral to the sheath.

Wall of the Femoral canal:

 Anterior is the inguinal ligament  Posterior is the iliopsoas, pectineal, and long adductor muscles (floor).  Medial is lacunar ligament  Lateral is femoral vessle

Femoral Hernia: Hernia medial to femoral vessels under inguinal ligament

Small femoral hernia may be unnoticed by the patient or disregarded for years perhaps until the day it strangulates. Adherence of the greater omentum sometimes causes a dragging pain. Rarely a large sac is present.

 Age ; uncommon in children , most common in old age female .

 Sex; women > men (but still commonest hernia in women the inguinal hernia )

 The patient came with local symptoms

1- Discomfort and pain

2- Swelling in the groin

 General ; femoral hernia is more likely to be strangulated than the inguinal hernia

 Multiplicity ; often bilateral

Femoral hernia versus inguinal hernia:

Inguinal hernia Femoral hernia

1- more common in male 1- more common in females

2- pass through the inguinal canal 2- pass through the femoral canal

3- neck of the sac is above and medial the pubic 3- neck of the sac is below and lateral the tubercle pubic tubercle

4- less common to be strangulated 4- more common to be strangulated

5- can be treated without surgery 5- must be treated surgically

6- the two diagnostic signs of hernia + 6- the two diagnostic signs of hernia - 7- the sac mainly contain ; bowel 7- the sac mainly contains ; omentum

Diagnosis:

 The diagnosis is largely a clinical one, generally done by physical examination of the groin. However, in obese patients, imaging in the form of ultrasonography, CT or MRI may aid in the diagnosis. An abdominal x-ray showing small bowel obstruction in a female patient with a painful groin lump needs no further investigation.  Several other conditions have a similar presentation and must be considered when forming the diagnosis: inguinal hernia, an enlarged inguinal lymph node, aneurysm of the femoral artery, saphena varix, and an abscess of the psoas.

Treatment:

Surgery:

 Some surgeons choose to perform "key-hole" or laparoscopic surgery (also called minimally invasive surgery) rather than conventional "open" surgery. With minimally invasive surgery, one or more small incisions are made that allow the surgeon to use a surgical camera and small tools to repair the hernia.  Either open or minimally invasive surgery may be performed under general or regional anaesthesia, depending on the extent of the intervention needed. Three approaches have been described for open surgery.  Lockwood s infra-inguinal approach  Lotheissen s trans-inguinal approach  McEvedy s’ high approach  The infra-inguinal’ approach is the preferred method for elective repair. The trans-inguinal approach ’involves dissecting through the inguinal canal and carries the risk of weakening the inguinal canal. McEvedy s approach is preferred in the emergency setting when strangulation is suspected. This allows better access to and visualisation of bowel for possible resection. In any approach, care should’ be taken to avoid injury to the urinary bladder which is often a part of the medial part of the hernial sac.  Repair is either performed by suturing the inguinal ligament to the pectineal ligament using strong non-absorbable sutures or by placing a mesh plug in the femoral ring. With either technique care should be taken to avoid any pressure on the femoral vein.

d) Liver abscess.

Introduction: A liver abscess is a pus-filled mass inside the liver. Common causes are abdominal infections such as appendicitis or diverticulitis due to haematogenous spread through the portal vein.

Types: There are three major forms of liver abscess, classified by etiology:

 Pyogenic liver abscess, which is most often polymicrobial, accounts for 80% of hepatic abscess cases in the United States.  Amoebic liver abscess due to Entamoeba histolytica accounts for 10% of cases.  Fungal abscess, most often due to Candida species, accounts for less than 10% of cases.

Causes: Major bacterial causes of liver abscess include the following:

 Streptococcus species (including Enterococcus)  Escherichia species  Staphylococcus species  Klebsiella species (Higher rates in the Far East)  Anaerobes (including Bacteroides species)  Pseudomonas species  Proteus species  Many cases are polymicrobial.

For amoebic liver abscess see the answer of 3.e of Group – C of Paper – I of 2014.

Pyogenic liver abscess:

 Signs and symptoms: Acute abscess Chronic abscess

 Fever  Fever  Lethargy  Abdominal discomfort  Discomfort in Right Upper Quadrant of Abdomen  Enlarged liver  Anorexia  Enlarged and Tender Liver  Pleural effusion

Cause:

 Biliary disease (most common)

E.g.: stones, cholangiocarcinoma

 Colonic disease

E.g.: diverticulitis, appendicitis, Crohn's disease

 Cryptogenic disease  Pancreatitis  Infection of blood  Intra-abdominal sepsis  Infection of biliary system  Traumatic introduction

E.g.: penetrating injury, iatrogenic (radiofrequency ablation)

Common bacterial cause:  Streptococcus milleri  E. coli  Streptococcus fecalis  Klebsiella pneumoniae  Proteus vulgaris  Bacteroides  Opportunistic Pathogens (Staphylococcus)

Diagnosis:

 Blood CP (No Leucocytosis)  Haemoglobin estimation (anaemia)  Serum albumin levels (Falls rapidly)  USG and CT Scanning

Treatment:

 Antibiotics: . Penicillins . Aminoglycosides . Metronidazole . Cephalosporins  Percutaneous drainage under USG or CT control  Laparotomy in intraabdominal disease d) Marjolin’s ulcer. See the answer of 4.a of Group – D of Paper – II of 2013. Answer. Introduction: Marjolin's ulcer refers to an aggressive ulcerating squamous cell carcinoma presenting in an area of previously traumatized, chronically inflamed, or scarred skin. They are commonly present in the context of chronic wounds including burn injuries, venous ulcers, ulcers from osteomyelitis, and post radiotherapy scars. Appearance: Slow growth, painlessness (as the ulcer is usually not associated with nerve tissue), and absence of lymphatic spread due to local destruction of lymphatic channels. Characteristic: Histologically, the tumour is a well-differentiated squamous cell carcinoma. This carcinoma is aggressive in nature, spreads locally and is associated with a poor prognosis. 40% occur on the lower limb and the malignant change is usually painless. This malignant change of the wound happens a long time after initial trauma, usually 10 25 years later. Its edge is everted and not always raised. Diagnosis: Wedge biopsy is the favored method of diagnosis. Tissue– specimens obtained should be taken from both the centre and margin of lesion, as the central ulcerated deposits may be necrotic. Treatment: Treatment is usually surgical, with a wide excision of the lesion; typically a 1 cm margin all around is required. Radiation therapy is also a good alternative in most cases. Group – D 4. Write briefly on (any three): 3x5 = 15 a) Carpal tunnel syndrome. b) Tardy ulnar nerve palsy. c) Supracondylar fracture of humerus. d) Giant cell tumour. e) Trendelenburg test for hip joint.

Answer.

Definition: Carpal tunnel syndrome, the most common focal peripheral neuropathy, results from compression of the median nerve at the wrist.

Epidemiology:

 Affects an estimated 3 percent of adult Americans

 Three times more common in women than in men

 High prevalence rates have been reported in persons who perform certain repetitive wrist motions (frequent computer users)

• 30% hand paresthesias

• 10% clinical criteria for carpal tunnel syndrome

• 3.5% abnormal nerve conduction studies

Clinical features:

 Pain

 Numbness

 Tingling

 Symptoms are usually worse at night and can awaken patients from sleep.

 To relieve the symptoms, patients often flick their wrist as if shaking down a thermometer (flick sign). “ ”

Diagnosis:

 History

 Physical examination

 Nerve Conduction Study

Differential Diagnostics:  Tendonitis

 Tenosynovitis

 Diabetic neuropathy

 Kienbock's disease

 Compression of the Median nerve at the elbow

Treatment:

 Conservative treatments

– General measures:  Avoid repetitive wrist and hand motions that may exacerbate symptoms or make symptom relief difficult to achieve.  Not use vibratory tools.  Ergonomic measures to relieve symptoms depending on the motion that needs to be minimized.

– Wrist splints

– Oral medications: Orally administered corticosteroids Prednisolone,20 mg per day for two weeks,followed by 10 mg per day for two weeks.

– Local injection:  A mixture of 10 to 20 mg of lidocaine (Xylocaine) without epinephrine and 20 to 40 mg of methylprednisolone acetate (Depo-Medrol) or similar corticosteroid preparation is injected with a 25-gauge needle at the distal wrist crease (or 1 cm proximal to it).  Splinting is generally recommended after local corticosteroid injection.  If the first injection is successful, a repeat injection can be considered after a few months.  Surgery should be considered if a patient needs more than two injections.

– Ultrasound therapy

– Predicting the outcome of conservative treatment  Surgery:  Should be considered in patients with symptoms that do not respond to conservative measures and in patients with severe nerve entrapment as evidenced by nerve conduction studies,thenar atrophy, or motor weakness.  It is important to note that surgery may be effective even if a patient has normal nerve conduction studies.

Complications of surgery:

o Injury to the palmar cutaneous or recurrent motor branch of the median nerve o Hypertrophic scarring o Laceration of the superficial palmar arch o Tendon adhesion o Postoperative infection o Hematoma o Arterial injury o Stiffness b) Tardy ulnar nerve palsy. Introduction: It is an abnormal condition characterized by atrophy of the first dorsal interosseous muscle and difficulty in performing fine manipulations. Causes: o It may be caused by injury of the ulnar nerve at the elbow and commonly affects individuals with a shallow ulnar groove or those who persistently rest their weight on their elbows. o Elbow fractures with resultant cubitus varus. Signs and symptoms: o Signs and symptoms of this disorder may include numbness of the small finger, of the contiguous half of the proximal and middle phalanges of the ring finger, and of the ulnar border of the hand. Treatment: o Treatment concentrates on the prevention of further injury to the ulnar nerve. o Therapy may include the use of a doughnut cushion for the elbow to relieve the pressure on the ulnar nerve. o Severe cases of this disorder may be corrected by surgical procedures that mobilize and transplant the nerve to a site in front of the medial epicondyle. c) Supracondylar fracture of humerus. Distal Humerus Anatomy

 Medial epicondyle proximal to trochlea –

 Lateral epicondyle proximal to capitellum –

 Radial fossa – accommodates margin of radial head during flexion

 Coronoid fossa – accepts coronoid process of ulna during flexion

Introduction:

 It is # which involves the lower end of the humerus usually involving the thin portion of the humerus through o Olecranon fossa or o Just above the fossa or o Metaphysis  Most common elbow injuries in children.  Makes up approximately 60% of elbow injuries.  Becomes uncommon as the age increases.

General consideration:

 Incidence of supracondylar #:

 Age: peak age: 5-7 yrs  Average age : 6.7 yrs  Sex: Boys = Girls (Latest Trends)  Side: Left > Righ ( Non dominant > dominant )  Nerve injurie:7% - Median> Radial > Ulnar  Vascular injuries:1%  Cause of # o Fall from height 70% ----- children > 3 yrs o Fall from bed children < 3 yrs o Non accidental injury (Child abuse) children < 15 months  Associated #s:

Distal radius > Scaphoid > Proximal humerus > Monteggia

 Clinical types: o Extension type: 98% o Flexion type : 2% o Open injuries : < 1%

Mechanism of injury:

 For Extension type of SC # humerus

Fall on outstretched hand Elbow hyper extended Fore arm pronated or supinated

 For Flexion– type of SC # humerus Fall directly on the elbow rather than out stretched hand

Radiography:

 Radiographic views:

Antero posterior Lateral

Oblique

Axial ( jones view )

Clinical features:

 In most cases, children will not move the elbow if a fracture is present, although this may not be the case for non-displaced fractures.

 Swelling about elbow is a constant feature, develop within first few hrs.  S shaped deformity

 Distal humeral tenderness

 Anterior plucker sign +ve

 Nerve injury incidence is high, between 7 and 16 % (median, radial and ulnar nerve)

 Anterior interosseous nerve is most commonly injured nerve

 In many cases, assessment of nerve integrity is limited , because children can not always cooperate with the exam

 Carefully document pre manipulation exam, as post manipulation neurologic deficits can alter decision making

 Thorough documentation of all findings is important. A simple record of neurovascular status is intact is unacceptable. “  Individual assessment” and recording of motor, sensory, and vascular function is essential

 Always palpate the arm and forearm for signs of compartment syndrome.

Treatment:

General principles:

 Splinting elbow in comfortable position -20-30degrees of flexion of elbow.  Careful physical examination & X-ray evaluation.  Tight bandaging/ excessive flexion or excessive extension should be avoided.  Associated life threatening complications (if any) to be attended first.

Type Treatment I  Simple immobilization with a post slab in 90deg. With a cuff and collar  Xray to be rpted at 5-7 days to document for any displacement  Slab kept for 3 weeks

II  Treatment – closed reduction under anaesthesia  Traction is applied followed by correction of rotational deformity  Extension deformity is corrected with pressure by thumb over the olecranon

III  Closed reduction & percutaneous K wire fixation  Open reduction & K wire fixation

Complications:

Neurovascular complications Tear or entrapment of the brachial artery; (b) spasm of the artery and (c) compression of the artery relieved by manipulation of the fracture (d) compression of ulnar nerve. Causing Pink and Pulseless hand in supracondylar fracture. Thus there is loss of circulation of forearm, causing lack of reperfusion of tissues resulting in tissue death causing compartment syndrome. Therefore the complications of elbow dislocations include the following:

 Posttraumatic periarticular calcification.  Myositis ossificans or calcific tendinitis  Neurovascular injuries (8-21% of cases) palsy to the anterior interosseus nerve at time of index injury is most common, followed by brachial artery injuries (5-13%). Injury to the ulnar nerve is reported with percutaneous pinning— through the medial epicondyle.  Osteochondral defects, intra-articular loose bodies, and avascular necrosis of the capitulum  Instability

Other injuries:

 Most commonly brachial artery injury, and if left untreated could lead to Volkmann's contracture (permanent flexion contracture of the hand at the wrist, resulting in a claw-like deformity of the hand and fingers).

 Cubitus varus: Also known as gunstock deformity. This is more of a cosmetic problem, and does not affect the range of motion of the elbow joint.

 Malunion: The condyles of the distal humerus do not correctly join together. It may be misaligned anteriorly, posteriorly, medially or laterally in respect to the shaft of the humerus. A thicker ossification will also appear on the distal end. Remove it one year later to prevent additional ossification from forming.

d) Introduction:

Giant cell tumours (GCTs) of bone, also known as osteoclastomas, are relatively common bone tumours, usually benign which are arising from metaphysis and typically extending into the epiphysis of the long bones.

Epidemiology  GCTs are common, comprising 18-23% of benign bone neoplasms and 4-9.5% of all primary bone neoplasms.

 They almost invariably (97-99%) occur when the growth plate has closed and are therefore typically seen in early adulthood

 80% of cases reported between the ages of 20 and 50, with a peak incidence between 20 and 30.

 There is overall a mild female predilection, especially when located in the spine, however malignant transformation is far more common in men (M:F of ~3:1). Clinical presentation Presentation is not specific, typically presents insidiously and relates to bone pain, soft tissue mass, compression of adjacent structures or acutely with a pathological fracture.

Pathology Giant cell tumours are believed to result from an over-expression in RANK/RANKL signalling pathway with resultant over-proliferation of osteoclasts.

These tumours contain numerous thin walled vascular channels predisposing to areas of haemorrhage and presumably related to the relatively frequent co-existence of aneurysmal bone cysts (ABC) found in 14% of cases.

Macroscopically, giant cell tumours are variable in appearance, depending on amount of haemorrhage, presence of co-existent ABC, and degree of fibrosis.

Microscopically they are characterised by prominent and diffuse osteoclastic giant cells and mononuclear cells ( round, oval, or polygonal and may resemble normal histiocytes).GCTs are low grade tumours even in radiologically aggressive appearing lesions.

Approximately 5-10% of GCTs are malignant. Sarcomatous transformation is seen, especially in radiotherapy treated inoperable tumours.

Although rare (3%) lung metastasis are possible.

Location They typically occur as single lesions. Although any bone can be affected, the most common sites are:

o Around the knee: distal femur and proximal tibia: 50-65% o Distal radius: 10-12% o Sacrum: 4-9% o Vertebral body Paget disease)

MultipleRadiographic locations: features ≈ 1% (multiple lesions usually occur in association with Classic appearance Four radiographic features are characteristic when a GCT is located in a long bone:

o Occurs only with a closed growth plate o Abuts articular surface: 84-99% come within 1 cm of the articular surface o Well defined with non-sclerotic margin (though < 5% may show some sclerosis) o Eccentric: if large this may be difficult to assess Plain film and CT General radiographic features include:

 Narrow zone of transition: a broader zone of transition is seen in more aggressive gcts  No surrounding sclerosis: 80-85%  Overlying cortex is thinned, expanded or deficient  Periosteal reaction is only seen in 10-30% of cases  Soft tissue mass is not infrequent  Pathological fracture may be present  No matrix calcification/mineralization MRI: can differentiate from aneurismal bone cyst Scintigraphy ( bone scan): Most GCT demonstrate increased uptake on delayed images, especially around the periphery, with a central photopenic region (doughnut sign). Increased blood pool activity is also seen, and can be seen in adjacent bones due to generalised regional hyperaemia (contiguous bone activity).

Angiography: If performed, usually in the setting of pre-operative embolisation, angiography usually demonstrates a hypervascular tumour (2/3rd of cases) with the rest being hypo or avascular. Treatment and prognosis  Classically, treatment is with curettage and packing with bone chips or polymethylmethacrylate (PMMA

 Local recurrence is from the periphery of the lesion and has historically occurred in up to 40-60% of cases.

 Newer intraoperative adjuncts such as thermal or chemical treatment of the resection margins have lowered the recurrence rate to 2.5-10%.

 Early work on monoclonal antibodies, as an adjuvent treatment, has been impressive in causing high rates of tumour necrosis.

 Wide local excision is associated with a lower recurrence rate, but has greater morbidity.

Differential diagnosis: There is a relatively wide differential similar to that of a lytic bony lesion:  Chondroblastoma  Multiloculated aneurysmal bone cyst (abc): younger age group, but may co-exist with GCT; fluid-fluid levels  Non-ossifying fibroma: usually younger age group.  Brown tumour: in the setting of hyperparathyroidism  Enchondroma: only really a consideration in lesions of small bones of the hand and foot 3  Chondrosarcoma: typically older age group  Metastases and multiple myeloma e) Trendelenburg test for hip joint. The Trendelenburg test is used to assess hip stability.

o The patient is asked to stand unassisted on each leg in turn, whilst the examiner's fingers are placed on the anterior superior iliac spines. The foot on the contralateral side is elevated from the floor by bending at the knee. An alternative approach is to have the patient undertake this manoeuvre facing the examiner and supported only by the index fingers of the outstretched hands; this accentuates any instability of balance shown during a positive test. o In normal function, the hip is held stable by gluteus medius acting as an abductor in the supporting leg. If the pelvis drops on the unsupported side - positive Trendelenburg sign - the hip on which the patient is standing is painful or has a weak or mechanically-disadvantaged gluteus medius.

A positive Trendelenburg test is found in:

 Any condition that brings the origin and insertion of gluteus medius together: o Subluxation or dislocation of the hip o Coxa vara o Greater trochanter fractures o Slipped upper femoral epiphysis  Abductor paralysis or weakness e.g.: o Polio o Root lesion o Post-operative nerve damage o Muscle-wasting disease  Any painful hip disorder which results in gluteal inhibition

Caution must be exercised as false positives have been noted in 10% of cases.

The West Bengal University of Health Sciences

M.B.B.S. 3rd Professional Part – II Examination, 2015

1 Subject: Surgery Time: 2 /2 hrs. Paper: I Marks: 60

Group – A 1) Describe the biological process of wound healing. What are the factors affecting wound healing? Treatment options for presternal keloid. 5+5+5 = 15 Answer. Biological process of wound healing: Wound healing is an intricate process where the skin or other body tissue repairs itself after injury. In normal skin, the epidermis (surface layer) and dermis (deeper layer) form a protective barrier against the external environment. When the barrier is broken, an orchestrated cascade of biochemical events is quickly set into motion to repair the damage. This process is divided into predictable phases: blood clotting (hemostasis), inflammation, the growth of new tissue (proliferation), and the remodeling of tissue (maturation). Sometimes blood clotting is considered to be part of the inflammation stage instead of its own stage.

 Hemostasis (blood clotting): Within the first few minutes of injury, platelets in the blood begin to stick to the injured site. This activates the platelets, causing a few things to happen. They change into an amorphous shape, more suitable for clotting, and they release chemical signals to promote clotting. This results in the activation of fibrin, which forms a mesh and acts as "glue" to bind platelets to each other. This makes a clot that serves to plug the break in the blood vessel, slowing/preventing further bleeding.  Inflammation: During this phase, damaged and dead cells are cleared out, along with bacteria and other pathogens or debris. This happens through the process ofphagocytosis, where white blood cells "eat" debris by engulfing it. Platelet-derived growth factors are released into the wound that cause the migration and division of cells during the proliferative phase.  Proliferation (growth of new tissue):

In this phase, angiogenesis, collagen deposition, granulation tissue formation, epithelialization, and wound contraction occur. In angiogenesis, vascular endothelial cells form new blood vessels. In fibroplasia and granulation tissue formation, fibroblasts grow and form a new, provisional extracellular matrix (ECM) by excreting collagen and fibronectin. Concurrently, re-epithelialization of the epidermis occurs, in which epithelial cells proliferate and 'crawl' atop the wound bed, providing cover for the new tissue.

In wound contraction, myofibroblasts decrease the size of the wound by gripping the wound edges and contracting using a mechanism that resembles that in smooth muscle cells. When the cells' roles are close to complete, unneeded cells undergo apoptosis.  Maturation (remodeling): During maturation and remodeling, collagen is realigned along tension lines, and cells that are no longer needed are removed by programmed cell death, or apoptosis.

See the Question 1 of Group – A of Paper – I of 2011. Treatment options for presternal keloid.

 Intralesional corticosteroids: This is the most frequently used modality, the steroid most commonly used, being depot preparation of triamcinolone acetonide. Postoperative intralesional triamcinolone after surgical excision seems to prevent recurrence  5-Fluorouracil (5-FU) intralesional injections: This treatment is increasingly becoming popular.  Bleomycin: Intralesional injection of Bleomycin appears to be an effective therapy in the treatment of keloids  Interferon a-2b: Intralesional injection of combination of interferon α-2b with triamcinolone has been reported to be superior to triamcinolone alone in reducing the depth and volume of keloids  Imiquimod: Imiquimod 5% cream is a novel immune modulator with localized therapeutic effects at the drug application site, capable of enhancing local production of immune- stimulating cytokines such as interferons, tumor necrosis factor, and interleukins.  Pressure therapy:  Radiotherapy: Radiotherapy has been used as a monotherapy or as an adjuvant to surgical excision. A combination of surgery followed 24 hours later by radiotherapy is thought to be the most effective approach for the management of extensive HTS and keloids  Lasers: Carbon dioxide laser monotherapy has a recurrence rate as high as 90% and is therefore not recommended  Surgical excision: Surgical excision alone is associated with recurrence in 50-100% of patients; however, an exception is earlobe keloid which recurs much less frequently  Cryosurgery: Cryosurgery with liquid nitrogen leads to total or partial success in almost two-third to three-fourth of keloids after at least three sessions Group – B 2. Answer any of the following questions: a) A 45 years old gentleman presents with intractable anaemia and a painless lump in right iliac fossa of 3 months duration. How would you investigate the case to confirm the diagnosis? Briefly outline a comprehensive management of the problem. 8+7 = 15

Answer. See the Question 2.b of Group – B of Supplementary Paper – I of 2011. b) A 50 years old male comes to you with painless progressive jaundice and on clinical examination the gall bladder is palpable. How will you investigate the patient to come to a diagnosis? Describe the preoperative preparations of jaundiced patients. 10+5 = 15

Answer. See the Question 2.b of Group – B of Paper – I of 2010. See the Question 3.a of Group – C of Paper – I of 2011. Group – C 3. Write short notes on (any three): 3x5 = 15 a) Ludwig’s angina. b) Oestrogen and Progesterone receptors. c) Types of Anorectal abscess. d) Appendicular lump. e) MODS. Answer. a) Ludwig’s angina. Introduction: Ludwig's angina, otherwise known as angina ludovici, is a serious, potentially life-threatening cellulitis, or connective tissue infection, of the floor of the mouth, usually occurring in adults with concomitant dental infections and if left untreated, may obstruct the airways, necessitating tracheotomy. The life-threatening nature of this condition generally necessitates surgical management with involvement of critical care physicians such as those found in an intensive care unit. Causes: Dental infections account for approximately 80% of cases of Ludwig's angina. Mixed infections, due to both aerobes and anaerobes, are of the cellulitis associated with Ludwig's angina. Typically, these include alpha-hemolytic streptococci, staphylococci and bacteroides groups. The route of infection in most cases is from infected lower molars or from , which is an infection of the surrounding the partially erupted lower (usually third) molars. Although the widespread involvement seen in Ludwig's usually develops in immunocompromised persons, it can also develop in otherwise healthy individuals. Thus, it is very important to obtain dental consultation for lower-third molars at the first sign of any pain, bleeding from the gums, sensitivity to heat/cold or swelling at the angle of the jaw. Signs and symptoms: Ludwig's angina is a fascial space infection with bilateral involvement of the submandibular, sublingual and submental spaces. The external signs may include bilateral lower facial edema around the and upper neck. Intraoral signs may include a raised floor of mouth due to sublingual space involvement and posterior displacement of the . Symptoms may include dysphagia, odynophagia, difficulty breathing, and pain. Ludwig's angina should be treated urgently due to the airway being compromised. The infection may rapidly spread to other fascial spaces of the head and neck, further compromising the airway. Treatment: Treatment involves appropriate antibiotic medications, monitoring and protection of the airway in severe cases, and, where appropriate, urgent ENT surgery, maxillo-facial surgery and/or dental consultation to incise and drain the collections. The antibiotic of choice is from the penicillin group. Incision and drainage of the abscess may be either intraoral or external. An intraoral incision and drainage procedure is indicated if the infection is localized to the sublingual space. External incision and drainage is performed if infection involves the perimandibular spaces. A nasotracheal tube is sometimes warranted for ventilation if the tissues of the mouth make insertion of an oral airway difficult or impossible. In cases where the patency of the airway is compromised, skilled airway management is mandatory. Fiberoptic intubation is common. b) Estrogens and Progesterone receptors. Introduction: Estrogen and progesterone receptors are found in breast cancer cells that depend on estrogen and related hormones to grow. All patients with invasive breast cancer or a breast cancer recurrence should have their tumors tested for estrogen and progesterone receptors.

Estrogen receptors: There are two different forms of the estrogen receptor, usually referred to as α and β, each encoded by a separate gene

Both ERs are widely expressed in different tissue types, however there are some notable differences in their expression patterns:

 The ERα is found in endometrium, breast cancer cells, ovarian stromal cells, and the hypothalamus. In males, ERα protein is found in the epithelium of the efferent ducts.  The expression of the ERβ protein has been documented in ovarian granulosa cells, kidney, brain, bone, heart,lungs, intestinal mucosa, prostate, and endothelial cells. The ERs are regarded to be cytoplasmic receptors in their unliganded state, but visualization research has shown that only a small fraction of the ERs reside in the cytoplasm, with most ER constitutively in the nucleus.The "ERα" primary transcript gives rise to several alternatively spliced variants of unknown function. Progesterone receptors:

The progesterone receptor (PR, also known as NR3C3 or nuclear receptor subfamily 3, group C, member 3), is a protein found inside cells. It is activated by the steroid hormone progesterone. In humans, PR is encoded by a single PGR gene residing on chromosome 11q22, it has two main forms, A and B, that differ in their molecular weight.

Classifying breast cancer based on hormone receptors and HER2 status:

If the breast cancer cells contain either estrogen or progesterone receptors, they can be called hormone receptor-positive (or just hormone-positive). Breast cancers that are hormone receptor- positive can be treated with hormone therapy drugs that lower estrogen levels or block estrogen receptors. This includes cancers that are ER-negative but PR-positive. Hormone receptor-positive cancers tend to grow more slowly than those that are hormone receptor-negative (and don’t have either estrogen or progesterone receptors). Women with these cancers tend to have a better outlook in the short-term, but cancers that are hormone receptor-positive can sometimes come back many years after treatment. Hormone receptor-positive cancers are more common in women after menopause. Hormone receptor-negative: If the breast cancer cells don’t have either estrogen or progesterone receptors, they are said to be hormone receptor-negative (or just hormone-negative). Treatment with hormone therapy drugs is not helpful for these cancers. These cancers tend to grow more quickly than hormone receptor-positive cancers. If they return after treatment, it is more often in the first few years. Hormone receptor- negative cancers are more common in women who have not yet gone through menopause. HER2 positive: Cancers that have too much HER2 protein or extra copies of the HER2 gene are called HER2 positive. These cancers can be treated with drugs that target HER2. HER2 negative: Cancers that don’t have excess HER2 are called HER2 negative. These cancers do not respond to treatment with drugs that target HER2. Triple-negative: If the breast cancer cells don’t have estrogen or progesterone receptors and don’t have too much HER2, they are called triple-negative. These cancers tend to occur more often in younger women and in women who are African-American or Hispanic/Latina. Triple-negative breast cancers tend to grow and spread more quickly than most other types of breast cancer. Because the tumor cells don’t have hormone receptors, hormone therapy is not helpful in treating these cancers. Because they don’t have too much HER2, drugs that target HER2 aren’t helpful, either. Chemotherapy can still be useful, though. Triple-positive: This term is used to describe cancers that are ER-positive, PR-positive, and have too much HER2. These cancers can be treated with hormone drugs as well as drugs that target HER2.  The medication tamoxifen helps stop cancer from coming back by blocking hormone receptors, preventing hormones from binding to them. It’s sometimes taken for up to 5 years after initial treatment for breast cancer.  A class of medicines called aromatase inhibitors actually stops estrogen production. These include anastrozole ,exemestane, and letrozole. They’re only used in women who’ve already gone through menopause. c) Types of Anorectal abscess. There are four types of anorectal abscesses: perianal, ischiorectal, intersphincteric, and supralevator.

Aetiology:

 Nonspecific: Cryptoglandular in origin.

 Specific :

Crohn’s TB Carcinoma Radiation Lymphoma Pelvic inflammation

Ulcerative colitis Actinomycosis Trauma Foreign body Leukemia

Treatment:

 Incision and drainage.

 Determine the most tender point, a 2 cm area of skin is injected with local freezing.

 Eliptical or cruciate incision.

 Drainage of pus. Destroy all loculations. d) Appendicular lump. Causes:

 Appendicular rupture  Appendicular abscess  Acute appendicitis  Chronic appendicitis  Carcinoid tumours  Appendicular carcinoma See the Question 3.e of Group – C of Supplementary Paper – II of 2011 for Oschner Sherren regime. e) MODS. Multiple organ dysfunction syndrome (MODS), previously known asmultiple organ failure (MOF) or multisystem organ failure (MSOF), is altered organ function in an acutely ill patient requiring medicalintervention to achieve homeostasis. Multiple organ failure is defined as two or more failed organ systems Cause: The condition usually results from infection, injury (accident, surgery), hypoperfusion and hypermetabolism. The primary cause triggers an uncontrolled inflammatory response.Sepsis is the most common cause in operative and non-operative patients. Sepsis may result in septic shock. In the absence of infection, a sepsis-like disorder is termed as systemic inflammatory response syndrome (SIRS). Both SIRS and sepsis could ultimately progress to multiple organ dysfunction syndrome. However, in one-third of the patients no primary focus can be found Multiple organ dysfunction syndrome is well established as the final stage of a continuum: SIRS + infection sepsis severe sepsis Multiple organ dysfunction syndrome. Currently, investigators are looking into genetic targets for possible gene therapy to prevent the progression to Multiple organ dysfunction syndrome. Some authors have conjectured that the inactivation of the transcription factors NF-κB and AP-1 would be appropriate targets in preventing sepsis and SIRS. These two genes are pro-inflammatory. However, they are essential components of a normal healthy immune response, so there is risk of increasing vulnerability to infection, which can also cause clinical deterioration.

Effects of organ failure:  Lung- Acute respiratory distress syndrome  Kidney- Acute renal insufficiency  Liver- Acute liver insufficiency  Clotting- Coagulopathy  Cardiac - Cardiovascular failure Group – D 4. Write briefly on (any three): 3x5 = 15 a) Ring sequestrum. b) Exostosis of bone. c) Volkmann’s ischaemic contracture. d) Pathological fracture. e) Ideal amputation stump. Answer.

a) Ring sequestrum. See the Question 4.d of Group – D of Supplementary Paper – I of 2012. Ring sequestrum is seen in amputation stump and at Steinmann pins. a) Exostosis of bone. Also known as Osteochondroma. See the Question 4.b of Group – D of Paper – I of 2014. b) Volkmann’s ischaemic contracture. See the Question 4.a of Group – D of Paper – I of 2013. c) Pathological fracture. See the Question 4.b of Group – D of 2009. d) Ideal amputation stump.

Introduction: Should be covered with durable – vascular – subcutaneous tissue and skin – that can withstand the pressure in weight bearing areas and friction in areas of prosthesis Methods for predicting the level to which adequate vascularity for primary wound healing:  Trans cutaneous measurement of oxygen tension – best  Tissue clearance of intradermally injected xe-133  Tissue uptake of i/v fluorescein  Thermography  Laser doppler flowmetry  If ambulation of patient chief concern - should be done at maximum distal level  If patient has no ambulatory potential – wound healing with decreased perioperative morbidity should be the chief concern  In any case the ideal stump should have minimum length to allow prosthetic fitting Ideal length and shape: Bone end well covered with muscles:  Stump should have end bearing  Should have side pad or bony surface of sufficient size that can bear weight for varying period – especially in partial foot amputations  In case of ankle or knee disarticulation – end bearing can be created by tibia – fibular synostosis  Disarticulation  Desirable in growing children and elderly  Children : overgrowth seen in diaphyseal amputation - avoided by retaining epiphysis  Elderly : end bearing improves prosthetic fitting and chances of rehabilitation  Non adherent incision scar  Scar  Position of scar not important with advent of modern total contact prosthetic socket  Should not be adherent to bone  Large dog ears not desirable  Redundant soft tissue creates problems in prosthetic fittings - skin and muscle flap over stump should be thick Muscular with good muscle power:  Muscle divided at least 5cm distal to the intended bone resection  Stabilized by myodesis ( muscle or tendon sutured to bone) or by myoplasty (muscle to periosteum or to fascia of opposing musculature)  Myodesis better option  Stronger insertion  Maximize strength  Minimize atrophy  Can counterbalance their antagonists  Prevent contracture  Maximize residual limb functions  Myodesis contraindicated in severe ischemia – risk of wound breakdown Absence of neuroma:  Nerves transected to reduce risk of neuroma Various methods include:  End-loop anastomosis  Perineural closure  Silastic capping  Sealing the epineural tube with butyl-cyanoacrylate  Ligation  Cauterization  Bury nerve ends in bone/ muscles  Nerves should be isolated – pulled distally to wound – divided with sharp knife – nerve retracts well proximal to level of bone resection Free from infection: Full and free movements at the joint above: No fixed deformity:  Excessive periosteal stripping not advised – results in ring sequestra, bone overgrowth  bone prominence not covered by adequate soft tissue resected – remaining surface rasped to smooth contour  definitive prosthesis fitted – strong and functional stump for maximum prosthetic use

The West Bengal University of Health Sciences

M.B.B.S. 3rd Professional Part – II Examination, 2014

1 Subject: Surgery Time: 2 /2 hrs.

Paper: I Marks: 60

Group – A

1) Classify shock. Discuss the pathophysiology and management of septic shock. 3+6+6 =15

Answer. According to Bailey and Love:

Classification of shock:

 Hypovolaemic shock  Cardiogenic shock  Obstructive shock  Distributive shock  Endocrine shock

Other classification:

a. Cardiogenic : Shock caused as a result of cardiac pump failure

Causes:

 Myocardial Infarction  Arrythmias (Atrial fibrillation, ventricular tachycardias, bradycardias, etc)  Mechanical abnormalities (valvular defects)  Extracardiac abnormalities (PE, pulm HTN, tension pneumothorax) b. Hypovolemic: Shock caused by decreased preload due to intravascular volume loss.

Causes:

 Hemorrhagic – trauma, GI bleed, hemorrhagic pancreatitis, fractures  Fluid loss induced – Diarrhea, vomiting, burns

c. Distributive: Shock as a result of severely diminished systemic vascular resistance.

a. Septic: secondary to an overwhelming infection

b. Anaphylactic: secondary to an overwhelming infection

c. Neurogenic: secondary to a sudden loss of the autonomic nervous system function. d. Combined.

Pathogenesis of sepsis and septic shock:

Pathophysiology of septic shock:

General Clinical Signs:  Flu-like symptoms – fever, chills – general malaise, irritability, lethargy  Tachycardia and hypotension  Hyperventilation  Site of infection may or may not be evident Cardiovascular:  Systemic vasodilation and hypotension (Psys < 90 mmHg)  Tachycardia (>100 beats/min)  Increased cardiac output (hyperdynamic), although contractility is depressed; hypodynamic in late shock  Ventricular dilation; decreased ejection fraction  Loss of sympathetic responsiveness  Hypovolemia due to vascular leakage; central venous pressure may be decreased or increased depending upon fluid resuscitation  Compromised nutrient blood flow to organs; decreased organ oxygen extraction Pulmonary and renal:  Hyperventilation with respiratory alkalosis  Pulmonary hypertension and edema  Hypoxemia (arterial pO2 < 50 mmHg)  Reduced pulmonary compliance; increased work  Respiratory muscle failure  Renal hypoperfusion; oliguria  Acute tubular necrosis and renal failure Other  Disseminated intravascular coagulation (DIC)  Blood dyscrasias: o Leukopenia o Thrombocytopenia o Polycythemia  Central and peripheral nervous dysfunction  Increased lactate occurs early

Management:

Diagnosis:

 Before the initiation of antimicrobial therapy, at least two blood cultures should be obtained  At least one drawn percutaneously.  At least one drawn through each vascular access device if inserted longer than 48 hours  Other cultures such as urine, cerebrospinal fluid, wounds, respiratory secretions or other body fluids should be obtained as the clinical situation dictates  Other diagnostic studies such as imaging and sampling should be performed promptly to determine the source and causative organism of the infection may be limited by patient stability.

Treatment primarily consists of:

1. Volume resuscitation 2. Early antibiotic administration 3. Early goal directed therapy 4. Rapid source identification and control. 5. Support of major organ dysfunction. 6. Sequestration of lipopolysaccharides.

 Antibiotics (early administration)  Hemodynamic support – (fluid resuscitation)  Restore tissue perfusion  Normalize cellular metabolism  Tight glycemic control  Vasopressor agents Dopamine, norepinephrine, dobutamine  Source control – Surgical debridement of infected, devitalized tissue – Catheter replacement  Supplemental oxygen (treatment of acute respiratory distress syndrome,ARDS)  Nutritional support  Anti-inflammatory agents : o Cortocosteroids o Ibuprofen, o Prostaglandin E1 o Pentoxifylline  Oxygen Scavengers :N-acetylcysteine , selenium  Drugs modifying coagulation – Anti-thrombin III  Drugs enhancing host defenses o Intravenous immunoglobulin (IVIG) o Interferon-gamma o GM-CSF o Immunonutrition  Other drugs o Growth hormone, antibiotics, fresh frozen plasma, anesthetic sedative and analgesic agents, catecholamines  Hemofiltration, plasma filtration, plasma exchange  Experimental therapies o Anti-endotoxin therapies – IVIG, BPI protein o IL-1Ra o Anti-TNF-alpha, soluble TNFR o PLA2 inhibitors, PAF inhibitors o iNOS inhibitors o Anti-coagulants (APC)

Group – B

2. Answer any of the following questions: a) Describe the clinical features, diagnosis and management of carcinoma stomach. 4+4+7 = 15

Answer.

Clinical features:

Symptoms:

 Dyspepsia (any new onset of dyspepsia over the age of 45 should be considered to be due to adenocarcinoma until proven otherwise).  Weight loss, anorexia, and lethargy.  Anaemia (iron deficiency due to chronic blood loss).  Occasionally presents as acute upper GI bleeding.  Dysphagia uncommon unless involving the proximal fundus and gastro-oesophageal junction.  Features of gastric outlet obstruction.

Signs: Weight loss. Cervical and axillary Sister Joseph’s nodule lymphadenopathy Palpable epigastric mass. Pleural effusion Ascites Palpable supraclavicular lymph Aspiration pneumonitis Rectal shelf of Blumer node (Troisier's sign) suggests disseminated disease.

In Early Gastric Cancer:

 New dyspepsia, however mild, in a patient over 40 years – gastric c.a. to be ruled out persistent dyspepsia at any age.

In Advanced Gastric Cancer:

 Early satiety,bloating,distension,vomiting  Tumour frequently bleeds – haematemesis, maleana, iron deficiency anaemia  May lead to gastric outlet obstruction– dysphagia,epigastric fullness,vomiting (alkalosis less compared to obstruction due to ulceration)

Paraneoplastic syndromes – thrombophlebitis(Trousseau’s sign), deep vein thrombosis, Acanthosis nigricans (hyperpigmentation of axilla and groin), peripheral neuropathy

Investigations:

 Upper gastrointestinal (GI) tract endoscopy and biopsy  Gold standard  Best pre-operative staging  Needle aspiration of LN w/ ultrasound guidance  Can even give preop neoadjuvant tx.  Chest/abdomen/pelvic computed tomography (CT) with oral and intravenous contrast  Positron emission tomography (PET) – CT evaluation if no evidence of M1 disease is found, and if clinically indicated  Complete blood cell count (CBC) and comprehensive chemistry profile  Endoscopic ultrasound if no evidence of M1 disease is found  Endoscopic resection for early-stage cancers  Biopsy of metastatic disease as clinically indicated  HER2-neu testing if metastatic adenocarcinoma is documented or suspected.

Staging investigations include:

 Thoraco-abdominal CT scan to assess for distant metastases and local lymphadenopathy;  Endoluminal ultrasound to assess for local disease;  Laparoscopy (for patients considered for potential resection) to exclude small volume peritoneal metastases.

Treatment: Surgical principle:  Surgery is the principal modality of treatment of gastric c.a. with curative intent  Stage of the disease, evidence of spread, age and build (general condition) – all should be considered  Components of curative gastric cancer treatment consist following – . Extent of gastric resection . Extent of lymphadenectomy . Splenectomy and Distal Pancreatectomy

Extent of Gastric Resection:  Extent of resection determined by obtaining a margin free of microscopic disease.  A line of resection at least 5-6 cm. from tumour mass necessary for low local recurrence.  In distal third cancer distal stomach with 2 cm of 1st part of duodenum removed and reconstruction by various methods.  In middle third cancer subtotal/total gastrectomy done. A minimum of 2 cm gastric remnant required for good reconstruction, so proximal margin of palpable tumour mass must be at least 7-8 cm from esophago-gastric junction for subtotal gastrectomy.  In proximal third cancer radical total gastrectomy with jejunal pouch esophageal anastomosis recommended.

Surgical resection: • Resection of tumour • Grossly negative margin of at least 5 cms • Partial gastrectomy • Confirmed on frozen section • En block resection of adjacent involved organs

Extent of Gastrectomy:

 Radical subtotal gastrectomy (Distal tumour)  Total gastrectomy (Proximal tumour)

Early gastric cancer (T1 or 2, N0/1) Advanced gastric cancer (T3 or more or any of N2)

Suitable for attempted curative resection if the patient is Surgical intervention very unlikely to be curative. medically fit enough. May be undertaken for palliative treatment. Surgery is simple or radical gastrectomy. Local ablation for symptom control occasionally possible. Local resection or ablation has an uncertain place in Palliative chemotherapy occasionally effective treatment. for disseminated disease.

Extent of lymphadenectomy:

N1 – 3 to 6 N2 – 1, 2, 7, 8 & 11 N3 – 9, 10 & 12

. N1 nodes are w/in 3cm of the tumor . N2 along hepatic & splenic arteries . N3 more distant nodes. Adjuvant chemo and radiotherapy: For stages II and III adenocarcinoma

 Chemotherapy:

. 5-fluorouracil, leucovorin, cisplatin, doxorubicin and methotrexate

. Can not prolong survival in unresectable, metastatic or recurrent diseases

 Radiation (4500cGy):

. Effective in palliation for pain and bleeding

b) A 50 years male presents with bleeding per rectum. How will you investigate and manage the patient? 7+8 = 15

Answer.

Causes of rectal bleeding can be:

 Haemorrhoids.  Infectious gastroenteritis  Anal fissure.  Angiodysplasia  Diverticular disease  Ischaemic colitis  Rectal cancer  Solitary rectal ulcer  Colonic cancer  Anal cancer  Colonic polyps  Sexually transmitted diseases  Radiation proctitis  Anorectal trauma

Clinical examination and Investigations:

 Examination of the abdomen to exclude abdominal mass and digital rectal examination (DRE) to examine for fissure and exclude rectal cancer may be useful. DRE may be deferred in patients with a good history for hemorrhoids.  Proctoscopy will confirm haemorrhoids.  Baseline blood tests may be useful to rule out severe anemiaand need for blood transfusion. Other blood tests will only be necessary if there are other features in the history, e.g. unexplained weight loss.  Faecal occult blood testing has no place if there is frank bleeding.  Faecal calprotectin is a useful screening tool for inflammatory bowel disease.. A positive faecal calprotectin result has a high positive predictive value (PPV) for finding inflammatory bowel disease at colonoscopy.  While proctoscopy can be used by as a screening tool in patients with rectal bleeding.  Flexible sigmoidoscopy is done to rule out serious pathology. Red flag symptoms and signs in patients with rectal bleeding include:

. Associated change in bowel habit, especially diarrhoea or increased frequency . Anaemia . Weight loss . Abdominal or rectal mass. This means that there should be a high index of suspicion of other pathology if symptoms and/or clinical cause do not follow common patterns suggestive of benign disease.

Further investigations:

 Anoscopy: A plastic or metal scope placed into the anus allows for quick examination of the rectal vault.  Flexible sigmoidoscopy: A flexible tube inserted into the rectum is used to evaluate the rectum and lower end of the colon.  Colonoscopy: A soft tube equipped with a light and camera is inserted into the rectum and pushed into the colon. The entire large colon is visualized. It is used to locate areas of bleeding, masses, or irregularities.Diagnostic biopsy can be taken also.  Barium enema X-ray: This study uses liquid barium inserted into the rectum. An X-ray is taken to highlight problem areas such as tumors or diverticula. However, sites of active bleeding cannot be distinguished.  Nuclear medicine studies: A tagged red blood cell scan may be used to pinpoint areas of slow bleeding.  CT scan: May be used to diagnose diverticulitis or tumors in the bowel.  Angiography: A contrast dye study is used to evaluate active areas of brisk bleeding.

Treatment:

General measures:

 Correct anemia.  Confirm diagnosis.

Specific treatment:

Haemorrhoids:

 Non-Surgical Treatment of Haemorrhoids:  Dietary: ↑ fluid & fibre  Medication – Stool softener – Bulk forming agent – Topical agent  Office treatment – Injection therapy – Rubber band ligation

 Haemorrhoidectomy:

 Anaemia due to bleeding haemorrhoids  Persistent significant bleeding after two attempts of rubber band ligation within 12 months.

Anal fissure:

 Medical treatment:  Dietary: ↑ fluid & fibre  Stool softener – Bulk forming agent  Topical analgesic  Topical ointment & bath  Surgical treatment of chronic fissure: (Lateral anal sphincterotomy)  Failure to heal after 8 weeks of medical treatment

Inflammatory bowel disease: Medical treatment

Colo-rectal cancer: Operative management followed by chemo/+ Radiotherapy.

Note: Differential Diagnosis of Lower Gastrointestinal Hemorrhage: Colonic bleeding Small bowel bleeding

Diverticular disease Angiodysplasias

Ischemia Erosions, ulcers (e.g., from potassium, NS AIDs)

Anorectal disease Crohn’s disease

Neoplasia Radiation

Infectious colitis Meckel’s diverticulum

Postpolypectomy Neoplasia

Inflammatory bowel disease Aortoenteric fistula

Angiodysplasia

Radiation colitis, proctitis

Other

Unknown

Note: Algorithm for diagnosis and management of lower GI hemorrhage.

NGT, Nasogastric tube.

Group – C

3. Write short notes on (any three): 3x5 = 15 a) Breast biopsy. b) Incarcerated hernia. c) Blood substitutes. d) Volvulus Neonatorum. e) Amoebic liver abscess.

3. Answers. a) Breast biopsy.

Breast biopsies: Breast Biopsy techniques: Fine needle aspiration Biopsy (FNAB):

The primary advantage of the FNAB is the simplicity of the technique, readily available and it is relatively atraumatic for the patient. Limitation of the FNAB is availability of a experienced cytopathologist for interpretation of the results. The major limitation of FNAB is the inability of the cytology to distinguish invasive cancer from in situ disease.

Core Needle Biopsy (CNB): The Core needle Biospy (CNB) is performed by using automated gun attached to 14 / 18 gauge cutting needle. Tissue cores can be processed for complete pathologic assessment. Sensitivity rate for CNB are almost 100% for diagnosing the breast lesions.

FNAC / core biopsy:  In a clinically and mammographically suspicious mass, sensitivity and specificity of FNAC approaches 100%.  FNAC requires an experienced cytopathologist for accurate interpretation Histopathologic type and grade of malignancy, ER / PR receptor status, HER2/neu status can also be reported on cytopathology by an experienced cytopathologist using a cell block.  False negative rate for core needle biopsy is very low. However, a tissue specimen that does not show breast cancer cannot conclusively rule out malignancy as sampling error can occur.  Histologic type and grade of malignancy, receptor status and HER2/neu status can be easily made out on core needle biopsy.  Fewer “suspicious for malignancy” report with core-needle biopsy. However, more painful,and leaves a parenchymal scar.  Any patient scheduled for neoadjuvant chemotherapy should have the tumour pathology type and grade, receptor and HER2/neu status documented either on FNAC or core needle biopsy before starting chemotherapy, for in patients with complete response to chemotherapy, there will be no tumour tissue in the surgical specimen.

Incisional Biopsy: There are few indications for open surgical biopsy to incise a suspicious breast mass.  It is done in a fungating breast mass. The edge biopsy with some normal skin margin is taken in fungating breast mass.  Some cases of inflammatory breast cancer may also be suitable for incisional biopsy if core biopsy is not diagnostic.

Excisional biopsy:

Any suspicious breast lesion in which histologic diagnosis was not possible by one of the needle biopsy techniques because of either technical considerations, then an excisional biopsy is indicated. Grossly, an attempt should be made to excise to excise an approx. 1 cm thickness of normal appearing tissue surrounding the index lesion. b) Incarcerated hernia.  An incarcerated hernia cannot easily be returned to its original compartment.  The overlying skin should appear to be normal, the contents should not be tense, and bowel sounds can sometimes be heard.  The incarcerated tissue may be bowel, omentum, or other abdominal contents.  A smaller aperture of herniation and adhesions can precipitate incarceration.  It can obstruct the bowel, leading to severe pain, nausea, vomiting, and the inability to have a bowel movement or pass gas.  An incarcerated hernia can often be reduced manually, especially with sufficient anesthesia. c) Blood substitutes. Introduction: A blood substitute (also called artificial blood or blood surrogates) is a substance used to mimic and fulfill some functions of biological blood. It aims to provide an alternative to blood transfusion, which is transferring blood or blood-based products from one person into another. The need for blood the need for blood substitutes: Three major problems in donor RBC – The need for cross matching – Relatively short storage life (42 days) – Transmission of infectious/anaphylactic agents  Immunological effects of blood transfusions are associated with higher frequencies of surgical infections, delayed wound healing and progression of malignant disease.  Decrease in potential donor population – Red Cross prohibits donations from people who have lived in the UK for greater than six months.

Characteristics of an ideal blood substitute: The most important adverse effects of blood transfusion relate to the antigenicity of donor blood and its ability to transmit infections. Hence, an ideal blood substitute should lack antigenicity and eliminate, or at least substantially reduce, the ability to transmit infections. In addition, it should be readily available, should have a long half-life, and should be capable of being stored at room temperature. The biologic properties of an ideal blood substitute should include a reasonable amount of oxygen delivery, when compared to normal human red blood cells.

Classification:

o Oxygen carriers:

1st generation products:

. Perflurocarbons: As name suggests, fluorine and carbon atoms. They are capable of dissolving large amounts of many gases, including oxygen. These molecules are hydrophobic in nature, and hence have to be emulsified prior to intravenous administration.

Stroma – free haemoglobin:

Hemoglobin-based Diaspirin cross-linked Increased mortality in phase III substitutes (human) hemoglobin/HemAssist (Baxter) trials Trials closed PolyHeme (Northfield) Elevated bilirubin and amylase Phase III trials Hemolink (Hemosol) Increased rate of myocardial infarcts, trials halted PHP (Apex Bioscience) Trials terminated Hemospan (Sangart) Phase II trials Hemoglobin-based Optro (Somatogen/Baxter) All phase I trials terminated substitute (recombinant)

 Next generation blood substitutes:

PEG(Polyethylene glycol) conjugated HB

Raffinose-x-linked HB

Adverse effects:

Adverse effects associated with hemoglobin-based oxygen carriers include hypertension, abdominal pain, skin rash, diarrhea, jaundice, hemoglobinuria, oliguria, fever, stroke, and laboratory anomalies such as an elevation in lipase levels. Although most of these side effects were transient and clinically asymptomatic, many clinical trials involving these agents have been discontinued or held due to the associated adverse effects.

d) Volvulus Neonatorum.

Malrotation with or without volvulus:  This is an embryonic abnormality that makes the embryonic midgut prone to twist in a clockwise manner around the superior mesenteric vessels due to a very narrow base of the mesentery.  It results from an embryonic failure of rotation and fixation of the midgut during 8-12 week of intrauterine life. During early embryogenesis the small bowel grows rapidly and extrude into the extra embryonic coelome. During 8-12 week it returns back to the abdominal cavity and rotates anticlockwise along the axis of SMA.  The DJ flexure rotates 270 degree to rest above and to the left of pylorus. The root of mesentery elongates so that the Ileocaecal junction lies in the right iliac fossa – the root of mesentery stretches from DJ to ICJ.  Failure of this process of rotation and fixation results in abnormal location of DJ, juxtaposition of duodenum and caecum close to each other and no length of the mesentery.  A band (Ladd band) runs from the retroperitoneum across the caecum and duodenum at the third part of the duodenum.  Clinical features may be due to volvulus of the small bowel around SMA (most common and most dreaded) or chronic duodenal obstruction due to Ladd bands or due to an intrinsic duodenal stenosis.  Volvulus of the midgut occurs most frequently within the first week of life with sudden onset of bile vomiting on 3-5 th day.  Rapid deterioration occurs because of gut ischemia and the child may present in shock.  A quick resuscitation should be followed by an upper GI study if the child is resuscitable.  Many a time laparotomy is required urgently as part of resuscitation.  The key to surgery is the root of the mesentery. The volved gut is untwisted anticlockwise (usually two and a half turns). Mesentery is widened and Ladd band is divided. The gut is positioned in the position of non rotation i.e. the DJ lies to the right with duodenum straight, whole small bowel on right, whole large bowel on left with the caecum in the left upper abdomen.  Inversion appendicectomy may be performed to avoid diagnostic difficulty and delay in treatment of a future appendicitis (due to changed location). Laparoscopic approach to malrotation is also possible.

Causes of neonatal intestinal obstruction: e) Amoebic liver abscess. Introduction:

Amebic liver abscess is caused by the parasitic protozoan Entamoeba histolytica.

Aetiology: Two species of ameba infect humans. E. dispar is associated with an asymptomatic carrier state and not with disease. E. histolytica is responsible for all forms of invasive disease. The life cycle involves cysts, invasive trophozoites, and fecally contaminated food or water to initiate the infection. Fecal-oral transmission occurs; the cyst passes through the stomach into the intestine unscathed, andthen pancreatic enzymes start to digest the outer cyst wall. The trophozoite is then released into the intestine and multiplies there. Normally, no invasion occurs, and the patient develops amebic dysentery alone or becomes an asymptomatic carrier. In a small number of cases, the trophozoite invades through the intestinal mucosa, travels through the mesenteric lymphatics and veins, and begins to accumulate in the hepatic parenchyma, forming an abscess cavity. Liquefied hepatic parenchyma with blood and debris gives a characteristic "anchovy paste" appearance to the abscess.

SYMPTOM Pain Fever Nausea and vomiting Anorexia Weight loss Malaise Diarrhea Cough or pleurisy Pruritus SIGN Hepatomegaly Right upper quadrant tenderness Pleural effusion or rub Right upper quadrant mass Ascites Jaundice

LABORATORY DATA Increased alkaline phosphatase WBC count >10,000/mm3

Hematocrit <36% Albumin <3 g/dL Bilirubin >2 mg/dL

Diagnosis • Blood culture • Haemoglobin Estimation • Stools Examination (Tropazoites and Cysts) • Radiography • Aspiration • Medical untrasonography and CT Scannig • Sigmoidoscopy • Liver fubction tests • Serological Tests

Distinguishing Clinical Characteristics of Patients with Hepatic Abscesses:

Amebic Pyogenic Age <50 years Age >50 years Male:female ratio 10:1 Male:female ratio 1:1 Hispanic descent No ethnic predisposition Recent travel to endemic area Malignancy Pulmonary dysfunction High fevers Abdominal pain Pruritus Diarrhea Jaundice Abdominal tenderness Septic shock Hepatomegaly Palpable mass

Management:

• Metronidazole 800mg TDS for 5-10 days • Aspiration • Repeated Imaging Of Liver

Complications:  Complications from amebic abscesses occur secondary to rupture of the abscess into the peritoneum, pleural cavity, or pericardium.  Extrahepatic sites also have been described in the lung, brain, skin, and genitourinary tract, presumably from hematogenous spread.  Ruptured amebic liver abscesses are associated with mortality rates upto 50%.

Group – D

4. Write briefly on (any three): 3x5 = 15

a) Fractures occurring due to fall on outstretched hand. b) Osteochondroma. c) Greenstick fracture. d) Fracture of patella. e) Spina bifida.

4. Answers. a) Fractures occurring due to fall on outstretched hand.

1. Colle’s fracture: The most common fracture occurring as a result of a fall on a palm is the Colle’s fracture. This fracture is transverse through the distal metaphysis of the radius. Because of the hyperextensive forces applied to the wrist, there is dorsal angulation and/or displacement of the distal fracture fragment relative to the radial shaft. 2. Bennetts fracture : Bennett fracture is a fracture of the base of the first metacarpal bone which extends into the carpometacarpal (CMC) joint. This intra-articular fracture is the most common type of fracture of the thumb, and is nearly always accompanied by some degree of subluxation or frank dislocation of the carpometacarpal joint. 3. Clavicle fracture (rare) : 4. Reverse Bennetts fracture: The fracture-dislocation of the base of the fifth metacarpal. 5. Rolando fracture: A Rolando fracture is a three part or comminuted intra-articular fracture-dislocation of the base of thumb (proximal first metacarpal). It can be thought of as a comminuted Bennett fracture. 6. Smiths fracture: Smith fractures (also known as a Goyrand fracture in the French literature) are fractures of the distal radius with associated palmar angulation of the distal fracture fragment. Classically, these fractures are extra-articular transverse fractures and can be thought of as a reverse Colles fracture. The term is sometimes used to describe intra-articular fractures with volar displacement (a reverse Barton fracture) or juxta- articular fractures 7. Scaphoid fracture: A scaphoid fracture is usually caused by a fall on an outstretched hand, with the weight landing on the palm. The end of one of the forearm bones (the radius) may also break in this type of fall, depending on the position of the hand on landing. b) Osteochondroma.

Introduction:  Is a benign tumor.  Also known as exostosis.  Commonest benign tumour of bone.  Metaphyseal developmental abnormality.

Clinical presentation:

 Children and teenagers most affected, Male >> female.  Tumors most commonly affect long bones in the leg,pelvis, or scapula (shoulder blade).  Clinically appear as slowly growing masses, painful if press on nerve tissue  Solitary or multiple  Multiple Hereditary Exostosis autosomal dominant disease with inactivation of both copies of EXT gene in growth plate chondrocytes.

Origin:

 Develops in bones of endochondral origin and arises from the metaphysis near the growth plate of long tubular bones.  Occurs in epiphyseal plate and grows laterally  Occasionally develops from bones of the pelvis, scapula and ribs.

Pathology:

 Benign projection of bone with cartilaginous cap  Cartilage cap resembles normal growth plate  Cartilage more disorganized  Exhibits cortex and medullary portion.  Covered by thin layer of periosteum  Binucleate chondrocytes in lacunae.  Conversion to sarcoma rare (<1%) but higher in patients with hereditary syndrome.  May convert to malignancy if cartilage cap becomes thicker and contains disorganized calcifications.

Imaging:

 X-ray: o Flat/ sessile / pedunculated o Tumour blending into metaphysis o Pedunculated orientated proximally o Cartilage cap with calcification  Ultrasound safe and inexpensive way to evaluate thickness of cartilaginous capsule  MRI method of choice to evaluate thicknesss of cartilaginous cap to rule out malignant conversion.

Treatment:

 Nil required unless symptomatic  Extra-capsular marginal excision  Recurrence < 5%

c) Greenstick fracture.

Introduction: A greenstick fracture is a fracture in a young, soft bone in which the bone bends and breaks. Despite the name fracture, during the majority of cases the bone is broken and hard to find in x- rays. Causes: Childhood fractures most commonly occur with a fall. Arm fractures are more common than leg fractures, since the usual reaction is to throw out the arms when one starts to fall. Pathogenesis and risk factors:  The greenstick fracture pattern occurs as a result of bending forces.  Activities with a high risk of falling are risk factors.  Non-accidental injury more commonly causes spiral (twisting) fractures but a blow on the forearm or shin could cause a green stick fracture.  The fracture usually occurs in children and teens because their bones are flexible, unlike adults whose more brittle bones usually break. Clinical features:  Some clinical features of a greenstick fracture are similar to those of a standard long bone fracture - greenstick fractures normally cause pain at the injured area.  As these fractures are specifically a pediatric problem, an older child will be protective of the fractured part and babies may cry inconsolably.  As per a standard fracture, the area may be swollen and either red or bruised.  Greenstick fractures are stable fractures as a part of the bone remains intact and unbroken so this type of fracture normally causes a bend to the injured part, rather than a distinct deformity, which is problematic. Diagnosis:  During the physical exam, inspect the affected area for tenderness, swelling, deformity or an open wound. The child will be asked to gently move that limb's fingers or toes.

 X-rays can reveal most greenstick fractures. Some greenstick fractures are difficult to see because a small bend in the bone may not show up as well on X-rays. Treatment:  Most fractures of the arms and legs require a cast to keep the bones in good alignment while the break heals. If the bones are in a poor alignment, they may need to be repositioned, typically under sedation.

 On occasion, a removable splint could work just as well, particularly if the break is mostly healed. The benefit of a splint is that the child might be able to take it off briefly for a bath or shower.

 X-rays are required in a few weeks to make sure the fracture is healing properly, to check the alignment of the bone, and to determine when a cast is no longer needed. Most fractures or breaks require four to eight weeks for complete healing, depending on the break and the age of the child.

After the cast is removed, the child should avoid high-impact activities for another one to two weeks to keep from re-injuring the arm or leg. Your child will quickly rebuild muscle and function in the limb with normal daily activities. Physical therapy usually isn't required. d) Fracture of patella.

Classification:  Displaced  Non-displaced o 1 to 2 mm articular step-off o 3 mm fragment separation  Transverse  Longitudinal  Comminuted

Mechanism of injury:

 Direct blow to patella  Indirect force from violent quadriceps contraction  Expected healing time: 8-12 weeks  Rehabilitation duration: 12-15 weeks

Clinical features:

 Pain  Swelling and redness over knee  Restriction of movement

Treatment goals:

 Alignment o Open repair of the retinaculum if extensor lag is present  Stability o Best achieved by restoring bony congruity with hardware to rigidly fix fracture

Treatment:

 Operative vs Nonoperative:  If able to straight leg raise and less than 2mm of step off, non-operative.  Disrupted mechanism – operative  If question, have patient lay on side to fire extensor  Patella Fractures: Surgical Options:  Fixation  Partial Patellectomy  Total Patellectomy  Ilizarov  Fixation of Patella Fractures:  Tension band, modified tension band  Cannulated screws  Suture repair  Cannulated Screw Tension Band with wire  Cannulated Screw Tension Band with cable  Plates  Tips for Modified Tension Band  Avoid prominent hardware with modified tension band  Bend both end of the wires to avoid migration  Helpful to use a suture for cerclage rather than wire  Tools for Using a Cannulated Screw Tension Band with Cable:  mm cable with preloaded crimp  4.0 mm partially threaded cannulated screws e) Spina bifida.

Definition: It is a congenital abnormality with developmental defect in the spinal column with incomplete closure of vertebral canal due to failure in fusion of vertebral arches ± protrusion and dysplasia of the spinal cord or its membranes.

 It arises in the first few weeks of gestation, with unknown cause

 It arises when one vertebrae does not close normally and leave the spinal cord exposed

 It is a midline defect of the: Skin, bone spinal column and spinal cord.

 3 in 1,000 births

Types:

Occulta :

 Mildest form of spina bifida

 The outer part of some of the vertebrae are not completely

 The split in the vertebrae is so small that the spinal cord does not protrude

 The condition is asymptomatic

 Bony abnormality seen by X – ray

 Nerves may be involved when associated with hairy patch or other skin changes

Cystica:

1- Meningeocele:

• Vertebral arches are unfused & with herniation of the meninges part of the cord or nerves root may present in the sac but they conduct impulses normally.

• Meningeocele is covered by the outer layer of skin and the inner layer of meninges which communicate with meninges lining the spinal subarachnoid space2- Myelomeningeocele:

• Sac contains neural elements that protrude through the spinal defects

• The overlying skin is thin & leaks of spinal fluid

• Secondary infection is common , Neurological and Orthopaedic problem are present. • Hydrocephalus(arnold- chiari malformation)

• Clinical Picture: It will differ according to the level of lesion (most common site is lumbosacral region)

• Flaccid paralysis, muscle weakness, wasting, decreased or absent tendon reflexes

• Decreased or absent extroceptive & proprioceptive sensation

• Rectal & bladder incontinence, Hydrocephalus, Sever vasomotor changes

• Paralytic or congenital deformities as in club foot , pes cavus.

• Pressure ulceration due to poor sensation, Osteoporosis , soft tissue contracture

• Physical emotion& mental delay

• Prognosis:

• With successful closure of simple meningeocele prognosis is good

• Myelomeningeocele - die from infection, if survive after proper closure - stationary disability

Proposed Aetiologies for Spina Bifida:

Multi-factorial Vitamin Zinc deficiency. Viral Mineral deficiency. inheritance. deficiencies/folate. infection. Potato blight. Maternal fever High sound Alcohol. Medication – Phenytoin, intensity. Epilim, etc.

Screening and diagnosis:

Prenatal Tests Evaluation Imaging Gait Analysis Studies

AFP Analysis of individual X-rays medial history Ultrasound Physical examination Ultrasound Testing of Evaluation of critical CT scan Amniotic fluid body systems MRI

Treatment:  No cure

 Regular check-ups with physician

 Surgery (24 hours after birth)

 Medication

 Physiotherapy

 Bowel , bladder training.

Surgery:

 Usually performed within 24 hours after birth.

 They remove the infected area and replace it with muscle tissue and skin.

 Helps protect against hydrocephalus.

The West Bengal University of Health Sciences

M.B.B.S. 3rd Professional Part – II Examination, 2013

1 Subject: Surgery Time: 2 /2 hrs.

Paper: I Marks: 60

Group – A

1. Discuss assessment of Burn wound. Write in short pathophysiology of Burn injury. How would you treat 30% burn in 50 kg. body weight female patient. 3+6+6 = 15

Answer: Assessment:

 The mechanism of injury  Associated injuries  Patient age: Patient age has a major effect on outcome, with infants and elderly patients being at highest risk.  State of health: Preexisting medical problems affecting management should be noted, including allergies, medications, hypertension, and diabetes mellitus.  Prehospital treatment  Prehospital treatment is ascertained and recorded, including care provided by the patient and by the emergency response team. Administered fluids are documented carefully and subtracted from estimated fluid requirements for the first 24 hours of injury. prehospital setting.

Primary survey:

 Primary survey should follow the guidelines established by the American College of Surgeons' Advanced Trauma Life Support Course.  Airway assessment and security is the number one priority.  Breathing is evaluated for effort, depth of respiration, and auscultation of breath sounds.  Circulation. Circulatory support in the form of aggressive and prompt fluid resuscitation is a cornerstone of early burn management.  Remove all clothing to halt continued burn from melted synthetic compounds or chemicals and to assess the full extent of body-surface involvement in the initial examination.

Burn-specific secondary survey:

Depth of burn:

o First-degree burns are limited to the epidermis. o Second-degree burns, which are subdivided into superficial or deep partial- thickness burns, are limited to the dermal layers of the skin. o Full-thickness (third- or fourth-degree) burns

Rule-of-Nines Estimation of Percentage of Body Surface Area:

Victim Head and Trunk Extremity Genital neck Anterior posterior Upper (for Lower(for each limb) each limb) Adult 9 18 18 9 18 1 Infant 18 18 18 9 14 -

Small areas: palm of patient's hand equals 1% of BSA

Pathophysiology of burn:

Local response

The three zones of a burn:

 Zone of coagulation: This occurs at the point of maximum damage. In this zone there is irreversible tissue loss due to coagulation of the constituent proteins.  Zone of stasis: The surrounding zone of stasis is characterised by decreased tissue perfusion. The tissue in this zone is potentially salvageable. The main aim of burns resuscitation is to increase tissue perfusion here and prevent any damage becoming irreversible. Additional insults—such as prolonged hypotension, infection, or oedema— can convert this zone into an area of complete tissue loss.  Zone of hyperaemia: In this outermost zone tissue perfusion is increased. The tissue here will invariably recover unless there is severe sepsis or prolonged hypoperfusion. These three zones of a burn are three dimensional, and loss of tissue in the zone of stasis will lead to the wound deepening as well as widening.

Systemic response

The release of cytokines and other inflammatory mediators at the site of injury has a systemic effect once the burn reaches 30% of total body surface area.

 Cardiovascular changes: Capillary permeability is increased, leading to loss of intravascular proteins and fluids into the interstitial compartment. Peripheral and splanchnic vasoconstriction occurs. Myocardial contractility is decreased, possibly due to release of tumour necrosis factor α. These changes, coupled with fluid loss from the burn wound, result in systemic hypotension and end organ hypoperfusion.

 Respiratory changes: Inflammatory mediators cause bronchoconstriction, and in severe burns adult respiratory distress syndrome can occur.  Metabolic changes: The basal metabolic rate increases up to three times its original rate. This, coupled with splanchnic hypoperfusion, necessitates early and aggressive enteral feeding to decrease catabolism and maintain gut integrity.

 Immunological changes: Non-specific down regulation of the immune response occurs, affecting both cell mediated and humoral pathways.pathways.

Treatment:

Primary Survey:

 Use ABCDEF to check the patient’s status: A. Airway/C spine immobilization B. Breathing and ventilation C. Circulation D. Disability, neurologic deficit E. Expose (remove all clothing and jewelry) Environmental control (keep warm) F. Fluid

Secondary Survey:

o Use head-to-toe approach o Remove all clothing and jewelry o Quickly assess percentage of skin involved and depth of burn o Cover patient with clean, dry sheet o Keep warm: Hypothermia occurs rapidly o Avoid use of ice or ointments o If material is stuck to the skin, do no attempt to remove o For circumferential burns, elevate burn extremity above the level of the heart.

 Fluid therapy:

According to Modified Parkland formula:

Amount of fluid needed in 1st 24 hours = 4 x Percentage of burn x body weight in kg

 So, the amount of Ringer’s lactate to be given = 4 x 30 x 50 = 6000 ml = 6L  Patient should receive 3L of RL in 1st 8 hours and 3L of RL in next 16hours.  Check for urine output, it should be 0.5ml – 1 ml/ kg body weight/hour.  Colloid-containing solutions should be held for intravenous therapy until after the first 24 hours postburn.

 Ensure adequate analgesia: strong opiates should be used.  Prevent hypothermia.  Prevent stress ulcer.  Foley’s catheterisation.

Further management:

 Circulatory insufficiency caused by a circumferentially burned limb is best relieved by escharotomy. Escharotomies are usually not required within the first six hours of burn injury.

 Fasciotomy: seldom required, but may be necessary to restore circulation for patients with associated skeletal trauma, crush injury or burns involving tissue beneath the investing fascia.

 Gastric tube insertion: if there is nausea, vomiting, abdominal distention, or if more than 20% of the total body surface area is burnt.

 Analgesia and sedation:

o Severely burned patients may be restless and anxious from hypoxaemia or hypovolaemia rather than pain. o Intravenous narcotic analgesics and sedatives may be administered in small, frequent doses.  Wound care: o Partial-thickness burns are painful when air currents pass over the burned surface. Gently covering the burn with clean linen relieves the pain and deflects air currents. o Do not break blisters or apply an antiseptic agent. o Any applied medication must be removed before appropriate antibacterial topical agents can be applied. o Application of cold compresses may cause hypothermia. Do not apply cold water to a patient with extensive burns.  Antibiotics: should be reserved for the treatment of infection.  Tetanus: determination of immunisation status is very important.  Full-thickness burns: require excision and grafting unless they are less than 1 cm in diameter. Grafting is required within three weeks in order to minimise scarring. After healing:  The area of healed burns should be moisturised and massaged to reduce dryness.  A high-factor sun cream should be used to prevent further damage and pigmentation changes.

Group – B

2. Answer any of the following questions: a. Classify colonic tumours. How will you manage a 60 years old man presenting with fresh bleeding per rectum. 5+10 = 15

Answer:

Classification of colonic tumours:

A) Epithelial Tumors of the Colon Type Class Subclassification Benign lesions Hyperplastic polyps Noninherited gastrointestinal Hyperplastic polyps polyposis syndromes Hamartomas Juvenile polyps Cowden syndrome Bannayan-Riley-Ruvalcaba syndrome Cronkite-Canada syndrome Inflammatory polyps Potentially malignant Inherited hamartomatous Juvenile polyposis syndromes lesions/syndromes polyposis syndromes Hereditary polyposis Peutz-Jeghers syndrome syndromes Familial adenomatous polyposis (FAP) Attenuated familial adenomatous polyposis (AFAP) Noninherited gastrointestinal Cronkite-Canada syndrome polyposis syndromes Adenomatous polyps Hereditary colon cancers Malignant lesions Epithelial tumors of the colon Sporadic colon cancers Familial colorectal cancer Hereditary nonpolyposis colon cancers (HNPCC) Hereditary polyposis colon cancers A) Epithelial Tumors of the Colon B) Nonepithelial Tumors of the Colon Type Class Benign lesions Lipomas and lipomatous polyposis Potentially malignant Carcinoid lesions/syndromes Gastrointestinal stromal tumors Nodular lymphoid hyperplasia of the colon Malignant lesions Lymphoma C) Secondary Tumors to the Colon Type Class Benign lesions Endometriosis Potentially malignant Leukemia lesions/syndromes Carcinoma arising in endometriosis Malignant lesions Lymphoma Malignant melanoma Carcinomas from other primary sites

See the Question 2.b of Group – B of Paper – I of 2014

Or b) Describe the clinical features, diagnosis and management of choledocholithiasis.

4+4+7 = 15

Answer.

Clinical features:

Symptoms:

 Choledocholithiasis may be asymptomatic or may produce sudden toxic cholangitis, leading to a rapid demise.  Biliary colic, jaundice, or pancreatitis may be isolated findings or may occur in any combination along with signs of infection (cholangitis).  Biliary colic from common duct obstruction cannot be distinguished from that caused by stones in the gallbladder. The pain is felt in the right subcostal region, epigastrium, or even the substernal area. Referred pain to the region of the right scapula is common.  Choledocholithiasis should be strongly suspected if intermittent chills, fever, or jaundice accompanies biliary colic.  Some patients notice transient darkening of their urine during an attack even though jaundice is not evident.  Pruritus is usually the result of persistent, longstanding obstruction. The itching is more intense in warm weather when the patient perspires and is usually worse on the extremities than on the trunk. It is much more common with neoplastic obstruction than with gallstone obstruction.

Signs:

 The patient may be icteric and toxic, with high fever and chills, or may appear to be perfectly healthy.  A palpable gallbladder is unusual in patients with obstructive jaundice from common duct stone because the obstruction is transient and partial, and scarring of the gallbladder renders it inelastic and nondistensible.  Tenderness may be present in the right upper quadrant but is not often as marked as in acute cholecystitis, perforated peptic ulcer, or acute pancreatitis.  Tender hepatic enlargement may occur.

Complications:

 Longstanding ductal infection can produce intrahepatic abscesses.  Hepatic failure or secondary biliary cirrhosis may develop in unrelieved obstruction of long duration. Since the obstruction is usually incomplete and intermittent, cirrhosis develops only after several years in untreated disease.  Acute pancreatitis, a fairly common complication of calculous biliary disease.  Rarely, a stone in the common duct may erode through the ampulla,,resulting in gallstone ileus.  Hemorrhage (hemobilia) is also a rare complication.

Diagnosis:

Laboratory investigations:

 In cholangitis, leukocytosis of 15,000/ L is usual, and values above 20,000/ L are common.  A rise in serum bilirubin often appears within 24 hours after the onset of symptoms. The absolute level usually remains under 10 mg/dL, and most are in the range of 2–4 mg/dL. The direct fraction exceeds the indirect, but the latter becomes elevated in most cases.  Fluctuating jaundice is so characteristic of choledocholithiasis that it fairly reliably differentiates between benign and malignant obstruction.  The serum alkaline phosphatase level usually rises and may be the only chemical abnormality in patients without jaundice.  When the obstruction is relieved, the alkaline phosphatase and bilirubin levels should return to normal within 1–2 weeks.  Mild increases in AST and ALT are often seen with extrahepatic obstruction of the ducts; rarely, AST levels transiently reach 1000 units.  Amylase and lipase: to document pancreatitis.  P-time / INR: should be less than 1.5.

Imaging:

 USG:

 Ultrasonography can document stones in the gallbladder and estimate the diameter of the common bile duct.  A dilated bile duct (>8 mm in diameter) on ultrasonography in a patient with gallstones, jaundice and biliary pain is highly suggestive of choledocholithiasis.  As stones in the distal bile duct slowly move down, bowel gas can preclude their visibility on ultrasound.  The prevalence of choledocholithiasis is significantly higher in the setting of a dilated common bile duct (diameter >5 mm) than in patients with a nondilated duct

 MRCP:

 MRCP provides excellent anatomic detail, with sensitivity and specificity of 95% and 98%, respectively, for common bile duct stones.  It avoids the need for invasive ERCP in more than 50% of patients.  It can be used as a screening test for patients at low or moderate risk for having common duct stones before ERCP.

 ERCP

 It is the diagnostic and potentially therapeutic test of choice for patients with suspected common bile duct stones.  Cannulation of the ampulla of Vater and diagnostic cholangiography are achieved in more than 90% of cases.  Minimal morbidity rates of less than 5% are now achieved in experienced hands and mainly consist of cholangitis and pancreatitis.

 EUS

 Endoscopic ultrasound (EUS) can also be used to identify bile duct stones without cannulation of the ampulla and its associated risks, but it is less sensitive than ERCP. Management: General measures:

 Maintain hydration and nutrion.  Antibiotics: 3rd generation cephalosporine / ciprofloxacin  Injection Vitamin K – 10 mg IM once daily for 3 consecutive days.  If INR is deranged: FFP

Definitive treatment:

. Miminally invasive procedures:

 Endoscopic Cholangiography  The use of endoscopic cholangiography in patients with suspected common bile duct stones not only confirms the diagnosis but also provides ductal clearance of the stones and sphincterotomy before subsequent laparoscopic cholecystectomy.  Patients with worsening cholangitis, ampullary stone impaction, biliary pancreatitis, multiple comorbidities, and cirrhosis are considered good candidates for preoperative endoscopic therapy.  After endoscopic sphincterotomy and stone extraction, patients with gallstones still remain at high risk for developing future biliary complications.  Therefore, prompt cholecystectomy after endoscopic clearance of the common bile duct should be performed during the hospital admission if the patient is fit for surgery.  Endoscopic sphincterotomy is unlikely to be successful in patients with large stones (eg, > 2 cm), and it is contraindicated in the presence of stenosis of the bile duct proximal to the sphincter. Laparotomy and common duct exploration are required in a few cases.

 Laparoscopic Common Bile Duct Exploration  An intraoperative cholangiogram at the time of cholecystectomy will also document the presence of common bile duct stones.  Laparoscopic common bile duct exploration through the cystic duct or with formal choledochotomy allows the stones to be retrieved during the same procedure.

. Open surgery:

 Open Common Bile Duct Exploration  With the increased use of endoscopic, percutaneous, and laparoscopic techniques, open common bile duct exploration is rarely performed today.  It should be performed when a concomitant biliary drainage procedure is indicated.  Open common bile duct exploration is associated with low operative mortality and morbidity.  The rate of retained common bile stones using intraoperative choledochoscopy is less than 5%.  Stones impacted in the ampulla may be difficult for both endoscopic ductal clearance and common bile duct exploration. In these cases, transduodenal sphincteroplasty and stone extraction should be performed. If this is not successful, a choledochoduodenostomy or a Roux-en-Y choledochojejunostomy should be performed.  If the CBD is not dilated and expertise for choledochojejunostomy is lacking- T-tube drainage of common bile duct is done.

 Transduodenal sphincterotomy (TDS) is useful in the management of choledocholithiasis when there is stone impaction in the ampulla of Vater, papillary stenosis, and multiple stones, particularly in the presence of a nondilated bile duct.  Choledochoduodenostomy is indicated in patients with recurrent stones requiring repeated interventions, impacted or giant stones, biliary sludge, and ampullary stenosis. A common bile duct diameter of at least 1.2 cm is important in assessing the feasibility of CDD because this allows a wide enough stoma to ensure good biliary drainage and avert stenosis.

Group – C

3. Write short notes on (any three): 3x5

a) Causes and treatment of metabolic acidosis. b) Venous ulcer. c) Fournier’s gangrene . d) Anorectal malformation. e) Torticollis.

Answer:

a) Causes and treatment of metabolic acidosis.

 Metabolic acidosis results from the accumulation of nonvolatile acids, reduction of renal acid excretion, or loss of alkali. The most common causes of metabolic acidosis are listed in Table 4-6. Metabolic acidosis has few specific signs. The appropriate diagnosis depends on the clinical setting and laboratory tests. o The anion gap (AG; normal = 12 ± 2 mmol/L) represents the anions, other than Cl- and - + HCO3 , that are necessary to counterbalance Na electrically:

+ - AG (mmol/L) = Na (mmol/L) + [Cl (mmol/L) + HCO3 (mmol/L)]

It is useful diagnostically to classify metabolic acidosis into increased or normal AG metabolic acidosis.

Causes of Metabolic Acidosis Increased anion gap Normal anion gap (hyperchloremic) a. Increased acid production a. Renal tubular dysfunction 1. Ketoacidosis 1. Renal tubular acidosis Diabetic 2. Hypoaldosteronism Alcoholic 3. Potassium-sparing Starvation diuretics 2. Lactic acidosis b. Loss of alkali 3.Toxic ingestion (salicylates, 1. Diarrhea ethylene glycol, methanol) 2. Ureterosigmoidostomy 3. Carbonic anhydrase b. Renal failure inhibitors c. Administration of HCl (ammonium chloride, cationic amino acids)

o Treatment of metabolic acidosis must be directed primarily at the underlying cause of the acid-base disturbance. Bicarbonate therapy should be considered in patients with moderate to - severe metabolic acidosis only after the primary cause has been addressed. The HCO3 deficit (mmol/L) can be estimated using the following equation:

- - HCO3 deficit (mmol/L) = body weight (kg) × 0.4 × [(desired HCO3 [mmol/L]) - (measured - HCO3 [mmol/L])]

- This equation serves to provide only a rough estimate of the deficit because the volume of HCO3 distribution and the rate of ongoing H+ production are variable. b) Venous ulcer. See the Question 2.d of Group –B of Supplementary Paper – II of 2009. c) Fournier’s gangrene.

Definition:Fournier gangrene is defined as a polymicrobial necrotizing fasciitis of the perineal, perianal, or genital areas.

Aetiology ansd risk factors:

 Necrotizing process commonly originates from infection in anorectum, urogenital tract or skin of genitalia.

Aetiology:

Ano-rectal causes Uro-genital causes Dermatologic causes Other less common causes Infection in the Infection in the Hidradenitis Consequence of bone perineal glands bulbourethral glands suppurativa marrow malignancy

Manifestation of Urethral injury Ulceration from Systemic lupus colorectal injury, scrotal pressure erythematosus malignancy or Iatrogenic injury diverticulitis Trauma to scrotum or Crohn’s diseases Lower urinary tract perineum infections

Risk factors:

Diabetes mellitus Malignancies Chronic steroid use Malnutrition Alcoholism Cirrhosis Liver HIV infection Morbid Obesity

Causative bacteria:

• Polymicrobial infection

• Minimum of four isolates per case

• Most common aerobe – E. coli

• Most common anaerobes – Bacteroids

• Others – Streptococcus, Staphylococcus, MRSA – Methicillin Resistant Staphylococcus aureus, Klebsiella Pseudomonas, Proteus & Clostridium.

Pathogenesis:

• Bacteria act synergistically causing obliterative endarteritis & production of various enzymes causing destruction

• There is imbalance between host immunity & virulence of organism Mechanism of spread Entry of bacteria (act through synergism) Fibrinoid coagulation of nutrient vessels

Decreased locally blood supply to skin Decreased tissue oxygen tension Growth of anaerobes & microaerophilic organisms Production of enzyme (Collagenase, Lecithinase, Hyaluronidase ) Digestion of fascial barrier Rapid spread of infection

Pathology:

Pathognomonic findings on pathological evaluation of tissue are:-

• Necrosis of superficial & deep fascial planes

• Fibrinoid coagulation of the nutrient arterioles

• Polymorphonuclear cell infiltration

• Presence of micro organisms within the involved tissues

• Air in the perineal tissue.

Incidence:

• Age : 30 – 60 years

• Sex : 10 times more common in males

• Social habits:More common in males homosexuals (more prone for Rectal injury)

Clinical features:

• Begins with insidious onset of pruritus and discomfort of external genitalia • Prodromal symptoms of fever and lethargy, which may be present for 2-7 days before gangrene

• The hallmark of Fournier gangrene is out of proportion pain and tenderness in the genitalia.

• Increasing genital pain and tenderness with progressive erythema of the overlying skin

• Dusky appearance of the overlying skin; subcutaneous crepitation; feculent odor

• Obvious gangrene of a portion of the genitalia; purulent discharge from wounds

• As gangrene develops, pain subsides (Nerve necrosis)

Investigations:

 (CBC) Complete blood count

 Electrolytes

 BUN / Serum creatinine

 Blood Sugar

 ABG

 Blood and urine culture with sensitivity

 Coagulation profile for DIC

 Imaging-

o Conventional radiography - Presence of gas in soft tissue o Ultrasonography -  Presence of gas in scrotal tissue  Excludes other conditions  Testicular blood flow - Normal  Limitations – Direct pressure on involved tissue causes inconvenience o C.T. Scanning –  Can detect smaller amount of soft tissue gas  Defines extent more specifically  Identifies underlying causes eg. Small perineal abscess

o MRI –  Yields greater soft tissue details  Create logistic challenges, especially in critically ill patients Medical treatment:

1. Restoration of normal organ perfusion

2. Reduction of systemic toxicity

3. Broad spectrum antibiotics to cover anaerobes as well (cipro+clinda+metro)

4. Vancomycin for MRSA

5. Tetanus prophylaxis

6. Irrigation with super oxidised water

7. Hyperbaric oxygen therapy

8. IV immunoglobulins to neutralize super antigen as streptotoxin A & B (as adjuvant)

9. Antifungal – if required

10. Non – conventional - dressing with gauge soaked with zinc per oxide

Surgical treatment:

• Repeated aggressive debridement

• Preservation of testes (subcutaneous pocket from desiccation)

• Reconstruction after infection is over

• Fecal diversion

• Urinary diversion

• Vacuum assisted closure (VAC)

Complications:

ARF Septicemia and gram negative shock Tetanus ARDS MSOF Death

d) Anorectal malformation. See the Question 3.a of Group –B of Paper – II of 2009. e) Torticollis.

Definition:

Torticollis is a dystonic condition defined by an abnormal, asymmetrical head or neck position, which may be due to a variety of causes.

Signs and symptoms:

Torticollis is a fixed or dynamic tilt, rotation, or flexion of the head and/or neck. The type of torticollis can be described depending on the positions of head and neck.

 Laterocollis : the head is tipped toward the shoulder  Rotational torticollis : the head rotates along the longitudal axis  Anterocollis : forward flexion of the head and neck  Retrocollis : hyperextension of head and neck backward

A combination of these movements may often be observed. Congenital muscular torticollis:  The cause of congenital muscular torticollis is unclear.  Birth trauma or intrauterine malposition is considered to be the cause of damage to the sternocleidomastoid muscle in the neck.  This results in a shortening or excessive contraction of the sternocleidomastoid muscle, which curtails its range of motion in both rotation and lateral bending. The head typically is tilted in lateral bending toward the affected muscle and rotated toward the opposite side.  Sometimes a mass, such as a sternocleidomastoid tumor, is noted in the affected muscle at the age of two to four weeks. Gradually it disappears, usually by the age of eight months, but the muscle is left fibrotic.  Initially, the condition is treated with physical therapies, such as stretching to release tightness, strengthening exercises to improve muscular balance, and handling to stimulate symmetry.  A TOT Collar is sometimes applied. About 5–10% of cases fail to respond to stretching and require surgical release of the muscle. Acquired torticollis Noncongenital muscular torticollis may result from scarring or disease of cervical vertebrae, adenitis, tonsillitis, rheumatism, enlarged cervical glands, retropharyngeal abscess, or cerebellar tumors. It may be spasmodic (clonic) or permanent (tonic). The latter type may be due to Pott's Disease (tuberculosis of the spine).

 A self-limiting spontaneously occurring form of torticollis with one or more painful neck muscles is by far the most common ('stiff neck') and will pass spontaneously in 1–4 weeks. Usually the sternocleidomastoid muscle or thetrapezius muscle is involved. Sometimes draughts, colds, or unusual postures are implicated; however in many cases no clear cause is found. These episodes are commonly seen by physicians.  Tumors of the skull base (posterior fossa tumors) can compress the nerve supply to the neck and cause torticollis, and these problems must be treated surgically.  Infections in the posterior can irritate the nerves supplying the neck muscles and cause torticollis, and these infections may be treated with antibiotics if they are not too severe, but could require surgical debridement in intractable cases.  Ear infections and surgical removal of the adenoids can cause an entity known as Grisel's syndrome, a subluxation of the upper cervical joints, mostly the atlantoaxial joint, due to inflammatory laxity of the ligaments caused by an infection. This bridge must either be broken through manipulation of the neck, or, surgically resected.  The use of certain drugs, such as antipsychotics, can cause torticollis.  Antiemetics - Neuroleptic Class - Phenothiazines  There are many other rare causes of torticollis. A very rare cause of acquired torticollis is fibrodysplasia ossificans progressiva (FOP), the hallmark of which is malformed great toes. Spasmodic torticollis:  Torticollis with recurrent, but transient contraction of the muscles of the neck and especially of the sternocleidomastoid.  Synonyms are "intermittent torticollis", "cervical dystonia" or "idiopathic cervical dystonia", depending on cause. Acquired torticollis in infants:  Infants often develop torticollis as a result of the amount of time they spend lying on their back during the day in car seats, swings, bouncers, strollers, and on play mats.  Infants with torticollis have a higher risk of plagiocephaly or flat head syndrome.  Most pediatricians recommend regular repositioning of a baby for healthy head and neck movement.  Torticollis is almost always preventable in infants.

Group – D

3. Write briefly on (any three): 3x5

a) Volkman’s ischaemic contracture. b) Ewing’s tumour. c) Core needle biopsy. d) Colle’s fracture. e) Bone graft.

Answer.

a) Volkman’s ischaemic contracture.

Definition:

Volkmann's ischaemic contracture is a permanent flexion contracture of the hand at the wrist, resulting in a claw-like deformity of the hand and fingers. It is more common in children. Passive extension of fingers is restricted and painful.

On examination the fingers are white or blue and cold and the radial pulse is absent.

Causes:  Any fracture in elbow region or upper arm may lead to Volkmann's ischemic contracture but commonly caused due to supracondylar fracture of the humerus.  Volkmann's contracture results from acute ischaemia/necrosis of the muscle fibres of the flexor group of muscles of the forearm, especially M. flexor digitorum profundus and M. flexor pollicis longus which becomes fibrotic and short.  It is caused by obstruction on the brachial artery near the elbow, possibly from improper use of a tourniquet, improper use of a plaster cast, or compartment syndrome.  It is also caused by fracture of forearm bones which cause profuse bleeding from major blood vessels of forearm. Classification:  Holden Classification: o Level I—Injury is proximal to the ischemia and later contracture, as in a brachial artery injury o Level II—Ischemia is directly under the injury (pressure)  Tsuge classification is based on the three levels of severity of Volkmann’s Contracture and is outlined below: o Mild—resulting contracture of 2-3 fingers only o Moderate—all fingers are flexed with thumb flexed in palm, wrist in flexion and partial loss of sensation in the hand o Severe—all muscles that flex and extend wrist and fingers are involved. Treatment:  Surgery to release the fixed tissues may help with the deformity and function of the hand.  All splints, plaster and bandages that might be obstructing the circulation should be removed and fascial compartment should be opened (fasciotomy) in the incipient stage. Prevention:

 Requires restoration of blood flow;  Reduction of compartmental pressure Management:

 Proper initial splinting of hand in the function position;  Release of forearm flexors:  Muscle slide  Tendon lengthening.  Test for intrinsic tightness  Free functional muscle transfer

b) Ewing’s tumour. See the Question 4.e of Group –D of Supplementary Paper – I of 2014 c) Core needle biopsy. See the Question 3.d of Group – C of Supplementary Paper – I of 2013 d) Colle’s fracture.

Definition : - It is fracture lower end of radius with a fracture dislocation of the inferior radioulnar joint . Occurs about 2.5 cm above the carpal extremity of the radius .

Presentation:

 Commonest age group- Elderly.( 60 yrs)  Women> Men.

MOA – fall in outstretched hand.

Force required to cause this fracture is 192 kg in women and 282 kg in men.

Clinical features:

• Swelling.

• Pain.

• Dinner fork defomity, it is not found in all cases but seen only if there is a dorsal tilt or rotation of the distal fragment

Examination:

 Distal neurovascular status.  External injuries. Frykmann`s classification :

Fracture line Distal ulnar fracture

Absent Present 1 .. Extra - articular 1 2 2. Intra – articular (involving RC 3 4 joint only ) 3 . Intra – articular (involving distal 5 6 RU joint only ) 8 4 . Intra – articular (both RC + 7 inferior RU joints ) RC = radiocarpal RU = radioulnar

Frykmann`s classification • Fig :

Radiography:

X – Ray of the wrist : -

• AP and lateral views and lower end of the radius

Displacement in a colle`s fracture:

• Dorsal displacement

• Dorsal rotation

• Lateral displacement

• Lateral rotation

• Impaction

• supination

Treatment:

• Conservative methods

• Operative methods

Conservative methods:

 Closed reduction under general anaesthesia (GA),or local anaesthesia (LA).  If the level of the styloid processes are restored back to normal , it indicates that the reduction has been achieved satisfactorily.  Limb is immobilised by colle`s cast and a check radiograph is taken  Removed after 6 – 8 weeks.  Physiotherapy

 Acceptable limits of colle`s fracture:

• A dorsal tilt of less than 10 degrees

• A radial shorteing of less than 5 mm.

 Operative methods: Open reduction and internal fixation.

Indication:

• Impaction

• Median nerve entrapment

e) Bone graft.

Definition:

A bone graft is a surgical procedure used to fix problems associated with bones or joints. Bone grafting or transplanting of bone tissue is beneficial in fixing bones after trauma, problem joints, or growing bone around implanted devices, such as total knee replacement.

Biological mechanism:

Osteoconduction: Osteoconduction occurs when the bone graft material serves as a scaffold for new bone growth that is perpetuated by the native bone. Osteoblasts from the margin of the defect that is being grafted utilize the bone graft material as a framework upon which to spread and generate new bone. In the very least, a bone graft material should be osteoconductive. Osteoinduction: Osteoinduction involves the stimulation of osteoprogenitor cells to differentiate into osteoblasts that then begin new bone formation. The most widely studied type of osteoinductive cell mediators are bone morphogenetic proteins (BMPs). A bone graft material that is osteoconductive and osteoinductive will not only serve as a scaffold for currently existing osteoblasts but will also trigger the formation of new osteoblasts, theoretically promoting faster integration of the graft. Osteopromotion: Osteopromotion involves the enhancement of osteoinduction without the possession of osteoinductive properties. For example, enamel matrix derivative has been shown to enhance the osteoinductive effect of demineralized freeze dried bone allograft (DFDBA), but will not stimulate de novo bone growth alone. Osteogenesis: Osteogenesis occurs when vital osteoblasts originating from the bone graft material contribute to new bone growth along with bone growth generated via the other two mechanisms.

Bone graft and graft substitutes:

Uses:  Fractures—a bone graft may be used in the case of multiple or complex fractures or those that do not heal well after an initial treatment  Fusion—most often done in the spine, fusion helps two bones heal together across a diseased joint  Regeneration—used for bone lost to disease, infection, or injury, this can involve using small amounts in bone cavities or large sections of bones  Implanted devices—a graft can be used to help bone heal around surgically implanted devices, like joint replacements, plates, or screws.

Risks:

 Pain  Nerve injury  Rejection of the bone graft  Inflammation

The West Bengal University of Health Sciences

M.B.B.S. 3rd Professional Part – II Examination, 2012

1 Subject: Surgery Time: 2 /2 hrs.

Paper: I Marks: 60

Group –A

1. What are the types of haemorrhage? What are the methods of determining acute blood loss? How would you treat haemorrhage? 3+6+6 = 15

Answer.

Types of haemorrgage:

Recognition of types of haemorrhage:

Arterial/ venous/capillary:

 Arterial haemorrhage: Arterial haemorrhage is recognised as bright red blood, spurting as a jet which rises and falls in time with the pulse. In protracted bleeding, and when quantities of intravenous fluids other than blood are given, it can become watery in appearance.  Venous haemorrhage: Venous haemorrhage is a darker red, a steady and copious flow. The colour darkens still further from excessive oxygen desaturation when blood loss is severe, or in respiratory depression or obstruction. Blood loss is particularly rapid when large veins are opened, e.g. common femoral or jugular. Venous bleeding can be under increased pressure as in asphyxia, or from ruptured varicose veins  Capillary haemorrhage: Capillary haemorrhage is bright red, often rapid, ooze. If continuing for many hours, blood loss can become serious, as in haemophilia.

Primary/reactionary/secondary:

 Primary haemorrhage: Primary haemorrhage occurs at the time of injury or operation. Reactionary haemorrhage  Reactionary haemorrhage: may follow primary haemorrhage within 24 hours (usually 4—6 hours) and is mainly due to rolling (‘slipping’) of a ligature, dislodgement of a clot or cessation of reflex vasospasm.  Secondary haemorrhage: Secondary haemorrhage occurs after 7—14 days, and is due to infection and sloughing of part of the wall of an artery. Predisposing factors are pressure of a drainage tube, a fragment of bone, a ligature in an infected area or cancer. It is also a complication of arterial surgery and amputations External/internal:

 External haemorrhage: External haemorrhage is visible, revealed haemorrhage.  Internal haemorrhage: Internal haemorrhage is invisible, concealed haemorrhage. Internal bleeding may be concealed as in ruptured spleen or liver, fractured femur, ruptured ectopic gestation or in cerebral haemorrhage. Concealed haemorrhage may become revealed as in haematemesis or melaena from a bleeding peptic ulcer, as in haematuria from a ruptured kidney, or via the vagina in accidental uterine haemorrhage of pregnancy.

Classes of haemorrhage

Methods of determining acute blood loss:

Assessment and management of blood loss must be related to the pre-existing circulating blood volume, which can be derived from the patient’s weight: • Infant 80—85 ml/kg; • Adult 65—75 ml/kg. Measuring blood loss • Blood clot: The size of a clenched fist is roughly equal to 500 ml. • Swelling in closed fractures: Moderate swelling in closed fracture of the tibia equals 500— 1500 ml blood loss. Moderate swelling in a fractured shaft of femur equals 500—2000 ml blood loss. • Swab weighing. In the operating theatre, blood loss can be measured by weighing the swabs after use and subtracting the dry weight. The resulting total obtained (1 g = 1 ml) is added to the volume of blood collected in the suction or drainage bottles. In extensive wounds and operations, the blood loss is grossly underestimated, due to evaporation of water from the swabs before weighing each batch.Prompt transfer of discarded swabs into polythene bags reduces this source of error. Blood, plasma and water are also lost from the vascular system because of evaporation from open wounds, into the tissues, sweating and expired water via the lungs. Indeed, for operations such as radical mastectomy or partial gastrectomy it may be necessary to multiply the swab weighing total by a factor of 1.5. For prolonged surgery via larger wounds, as in abdominothoracic or abdominoperineal operations, the total measured may need to be multiplied by 2.  Haemoglobin level This is estimated in g/100 ml (g/dl), normal values being 12—16 g/100 ml (12—16 g/dl). There is no immediate change in haemorrhage, but after some hours the level falls by influx of interstitial fluid info the vascular compartment in order to restore the blood volume.  Measurement of central venous pressure  Continuous tissue oxygen tension measurement

Managing internal bleeding:

ABC’s

High concentration oxygen

Assist ventilations

Control external bleeding

Stabilize fractures

RICE – resuscitation, investigations, clinical examination, evaluation

Transport rapidly to appropriate facility

Control of external bleeding:

Pressure Dressing: Use bandage to secure dressing in place

Tourniquets:

o Final resort when all else fails o Used for amputations o 3-4” wide (blood pressure cuffs) o Write “TK” and time of application on forehead of patient o Notify other personnel o Once applied, DO NOT REMOVE

Treatment of haemorrgagic shock:

The primary treatment of hemorrhagic shock is to control the source of bleeding as soon as possible and to replace fluid. o In controlled hemorrhagic shock (CHS), where the source of bleeding has been occluded, fluid replacement is aimed toward normalization of hemodynamic parameters. In uncontrolled hemorrhagic shock (UCHS), in which the bleeding has temporarily stopped because of hypotension, vasoconstriction, and clot formation, fluid treatment is aimed at restoration of radial pulse or restoration of sensorium or obtaining a blood pressure of 80 mm Hg by aliquots of 250 mL of lactated Ringer's solution (hypotensive resuscitation). o When evacuation time is shorter than 1 hour (usually urban trauma), immediate evacuation to a surgical facility is indicated after airway and breathing (A, B) have been secured ("scoop and run"). Precious time is not wasted by introducing an intravenous line. When expected evacuation time exceeds 1 hour, an intravenous line is introduced and fluid treatment is started before evacuation. The resuscitation should occur before, or concurrently with, any diagnostic studies. o Crystalloid is the first fluid of choice for resuscitation. Immediately administer 2 L of isotonic sodium chloride solution or lactated Ringer’s solution in response to shock from blood loss. Fluid administration should continue until the patient's hemodynamics become stabilized. Because crystalloids quickly leak from the vascular space, each liter of fluid expands the blood volume by 20-30%; therefore, 3 L of fluid need to be administered to raise the intravascular volume by 1 L. o Alternatively, colloids restore volume in a 1:1 ratio. Currently available colloids include human albumin, hydroxy-ethyl starch products (mixed in either 0.9% isotonic sodium chloride solution or lactated Ringer’s solution), or hypertonic saline-dextran combinations. The sole product that is avoided routinely in large-volume (>1500 mL/d) restoration is the hydroxy-ethyl starch product mixed in 0.9% isotonic sodium chloride solution because it has been associated with the induction of coagulopathy. The other products have not been so implicated. o In patients with hemorrhagic shock, hypertonic saline has the theoretical benefit of increasing intravascular volume with only small amounts of fluid. The combination of dextran and hypertonic saline may be beneficial in situations where infusion of large volumes of fluid may be harmful, such as in elderly persons with impaired cardiac activity. Additional trials will be required before this combination is accepted as standard of care. o PRBCs should be transfused if the patient remains unstable after 2000 mL of crystalloid resuscitation. For acute situations, O-negative noncrossmatched blood should be administered. Administer 2 U rapidly, and note the response. For patients with active bleeding, several units of blood may be necessary. o There are recognized risks associated with the transfusion of large quantities of PRBCs. As a result, other modalities are being investigated. One such modality is hemoglobin- based oxygen carriers (HBOC). Clinical application has been limited by its toxic effect profile. o If at all possible, blood and crystalloid infusions should be delivered through a fluid warmer. A blood sample for type and cross should be drawn, preferably before blood transfusions are begun. Start type-specific blood when available. Patients who require large amounts of transfusion inevitably will become coagulopathic. FFP generally is infused when the patient shows signs of coagulopathy, usually after 6-8 U of PRBCs. Platelets become depleted with large blood transfusions. Platelet transfusion is also recommended when a coagulopathy develops.

Group – B

2. Answer any of the following questions: a) Middle aged patient presented with a big tense lump in the upper abdomen following an attack of acute abdomen. How would you investigate the patient and plan the management? 8+7 = 15

Answer.

The provisional diagnosis according to me is pseudocyst pancreas after an attack of acute pancreatitis: Differential diagnosis:  Gastric outlet obstruction due to peptic ulcer disease or gastric carcinoma  Hepatocellular carcinoma  Empyema/mucocele gall bladder

From history and clinical examination:

Symptoms: Signs:

o Abdominal pain > 3 weeks (80 – 90%) o Tenderness o Nausea / vomiting o Abdominal fullness o Early satiety o Bloating, indigestion

 Clinically suspect a pseudocyst o Episode of pancreatitis fails to resolve o Amylase levels persistantly high o Persistant abdominal pain o Epigastric mass palpated after pancreatitis

Investigation:

USG:

It will be able to tell about the organ involved

Upper GI endoscopy:

It will rule out any gastric neoplasm or peptic ulcer disease.

Serum amylase:

Serum amylase level will be persistently high.

Lipase:

Serum lipase level may be high also.

Management after diagnosis:

 Computerized tomography: This is the gold standard for initial assessment and follow-up Large Pancreatic Pseudocyst  Magnetic resonance cholangiopancreatography (MRCP): to establish the relationship of the pseudocyst to the pancreatic ducts.

 50% resolve spontaneously

Treatment: The principal indications for treating pancreatic pseudocysts are to relieve symptoms and to prevent complications. In the absence of symptoms or evidence of enlargement, expectant management usually is reasonable. An enlarging asymptomatic pseudocyst that has been present for 6 weeks usually is treated. This relatively conservative approach is based on the low risk of complications. A natural- history study from India indicates that asymptomatic pseudocysts less than 7.5 cm in diameter and without internal debris will resolve spontaneously at an average of 5 months. Complications:  Infection  S/S – Fever, worsening abd pain, systemic signs of sepsis  CT – Thickening of fibrous wall or air within the cavity  GI obstruction  Perforation  Hemorrhage  Thrombosis – SV (most common)  Pseudoaneurysm formation – Splenic artery (most common), GDA, PDA

Indications for drainage Intervention

 Presence of symptoms (> 6 wks)  Percutaneous drainage  Enlargement of pseudocyst ( > 6 cm)  Endoscopic drainage  Complications  Surgical drainage  Suspicion of malignancy

Percutaneous drainage:

 Continuous drainage until output < 50 ml/day + amylase activity ↓  Failure rate 16%  Recurrence rates 7%  Complications  Conversion into an infected pseudocyst (10%)  Catheter-site cellulitis  Damage to adjacent organs  Pancreatico-cutaneous fistula  GI hemorrhage

Endoscopic drainage:  Transenteric drainage

 Cystogastrostomy

 Cystoduodenostomy

 Transpapillary drainage

 40-70% of pseudocysts communicate with pancreatic duct  ERCP with sphincterotomy, balloon dilatation of pancreatic duct strictures, and stent placement beyond strictures

Surgical options:

 Excision

 Tail of gland & along with proximal strictures – distal pancreatectomy & splenectomy

 Head of gland with strictures of pancreatic or bile ducts – pancreaticoduodenectomy

 External drainage

 Internal drainage

 Cystogastrostomy

 Cystojejunostomy

 Permanent resolution confirmed in between 91%–97% of patients

 Cystoduodenostomy

 Can be complicated by duodenal fistula and bleeding at anastomotic site

or b) What are the causes of benign biliary stricture? Discuss management of retained stone in common bile duct. 5+10 =15

Answer.

Causes:

 Postoperative injury after cholecystectomy  Pancreatitis  Mirizzi syndrome  Primary sclerosing cholangitis  Choledochal cysts  Recurrent pyogenic cholangitis  Orthotopic liver transplantation (OLT)  Radiation  Blunt abdominal trauma  Inflammatory strictures  Endoscope-related strictures  HIV cholangiopathy  Polyarteritis nodosa and systemic lupus erythematosus (SLE)  Tuberculosis and histoplasmosis  Chemotherapeutic drugs: Hepatic artery infusion of 5-fluorodeoxyuridine (FdUrd, FUDR) or other chemotherapeutic drugs may cause bile duct strictures. Sphincter of Oddi dysfunction or papillary stenosis  Idiopathic  Miscellaneous: Strictures have been described in association with duodenal diverticulum, Crohn disease, hepatic artery aneurysm, cystic fibrosis with liver involvement, eosinophilic cholecystitis, and cholangitis.

Management of retained CBD stone:

Preoperative diagnosis:  Blood tests (elevated LFT’s)  Abdominal U/S -15-30% sensitivity, If CBD >10mm90%  EUS - Sensitivity and specificity 92-100%  MRCP - 90% sensitive, 100% specificity  ERCP: Diagnostic and therapeutic. Endoscope into 2nd portion of duodenum Papilla visualized & cannulated – Radioopaque dye injected under fluroscopy – Stones appear as filling defects Performed in conjunction with sphincterotomy and stone extraction

Various options are available to treat patients with retained common bile duct stones. Management of an individual patient depends upon the expertise and facilities available, the age and general health of the patient, size of retained stones and whether a T-tube is in place. They are: a) Mechanical nonoperative extraction b) Chemical dissolution c) Endoscopic sphincterotomy d) Surgical reoperation  Retained stones in the common bile duct can be removed if the T- tube is insitu, by saline or heparinized saline flushed down the T-tube. This method is indicated if the stones are small and distal to the T-tube and the sphincter of Oddi is relaxed with glycerol trinitrate or glucagon.  Extraction of stones via the T-tube tract using steerable catheters is claimed to be the procedure of choice in a selected group of patients who have a T-tube. This procedure is done as a day case in the radiology department after the T- tube has been in place for 4 weeks or more and the tract has ‘matured’. No fasting or premedication are necessary.  ERCP/sphincterotomy is also a valuable method of treatment, however, it should be avoided in patients who have had recent surgery and aT-tube is in situ.  Surgical re-exploration should be avoided if these non-operative modalities are available as it has a higher morbidity and mortality, however, if the stones are large or when complications arise as a result of ERCP/sphincterotomy, it may become necessary.

Group – C

3. Write short notes on (any two): 2x5 = 10

a) Postoperative pain management. b) Creating pneumo- peritoneum in Lap.Surgery (Procedures only). c) Burst abdomen. d) Decubitus ulcer.

Answer. a) Postoperative pain management.

Clinical definition of pain:

An unpleasant sensory and emotional experience associated with actual or potential tissue damage, or described in terms of such damage.

Methods of post operative pain relief:

 Simple analgesics: NSAIDS, Opioid analgesics, Combinations of the two  LA / Regional blocks  PCA (patient control analgesia)

Routes of administration:

 Oral / Rectal (suppository)  IM – erratic effect, low patient satisfaction  Subcutaneous-reserved for terminally ill patients  Transdermal- chronic pain  Intravenous- paracetamol, diclofenac,opioids  Intrathecal / Epidural

Post operative pain relief:

 Effective analgesia is an essential part of postoperative management.  Important injectable drugs for pain are the opiate analgesics  Nonsteroidal anti-inflammatory drugs (NSAIDs), such as diclofenac (1 mg/kg) and ibuprofen can also be given orally and rectally, as can paracetamol (15 mg/kg).  There are three situations where an opiate might be given: preoperatively, intra-operatively, post-operatively. Opiate premedication is rarely indicated, although an injured patient in pain may have been given an opiate before coming to the operating room. Opiates given pre- or intraoperatively have important effects in the postoperative period since there may be delayed recovery and respiratory depression, even necessitating mechanical ventilation. Naloxone antagonizes (reverses) all opiates, but its effect quickly wears off.  Patient Controlled Analgesia:  Patients titrate their analgesic requirements against sedation & other side effects of opioids  Made possible with the availability of programmable pumps capable of delivering accurate doses of opioid  Patient controls the analgesia – no delay in administration  Drug is delivered I.V. – erratic absorption is bypassed  Lower incidence of side effects & improved patient satisfaction  Ideal drug for PCA – Rapid onset of action – High analgesic efficacy with no ceiling effect – Intermediate duration of effect for better controllability:

Morphine & Pethidene satisfy most criteria o Background infusion can be incorporated into the prescription regimen o Results in higher trough levels of opioid- reduction in movement associated pain o Risk of respiratory depression, sedation and other side effects are higher.  Local/regional anesthesia:  Local infiltration into incision site  Block of single nerve / plexus  Bupivacaine (0.5%) is most commonly employed  Post op analgesia for 4 - 6 hours  Reduces the analgesic requirements in the first few hours  Patient should be instructed to protect the limb/area as it will be numb – to prevent inadvertant injury to the limb. b) Creating pneumo - peritoneum in Lap.Surgery (Procedures only).

 Laparoscopy is endoscopic visualization of the peritoneal cavity usually assisted by a pneumoperitoneum that distends and separates the abdominal wall from its contents. Visual clarity, space to perform diagnostic and therapeutic procedures and maintenance of a normal physiologic state is required for safe effective surgery. To perform laparoscopic procedures the abdominal cavity is inflated with gas to create the pneumoperitoneum.  Factors that determine the most appropriate gas for pneumoperitoneum are type of anesthesia, physiologic compatibility, toxicity, ease of use, safety, delivery method, cost, and non-combustibility. Gases used for pneumoperitoneum include carbon dioxide (CO2), air, oxygen, nitrous oxide (N2O), argon, helium and mixtures of these gases.  CO2 gas insufflation is preferred by most laparoscopists because it has a high diffusion coefficient and is a normal metabolic end product rapidly cleared from the body. Also, CO2 is highly soluble in blood and tissues and does not support combustion. The risk of gas embolism is lowest with CO2. Cardiac arrhythmias can occur with CO2 pneumoperitoneum because of possible CO2 induced hypercarbia, N2O may be preferred in patients with cardiac disease. With prolonged procedures, CO2 retention is possible as evidenced by tachycardia and acidosis.  Pneumoperitoneum is usually initiated by use of a needle (Veress or Tuohy) or trocar device to transverse the abdominal wall and distend the peritoneal cavity. Another method of access is with an open incision and entering by direct vision through the peritoneum. Caution is required with any method of abdominal entrance or distention. Abdominal penetration complications and incorrect gas placement may result in bleeding or gas dissection within the abdominal wall. Bowel injury, puncture of intra-abdominal vessels, dissection of the fascia or omentum can occur.  After peritoneal access, a gas delivery system is used to inflate and maintain the abdominal distention. Preset pressures of 15 mm Hg or less are safest to maintain pneumoperitoneum and allow performance of laparoscopic techniques. Intra-abdominal pressures in excess of 25 mm Hg are associated with increased airway pressure, increased intrathoracic pressure, increased femoral venous pressure and signs of cardiovascular stimulation with tachycardia and hypertension.Large patients and those who have had multiple abdominal surgeries present a challenge to establish a pneumoperitoneum. Patient selection for laparoscopic procedures and surgical judgment concerning the appropriateness of laparoscopic versus open surgery should be individualized for each circumstance.  Gas delivery systems are composed of a containment cylinder, insufflator (gas throttling down pressure regulating unit), tubing, filter and abdominal entry device or port. .  The temperature of carbon dioxide gas is about 20.1° C as it enters the abdomen. The cool gas causes hypothermia if the gas is not pre-conditioned. Gas flow also contributes to hypothermia by convection effects. There is enhanced evaporation from the bowel surface due to gas turbulence from pressurized delivery. Additionally, general anesthesia causes patients to be unable to maintain thermal stability. The net effect is a loss of 0.3 degree C per 60 liters of gas insufflated. In addition, hypothermia may cause decreased gastrointestinal motility and lead to increased potential for ileus. When the laparoscope is first introduced into the abdominal cavity lens fogging often occurs. This phenomenon is due to the relatively cold dry lens being introduced into a warm moist environment causing the dew point to be reached. This results in condensation forming on the internal lens surface. When the insufflation gas is heated and hydrated or a surface wetting agent is used, no lens fogging occurs and the visual field is clear.  The gases used for pneumoperitoneum have low water content. CO2 has less than 200 parts per million of water. Dry insufflation gases cause drying of the peritoneum and result in intact mesothelial cells being lost or desiccated from the peritoneum surface. To preserve peritoneal surface integrity and decrease the tendency to adhesion formation continuous or intermittent moistening should be performed.  Initial abdominal entry pressure readings should be low—less than 2-3 mm Hg. Elevated initial pressures indicate improper placement. Increased intra-abdominal pressures after proper access can impede venous return and result in potential anesthesia complications. Pressure on intra-abdominal surfaces due to the pneumoperitoneum can inhibit bleeding giving a false sense of security regarding hemostasis. Prior to concluding any procedure, surgical sites need to be observed with reduced pressure to assure appropriate hemostasis.  The seemingly inactive invisible pneumoperitoneum is not a static condition and must not be ignored in laparoscopic surgery. The pneumoperitoneum is a dynamic space that affects the patient’s general well being, and specific physiologic cellular processes. The insufflation gas needs to be filtered to reduce contamination, heated to reduce hypothermia and hydrated to preserve cellular integrity and reduce adhesion formation. It is important to recognize the affects of intra-abdominal therapy and the consequences of surgical devices. This includes tissue particles, aerosol production, the by-products of combustion, and their effect on peritoneal tissues locally and the body chemistry and metabolism as a whole. c) Burst abdomen.

Introduction:

 It is the disruption of an abdominal wound, occurring usually between the 6th and 8th days after an operation.  Usually sutures opposing the deep layers, i.e.. Peritoneum and rectus sheath tear through causing burst abdomen.

Clinical features:

 A sudden feeling of giving away from the wound – on the 6th to 8th postoperative day often precipitated by bouts of severe cough.  Pinkish serosanguinous discharge from the wound.  Often omentum or coils of intestine are forced out of the wound.  Pain and shock is often present.  Clinically burst abdomen can be diagnosed without fail.

Factors related to burst abdomen:

• Choice of suture materials used.

• Method of closure: Continuous sutures more likely to disrupt than interrupted sutures.

• Midline and vertical wounds are more likely to disrupt than transverse. • Surgical wounds of peritonitis, acute abdomen, major surgeries like pancreatic, hepatic, gastric, surgeries for malignancies have a high incidence of disruption.

• Severe cough, vomiting and distension in early post-operative period.

• Poor general condition of patient – Anemia, jaundice, hypoproteinemia, obesity, uremia and diabetes mellitus.

Treatment:

 Nasogastric aspiration

 IV fluids

 Emergency surgery

Surgery:

 Each protruding coil of intestine is gently washed with saline solution and returned to the abdominal cavity.  Protruding greater omentum treated similarly and spread over the intestine.  Having cleansed the abdominal wall all layers are approximated by through and through sutures of monofilament nylon, which may be passed through through a soft rubber or plastic tube collar.  The abdominal wall may be supported by strips of adhesive plaster encircling the anterior two thirds of the circumference of the trunk.  Antibiotic therapy is started.  Wound usually heals well without second dehiscence. Late problem, maybe development of incisional hernia. d) Decubitus ulcer.

Also known as bed sore.

Bedsores — also called pressure sores or pressure ulcers — are injuries to skin and underlying tissue resulting from prolonged pressure on the skin. Bedsores most often develop on skin that covers bony areas of the body, such as the heels, ankles, hips and tailbone.

Bedsores fall into one of four stages based on their severity.

Stage I Stage II Stage III Stage IV

The skin is not The outer layer of skin The ulcer is a deep Ulcer shows large- broken. (epidermis) and part wound. The ulcer scale loss of tissue of the underlying layer looks crater-like. of skin (dermis) is damaged or lost.

The skin appears red The wound may be The loss of skin The wound may on people with lighter shallow and pinkish or usually exposes some expose muscle, bone skin color red. fat. or tendons.

On people with darker The wound may look The bottom of the The bottom of the skin like a fluid-filled wound may have wound likely contains blister or a ruptured some yellowish dead dead tissue that's blister. tissue. yellowish or dark and crusty.

The site may be tender The damage often extends beyond the primary wound below layers of healthy skin.

Unstageable

A pressure ulcer is considered unstageable if its surface is covered with yellow, brown, black or dead tissue. It’s not possible to see how deep the wound is.

Common sites of pressure sores

For people who use a wheelchair, pressure sores often occur on skin over the following sites:

 Tailbone or buttocks  Shoulder blades and spine  Backs of arms and legs where they rest against the chair

For people who are confined to a bed, common sites include the following:

 Back or sides of the head  Rim of the ears  Shoulders or shoulder blades  Hip, lower back or tailbone  Heels, ankles and skin behind the knees Bedsores are caused by pressure against the skin that limits blood flow to the skin and nearby tissues.

Three primary contributing factors are:

 Sustained pressure.

 Friction.

 Shear.

Risk factors: People are at risk of developing pressure sores if they have difficulty moving and are unable to easily change position while seated or in bed. Immobility may be due to:

 Generally poor health or weakness  Paralysis  Injury or illness that requires bed rest or wheelchair use  Recovery after surgery  Sedation  Coma

Other factors that increase the risk of pressure sores include:

 Advancing Age.  Bowel incontinence. .  Lack of sensory perception.  Medical conditions affecting blood flow.  Weight loss.  Smoking.  Poor nutrition and hydration  Limited alertness.  Excess moisture or dryness.  Muscle spasms.

Complications of pressure ulcers include:

 Sepsis.  Cellulitis.  Bone and joint infections.  Cancer. Another complication is the development of a type of squamous cell carcinoma that develops in chronic, nonhealing wounds (Marjolin ulcer). This type of cancer is aggressive and usually requires surgery.

Evaluating a bedsore: To evaluate a bedsore:

 Determine the size and depth of the ulcer  Check for bleeding, fluids or debris in the wound that can indicate severe infection  Try to detect odors indicating an infection or dead tissue  Check the area around the wound for signs of spreading tissue damage or infection  Check for other pressure sores on the body

Investigations:

 Blood tests: complete haemogram, LFT, sugar, urea , creatininie, blood culture.  Tissue cultures to diagnose a bacterial or fungal infection in a wound that doesn't heal with treatment or is already at stage IV  Tissue cultures to check for cancerous tissue in a chronic, nonhealing wound Treatment:

Stage I and II bedsores usually heal within several weeks to months with conservative care of the wound and ongoing, appropriate general care. Stage III and IV bedsores are more difficult to treat.

Reducing pressure

The first step in treating a bedsore is reducing the pressure that caused it. Strategies include the following:

 Repositioning. If you have a pressure sore, you need to be repositioned regularly and placed in correct positions. If you use a wheelchair, try shifting your weight every 15 minutes or so. Ask for help with repositioning every hour. If you're confined to a bed, change positions every two hours.

 Using support surfaces. Use a mattress, bed and special cushions that help you lie in an appropriate position, relieve pressure on any sores and protect vulnerable skin. If you are in a wheelchair, use a cushion. Styles include foam, air filled and water filled. Select one that suits your condition, body type and mobility. Cleaning and dressing wounds

Care that helps with healing of the wound includes the following:  Cleaning.  Applying dressings. Removing damaged tissue

To heal properly, wounds need to be free of damaged, dead or infected tissue.

 Surgical debridement involves cutting away dead tissue.  Mechanical debridement .  Autolytic debridement .  Enzymatic debridement .

Other interventions

Other interventions that may be used are:

 Pain management.  Antibiotics.  A healthy diet. To promote wound healing.  Management of incontinence.  Muscle spasm relief. Muscle relaxants — such as diazepam (Valium), tizanidine (Zanaflex), dantrolene (Dantrium) and baclofen (Gablofen, Lioresal) — may inhibit muscle spasms and help sores heal.  Negative pressure therapy (vacuum-assisted closure, or VAC).

Surgery:

A pressure sore that fails to heal may require surgery. The goals of surgery include improving the hygiene and appearance of the sore, preventing or treating infection, reducing fluid loss through the wound, and lowering the risk of cancer.

In general, most pressure sores are repaired using a pad of your muscle, skin or other tissue to cover the wound and cushion the affected bone (flap reconstruction).

Group – D

4. Write briefly on (any five): 5x4 = 20

a) Fracture clavicle. b) Tennis elbow. c) Supracondylar fracture of humerus. d) Dupuytren’s contracture . e) Ruptured tendoachilles. f) Mallet finger.

Answer. a) Fracture clavicle.

Introduction:

 Clavicle one of the most commonly fractured bones.  Midshaft clavicle fractures account for 69% - 80%  Osseous anatomy and muscular/ligamentous attachments play a pivotal role in determining fracture patterns  # Most common at junction of outer and middle 3rd

o Thinnest part of bone

o Not protected by muscle/ligamentous attachments

Deformity:

 SCM pulls proximal fragment superiorly and posteriorly.  Weight of arm and pectoralis muscles pull distal segment medially and inferiorly

Various mechanisms:

 Direct blow onto the point of the shoulder  Direct blow to clavicle  Fall on outstretched hand(FOOSH)  Others o Sports injury o Motor vehicle accidents

Physical Examination:

 Inspection o Evaluate deformity and/or displacement o Beware of rare inferior or posterior displacement of distal or medial ends of clavicle o Compare to opposite side.  Palpation o Evaluate pain o Look for instability with stress o Neurovascular examination o Evaluate upper extremity motor and sensation o Measure shoulder range-of-motion

X-ray:

 AP view

 10 -15° cephalic tilt view

 # pattern

 Comminution

 Displacement

 Shortening

 Chest, shoulder, c-spine

Classification:

Craig Classification

 Group I: Fracture of the middle third.  Group II: Fracture of the distal third. Subclassified according to the location of coracoclavicular ligaments relative to the fracture.  Group III: Fracture of the proximal third.

Treatment options:

 Nonoperative

o Sling

o Brace

 Most commonly, a sling or figure-of-8 brace applied in the acute setting -immobilization typically for 2 to 6 weeks  Gradual return to normal activities

 Surgical

o Plate Fixation

o Screw or Pin Fixation. b) Tennis elbow. Introduction:

 Also known as lateral epicondylitis.  Tennis elbow is a lesion affecting the tendinous origin of the wrist extensor. Most commonly involves the extensor carpi radialis brevis (ECRB), occasionally, the anterior edges of the extensor communis, the extensor radialis longus, and rarely the origin of the extensor carpi ulnaris. We can say that the lesion may be tenoperiosteal, tendinous or musculotendinous.

Cause:

Overuse Recent studies show that tennis elbow is often due to damage to a specific forearm muscle. The extensor carpi radialis brevis (ECRB) muscle helps stabilize the wrist when the elbow is straight. This occurs during a tennis groundstroke, for example. When the ECRB is weakened from overuse, microscopic tears form in the tendon where it attaches to the lateral epicondyle. This leads to inflammation and pain.

The ECRB may also be at increased risk for damage because of its position. As the elbow bends and straightens, the muscle rubs against bony bumps. This can cause gradual wear and tear of the muscle over time.

Activities

Athletes are not the only people who get tennis elbow. Many people with tennis elbow participate in work or recreational activities that require repetitive and vigorous use of the forearm muscle.

Painters, plumbers, and carpenters are particularly prone to developing tennis elbow. Studies have shown that auto workers, cooks, and even butchers get tennis elbow more often than the rest of the population. It is thought that the repetition and weight lifting required in these occupations leads to injury.

Age Most people who get tennis elbow are between the ages of 30 and 50, although anyone can get tennis elbow if they have the risk factors. In racquet sports like tennis, improper stroke technique and improper equipment may be risk factors.

Unknown

Clinical Examination:

 Pain at the common extensor origin  Often pain 5mm distal and slightly anterior to lateral humeral epicondyle  Pain with resisted wrist extension and digit extension. Pathophysiology

 Micro tears at ECRB  Incomplete healing  Granulation tissue formation  Development of angiofibroblastic hyperplasia

Imaging:

 Plain film  MRI

Nonsurgical treatment:

 Activity modification  Counterforce brace  Wrist extension splint  Stretching  Ultrasound  Strengthening after acute phase  NSAIDs  Steroid injections

Surgical treatment:

 Debride degenerative tissue at ECRB origin  Remove traction spurs  Stimulate angiogenesis  Repair defect/ superficial extensor closure c) Supracondylar fracture of humerus. Distal Humerus Anatomy

 Medial epicondyle proximal to trochlea –

 Lateral epicondyle proximal to capitellum –

 Radial fossa – accommodates margin of radial head during flexion

 Coronoid fossa – accepts coronoid process of ulna during flexion

Introduction:

 It is # which involves the lower end of the humerus usually involving the thin portion of the humerus through o Olecranon fossa or o Just above the fossa or o Metaphysis  Most common elbow injuries in children.  Makes up approximately 60% of elbow injuries.  Becomes uncommon as the age increases.

General consideration:

 Incidence of supracondylar #:

a) Age: peak age: 5-7 yrs

Average age : 6.7 yrs b) Sex: Boys = Girls (Latest Trends)

c) Side: Left > Right

( Non dominant > dominant )

d) Nerve injurie:7% - Median> Radial > Ulnar

e) Vascular injuries:1%

g) Cause of #

o Fall from height 70% ----- children > 3 yrs o Fall from bed children < 3 yrs o Non accidental injury (Child abuse) children < 15 months

h) Associated #s

Distal radius > Scaphoid > Proximal humerus > Monteggia

i) Clinical types

Extension type: 98%

Flexion type : 2%

Open injuries : < 1%

Mechanism of injury:

 For Extension type of SC # humerus

Fall on outstretched hand Elbow hyper extended Fore arm – pronated or supinated

 For Flexion type of SC # humerus

Fall directly on the elbow rather than out stretched hand

Radiography:

 Radiographic views:

Antero posterior Lateral

Oblique

Axial ( jones view )

Clinical features:

 In most cases, children will not move the elbow if a fracture is present, although this may not be the case for non-displaced fractures.  Swelling about elbow is a constant feature, develop within first few hrs.

 S shaped deformity

 Distal humeral tenderness

 Anterior plucker sign +ve

 Nerve injury incidence is high, between 7 and 16 % (median, radial and ulnar nerve)

 Anterior interosseous nerve is most commonly injured nerve

 In many cases, assessment of nerve integrity is limited , because children can not always cooperate with the exam

 Carefully document pre manipulation exam, as post manipulation neurologic deficits can alter decision making

 Thorough documentation of all findings is important. A simple record of “neurovascular status is intact” is unacceptable.

 Individual assessment and recording of motor, sensory, and vascular function is essential

 Always palpate the arm and forearm for signs of compartment syndrome.

Treatment:

General principles:

 Splinting elbow in comfortable position -20-30degrees of flexion of elbow.  Careful physical examination & X-ray evaluation.  Tight bandaging/ excessive flexion or excessive extension should be avoided.  Associated life threatening complications (if any) to be attended first.

Type Treatment I  Simple immobilization with a post slab in 90deg. With a cuff and collar  Xray to be rpted at 5-7 days to document for any displacement  Slab kept for 3 weeks

II  Treatment – closed reduction under anaesthesia  Traction is applied followed by correction of rotational deformity  Extension deformity is corrected with pressure by thumb over the olecranon

III  Closed reduction & percutaneous K wire fixation  Open reduction & K wire fixation

Complications:

Neurovascular complications

(a) tear or entrapment of the brachial artery; (b) spasm of the artery and (c) compression of the artery relieved by manipulation of the fracture (d) compression of ulnar nerve. Causing Pink and Pulseless hand in supracondylar fracture. Thus there is loss of circulation of forearm, causing lack of reperfusion of tissues resulting in tissue death causing compartment syndrome. Therefore the complications of elbow dislocations include the following:

 Posttraumatic periarticular calcification.  Myositis ossificans or calcific tendinitis  Neurovascular injuries (8-21% of cases) — palsy to the anterior interosseus nerve at time of index injury is most common, followed by brachial artery injuries (5-13%). Injury to the ulnar nerve is reported with percutaneous pinning through the medial epicondyle.  Osteochondral defects, intra-articular loose bodies, and avascular necrosis of the capitulum  Instability

Other injuries:

 Most commonly brachial artery injury, and if left untreated could lead to Volkmann's contracture (permanent flexion contracture of the hand at the wrist, resulting in a claw-like deformity of the hand and fingers).

 Cubitus varus: Also known as gunstock deformity. This is more of a cosmetic problem, and does not affect the range of motion of the elbow joint.

 Malunion: The condyles of the distal humerus do not correctly join together. It may be misaligned anteriorly, posteriorly, medially or laterally in respect to the shaft of the humerus. A thicker ossification will also appear on the distal end. Remove it one year later to prevent additional ossification from forming.

d) Dupuytren’s contracture.

Introduction: Dupuytren's contracture is a thickening of the fibrous tissue layer underneath the skin of the palm and fingers. Although painless, the thickening and tightening (contracture) of this fibrous tissue can cause the fingers to curl (flex).

Aetiopathogenesis:

 Common in men older than 40 years; in persons of Northern European descent; and in persons who smoke,use alcohol, or have diabetes (3 to 33 %)

 Present with a small, pitted nodule (or multiple nodules) on the palm, which slowly progresses to contracture of the fingers

 Progresses' faster in <50 yr olds

 Smoking and alcohol use increase the chance that surgery will be needed

Clinical features:

 Found on the palm of the hand proximal to the metacarpo-phalangeal (MCP) joint. Can be bilateral.

 Patients usually have difficulty with tasks such as face washing, hair combing, and putting their hands in their pockets.

 Note the site of the nodule and the presence of contractures; bands; and skin pitting, tenderness, and dimpling.

Treatment:

 Nonsurgical Treatment o Steroid injection. If a lump is painful, an injection of corticosteroid - a powerful anti- inflammatory medication - may help relieve the pain. In some cases, it may prevent the progression of contracture. Several injections may be needed for a lasting effect. o Splints. Splinting does not prevent increased bend in the finger. Forceful stretching of the contracted finger will not help either, and may speed the progression of contracture.

 Surgical Treatment Surgery is recommended when your doctor has confirmed through measurements over time that the disease is progressing. Some patients turn to surgical treatment when hand function is limited; they have trouble grasping objects or putting their hands in their pockets.

 Surgical procedure. Surgery for Dupuytren's contracture divides or removes the thickened bands to help restore finger motion. Sometimes the wound is left open and allowed to heal gradually. Skin grafting may be needed.  Complications:Although rare, risks of surgery include injury to nerves and blood vessels, and infection. Permanent stiffness of the fingers may occur, although this is also rare.  Recovery. Some swelling and soreness are expected after surgery, but severe problems are rare.  Most people will be able to move their fingers better after surgery.  Recurrence. Approximately 20% of patients experience some degree of recurrence. This may require further surgery.

e) Ruptured tendoachilles.

Introduction:

 Achilles tendon rupture is when the achilles tendon breaks.  The achilles is the most commonly injured tendon.  Rupture can occur while performing actions requiring explosive acceleration, such as pushing off or jumping.  The male to female ratio for Achilles tendon rupture varies between 7:1 and 4:1

Causes:

 The Achilles tendon is most commonly injured by sudden plantarflexion ordorsiflexion of the ankle, or by forced dorsiflexion of the ankle outside its normal range of motion.  Other mechanisms by which the Achilles can be torn involve sudden direct trauma to the tendon, or sudden activation of the Achilles after atrophy from prolonged periods of inactivity.  Fluoroquinolone antibiotics, famously ciprofloxacin, are known to increase the risk of tendon rupture, particularly Achilles.  Most cases of Achilles tendon rupture are traumatic sports injuries. The average age of patients is 29–40 years with a male-to-female ratio of nearly 20:1.

Symptoms of an Achilles tendon injury are:

 Pain along the back of foot and above heel, especially when stretching of ankle or standing on toes; with tendinitis, pain may be mild and worsen gradually. If there is rupture of the tendon, pain can be abrupt and severe.  Tenderness

 Swelling  Stiffness

 Hearing a snapping or popping noise during the injury

 Difficulty in flexing foot or pointing toes (in complete tears of the tendon)

Diagnosis:

 Diagnosis is made by clinical history; typically people say it feels like being kicked or shot behind the ankle.  Upon examination a gap may be felt just above the heel unless swelling has filled the gap and the Simmonds' test (Thompson test) will be positive; squeezing the calf muscles of the affected side while the patient lies prone, face down, with his feet hanging loose results in no movement (no passive plantarflexion) of the foot, while movement is expected with an intact Achilles tendon and should be observable upon manipulation of the uninvolved calf. Walking will usually be severely impaired, as the patient will be unable to step off the ground using the injured leg. The patient will also be unable to stand up on the toes of that leg, and pointing the foot downward (plantarflexion) will be impaired. Pain may be severe, and swelling is common. Imaging:  Musculoskeletal ultrasonography can be used to determine the tendon thickness, character, and presence of a tear.  Magnetic resonance imaging (MRI) can be used to discern incomplete ruptures from degeneration of the Achilles tendon, and MRI can also distinguish between paratenonitis, tendinosis, and bursitis.  Radiography can also be used to indirectly identify Achilles tears. Treatment:  Treatment options for an Achilles tendon rupture include surgical and non-surgical approaches. Among the medical profession opinions are divided what is to be preferred.  Non-surgical management traditionally was selected for minor ruptures, less active patients, and those with medical conditions that prevent them from undergoing surgery.  It traditionally consisted of restriction in a plaster cast for six to eight weeks with the foot pointed downwards (to oppose the ends of the ruptured tendon). But recent studies have produced superior results with much more rapid rehabilitation in fixed or hinged boots. Surgery: There are two different types of surgeries; open surgery and percutaneous surgery.  During an open surgery an incision is made in the back of the leg and the Achilles tendon is stitched together. In a complete or serious rupture the tendon of plantaris or another vestigial muscle is harvested and wrapped around the Achilles tendon, increasing the strength of the repaired tendon. If the tissue quality is poor, e.g. the injury has been neglected, the surgeon might use a reinforcement mesh.  In percutaneous surgery, the surgeon makes several small incisions, rather than one large incision, and sews the tendon back together through the incision. Surgery may be delayed for about a week after the rupture to let the swelling go down. For sedentary patients and those who have vasculopathy or risks for poor healing, percutaneous surgical repair may be a better treatment choice than open surgical repair.

Rehabilitation:

After treatment, whether surgical or nonsurgical, patient will go through a rehabilitation program involving physical therapy exercises to strengthen the leg muscles and Achilles tendon. Most people return to their former level of activity within four to six months.

f) Mallet finger.

Introduction:

 Traumatic avulsion of the DIP extensor tendon of the finger.  Most common extensor tendon injury.  Sometimes called baseball finger or drop finger.  Tendon may be stretched, completely torn, or associated with a bony avulsion.

Mechanism of injury:

 Caused by sudden flexion of the distal interphalangeal (DIP) joint while the finger is in an extended position.  Often occurs from a blow to the tip of an extended finger.

Evaluation:

 Dorsal DIP area is tender, painful, swollen, and red initially, but after 2 weeks the fingertip is usually no longer painful  DIP joint is in flexion, and patient is unable to extend the joint.

Treatment:  For most patients, treatment consists of extension splinting of the DIP joint; full time for 6 weeks, followed by nighttime splinting for 6 more weeks

 Dorsal aluminum splint or stack splint

 Patient compliance is necessary - if the tip of the finger droops at any time after splinting begins, the splinting period starts over

o Weekly follow up visits to monitor progress usually lead to better outcome (especially if concerned about compliance)

 After 6 weeks of continuous splinting, may begin guarded active flexion if no extensor lag is evident

 Noncompliant patients, or those in occupations in which continuous splinting is difficult may be better treated surgically

Surgical treatment:

 Surgical evaluation should be sought for patients with an unstable joint, an avulsion fracture involving greater than 25% of the joint surface, or with compliance issues

 Surgical treatment consists of wire fixation

 Surgical reconstruction may be needed for injuries more than 6 weeks old which lead to significant functional loss to the patient.

Complications:

 Left untreated, persistent flexion deformity or a swan-neck deformity is possible

– Allowing PIP flexion while splinting is important to help prevent this

 Even with treatment, some flexion deformity may persist.

The West Bengal University of Health Sciences

M.B.B.S. 3rd Professional Part – II Examination, 2011

1 Subject: Surgery Time: 2 /2 hrs. Paper: I Marks: 60

Group –A 1. Define and classify wound. Discuss factors influencing wound healing. Write in brief management of Diabetic ulcer affecting foot, in a middle aged man. 4+5+6 = 15

Answer: Wound can be defined as an injury, usually involving division of tissue or rupture of the integument or mucus membrane, due to external violence or some mechanical agency rather than disease. General wounds are classified as being:

. Superficial (loss of epidermis only) . Partial thickness (involve the epidermis and dermis) . Full thickness (involve the dermis, subcutaneous fat and sometimes bone)

According to level of contamination a wound can be classified as

 Clean wound, a wound made under sterile conditions where there are no organisms present in the wound and the wound is likely to heal without complications.  Contaminated wound, where the wound is as a result of accidental injury where there are pathogenic organisms and foreign bodies in the wound.  Infected wound, where the wound has pathogenic organisms present and multiplying showing clinical signs of infection, where it looks yellow, oozing pus, having pain and redness.  Colonized wound, where the wound is a chronic one and there are a number of organisms present and very difficult to heal as in a bedsore. Open wounds can be classified according to the object that caused the wound. The types of open wound are:

 Incisions or incised wounds, caused by a clean, sharp-edged object such as a knife, razor, or glass splinter.  Lacerations, irregular tear-like wounds caused by some blunt trauma. Lacerations and incisions may appear linear (regular) or stellate (irregular). The term laceration is commonly misused in reference to incisions.  Abrasions (grazes), superficial wounds in which the topmost layer of the skin (the epidermis) is scraped off. Abrasions are often caused by a sliding fall onto a rough surface.  Avulsions, injuries in which a body structure is forcibly detached from its normal point of insertion. A type of amputation where the extremity is pulled off rather than cut off.  Puncture wounds, caused by an object puncturing the skin, such as a splinter, nail or needle.  Penetration wounds, caused by an object such as a knife entering and coming out from the skin.  Gunshot wounds, caused by a bullet or similar projectile driving into or through the body. There may be two wounds, one at the site of entry and one at the site of exit, generally referred to as a "through-and-through." Closed wounds have fewer categories, but are just as dangerous as open wounds. The types of closed wounds are:

 Hematomas, also called a blood tumor, caused by damage to a blood vessel that in turn causes blood to collect under the skin.  Hematomas that originate from internal blood vessel pathology are petechiae, purpura, and ecchymosis. The different classifications are based on size.  Hematomas that originate from an external source of trauma are contusions, also commonly called bruises.  Crush injury,caused by a great or extreme amount of force applied over a long period of time.

Surgical wounds:

 Class I: Clean • Uninfected operative wound where no inflammation is encountered and respiratory, GI, genital, and urinary tracts aren’t entered. • Wounds are primarily closed, and a drain (if needed) is connected to a closed system.  Class I: Clean • Uninfected operative wound where no inflammation is encountered and respiratory, GI, genital, and urinary tracts aren’t entered. • Wounds are primarily closed, and a drain (if needed) is connected to a closed system.  Class III: Contaminated • Open, fresh, accidental wound from surgery with a major break in sterile technique or gross spillage from GI tract; incision in which acute, nonpurulent inflammation is encountered (including necrotic tissue without evidence of purulent drainage, such as dry gangrene)  Class IV: Dirty/infected • Old traumatic wounds with retained devitalized tissue; procedures with existing clinical infection (purulence already present in wound) or perforated viscera.

Local and Systemic Impediments to Wound Healing.

Systemic Local Malnutrition Wound infection Diabetes mellitus Wound necrosis Drugs (steroids, cytotoxins) Foreign bodies Obesity Wound hypoperfusion and hypoxia Shock Repeat trauma Immunodeficiency Irradiated tissue Renal failure Neoplasm

Intrinsic or local factors:

 Ischemia and hypoxia are common contributing causes of nonhealing wounds. Atherosclerosis or local damage to vessels in the form of trauma or vasculitis causes ischemia and subsequent hypoxia in the wound. Hypoxia leads to impaired collagen synthesis, prevents fibroblast migration, and increases the susceptibility of the wound to infection.  Infection in the wound delays healing. The host's resistance can be impaired by diabetes, malnutrition, malignancy, steroids, or other immunosuppressive therapies. If there are allowed to persist, wound infections lead to increased tissue destruction and alter the effect of cytokines on wound healing.  The presence of foreign bodies and necrotic tissue can contribute to delayed wound healing.  Chronic venous insufficiency leads to persistent venous hypertension and chronic edema in the lower extremities. These factors in turn lead to pericapillary fibrosis, tissue ischemia, and the liberation of superoxide radicals, which are thought to result in delayed wound healing in extremities with chronic venous insufficiency.  Ionizing radiation to the wound leads to abnormal wound healing.  Edema. Acute swelling, especially around joints, can lead to skin breakdown and full- thickness skin loss. Extrinsic or systemic factors:

 Malnutrition alters normal healing through the indirect and the direct effects of vitamin and mineral deficiency. An example is the patient with clinical or subclinical scurvy (vitamin C deficiency); such an individual produces inadequately hydroxylated collagen, and the healed wound becomes significantly weakened as a result.  Diabetes mellitus is believed to affect healing adversely at every level and in every phase of the healing process.  Steroids and antineoplastic drugs can markedly diminish the speed and quality of the healing process. Vitamin A seems to cause a partial reversal of the detrimental effects of steroids on healing.  Smoking contributes to delayed wound healing by causing cutaneous vasoconstriction, decreasing the oxygen-carrying capacity of hemoglobin, and contributing to atherosclerosis.  Collagen vascular diseases are often accompanied by a vasculitic component, which needs to be controlled before healing can begin..  Cleansing agents such as chlorhexidine gluconate (Hibiclens) or povidone-iodine (Betadine) or chemicals may impair wound healing by affecting cell migration.  Repetitive trauma, intentional or otherwise, from shearing or pressure forces often leads to a failure in healing. Wound areas over pressure points often require stabilization of the overlying skin envelope with external taping or splinting.  Wounds in patients with renal disease and liver disease often heal more slowly due to chronic protein deficiencies and reduced capacities for protein synthesis.  Hematopoietic disorders. Sickle cell disease, with its high incidence of ankle wounds and leukoclastic and granulomatous processes, and mycosis fungoides are associated with poorly healing wounds. Maximal medical treatment for the underlying disorder is needed to effect meaningful healing.

Management of diabetic ulcer:

Successful diagnosis and treatment of patients with DFUs involves a holistic approach that includes:

 Optimal diabetes control  Effective local wound care  Infection control  Pressure relieving strategies  Restoring pulsatile blood flow.

Classification and severity of diabetic foot infections:

The essential components of management are:

 Treating underlying disease processes  Ensuring adequate blood supply  Local wound care, including infection control  Pressure offloading

Treating the underlying disease processes:

 One should identify the underlying cause of the DFU during the patient assessment and, where possible, correct or eliminate it.  Treating any severe ischaemia is critical to wound healing, regardless of other interventions.  It is recommended that all patients with critical limb ischaemia, including rest pain, ulceration and tissue loss, should be referred for consideration of arterial reconstruction.  Achieving optimal diabetic control. This should involve tight glycaemic control and managing risk factors such as high blood pressure, hyperlipidaemia and smoking.  Nutritional deficiencies should also be managed.  When possible and appropriate one should check other footwear worn at home and at work (eg slippers and work boots).

Ensuring adequate blood supply:

 A patient with acute limb ischaemia is a clinical emergency and may be at great risk if not managed in a timely and effective way.  It is important to appreciate that, aside from critical limb ischaemia, decreased perfusion or impaired circulation may be an indicator for revascularisation in order to achieve and maintain healing and to avoid or delay a future amputation.

Optimising local wound care:

 Emphasis in wound care for DFUs should be on radical and repeated debridement, frequent inspection and bacterial control and careful moisture balance to prevent maceration.  Tissue debridement  Inflammation and infection control  Moisture balance (optimal dressing selection)  Epithelial edge advancement.  Tissue debridement: There are many methods of debridement used in the management of DFUs including surgical/sharp, larval, autolytic and, more recently, hydrosurgery and ultrasonic. Debridement may be a one-off procedure or it may need to be ongoing for maintenance of the wound bed. The requirement for further debridement should be determined at each dressing change The benefits of debridement include:

 Removes necrotic/sloughy tissue and callus  Reduces pressure  Allows full inspection of the underlying tissues  Helps drainage of secretions or pus  Helps optimise the effectiveness of topical preparations  Stimulates healing.  It is important to debride the wound margins as well as the wound base to prevent the ‘edge effect’, whereby epithelium fails to migrate across a firm, level granulation base.

Autolytic debridement: This is a natural process that uses a moist wound dressing to soften and remove devitalised tissue. Care must be taken not to use a moisture donating dressing as this can predispose to maceration. In addition, the application of moisture-retentive dressings in the presence of ischaemia and/or dry gangrene is not

 Clinically uninfected wounds should not be treated with systemic antibiotic therapy. However, virtually all infected wounds require antibiotic therapy.  Superficial DFUs with skin infection (mild infection) – For mild infections in patients who have not recently received antibiotic treatment:  Start empiric oral antibiotic therapy targeted at Staphylococcus aureus and ß- haemolytic Streptococcus  Change to an alternate antibiotic if the culture results indicate a more appropriate antibiotic  Obtain another optimum specimen for culture if the wound does not respond to treatment.

Role of topical antimicrobials: The increasing prevalence of antimicrobial resistance (eg meticillin-resistant S. aureus [MRSA]) or other complications (eg Clostridium difficile infection) has led to a rise in the use of topical antimicrobial treatments for increased wound bioburden. Antimicrobial agents that are used topically have the advantage of not driving resistance. Such agents provide high local concentrations, but do not penetrate intact skin or into deeper soft tissue. Topical antimicrobials may be beneficial in certain situations.

Pressure offloading:

 In patients with peripheral neuropathy, it is important to offload at-risk areas of the foot in order to redistribute pressures evenly.  Inadequate offloading leads to tissue damage and ulceration.  The gold standard is the total contact cast (TCC).  Using a TCC in patients with a unilateral uncomplicated plantar ulcer can reduce healing time by around six weeks.  Disadvantages of TCCs include:  Must be applied by fully trained and experienced practitioners  May cause skin irritation and further ulcers if applied inappropriately  Prevents daily inspection (signs of spreading infection may go unnoticed)  May disturb sleep  Makes bathing difficult  Patient may not tolerate it (especially in warm climates)  May prevent patient's ability to work  Relatively high cost/low availability.  In patients with ischaemic or neuroischaemic ulcers, the priority is to protect the margins of the foot (eg using Scotchcast boots or healing sandals). TCCs are contraindicated in patients with ischaemia because of the risk of inducing further DFUs. Removable devices (such as removable cast walkers, Scotchcast boots, healing sandals and crutches, walkers and wheelchairs) should be selected in these patients. Removable devices may also be more pragmatic choices for less motivated patients because they allow patients to bathe and sleep more comfortably.

Prevention:

 Multidisciplinary team approach  Patient education  Daily foot inspections by patient  Shoegear changes  Elective surgery to decrease risk

Group – B 2. Answer any of the following questions:

a. A forty five year old lady presents with acute upper abdominal pain, discuss the differential diagnosis and management of such a patient. 7+8 = 15 Answer: Differential diagnosis:

Common causes Diffuse pain Specific for women

 Biliary disease: cholelithiasis  Mesenteric ischemia  Pelvic inflammatory disease and choledocholithiasis and infarction  Adnexal pathology  Acute pancreatitis  Ruptured aneurysm  Endometriosis  Gastritis.  Peritonitis  Ectopic pregnancy  Dyspepsia  Intestinal obstruction  Endometritis  Hiatus hernia  Leiomyomas  Pneumonia  Myocardial infarction  Splenic abscess or infarction

History and clinical examination:  Character, duration and frequency of the pain;  Location and distribution of referred pain: This gives a clue to the anatomical site;  Aggrevating and relieving factors: The relationship of the pain to food and toileting gives clues to the possible cause;  Any associated symptoms such as fever, chills, weight loss, nausea, vomiting, diarrhoea, constipation, jaundice, change in colour of the urine or stool, chest pain or shortness of breath;  Which medications patient is taking;  Previous medical problems or past surgery;  Menstrual history

Investigations:  Blood tests which can help find if there is an infection.  Urine pregnancy test.  Chest and abdominal x-rays which can show dilated areas of bowel;  Ultrasound is useful for identifying pancreatitis, cholecystitis, appendicitis (if examined by experienced personnel) aortic aneurysm and gynaecological problems;  CT which is a very reliable and accurate investigation;  Endoscopic procedures are useful in evaluating chronic causes of abdominal pain including peptic ulceration, inflammatory bowel disease and gastrointestinal cancer.  Surgery: Sometimes a camera will need to be inserted into the abdomen (laparoscopy) or the abdominal wall opened up (open laparotomy) to directly examine the organs and find the possible cause. Treatment can often occur at the same time.

Management:  The management of abdominal pain largely depends on the cause of pain.  Some conditions such as appendicitis always need surgery (either open or laparoscopic) whilst some infections can be treated easily with antibiotics only.  Irritable bowel syndrome can be treated with lifestyle changes and the use of laxatives as required.  Abdominal pain may require hospital admission to allow a surgical opinion and ensure appropriate access to all the required tests.  If abdominal pain is really severe and acute you may require drips, oxygen and close monitoring. Pain relief will be given to patients with abdominal pain if it is considered appropriate and will usually be a low dose morphine-derivative (opioid).  If biliary problems are suspected NSAIDs are used instead because opioids can exacerbate these problems.  Acute cholecystitis and pancreatitis are treated conservatively.

b. Enumerate the common causes of intestinal obstruction in an infant. Write the clinical features, investigations and management of intussusception in a 7 months old child. 5+10 = 15 Answer. Common causes of intestinal obstruction in an infant:  Intussusceptions  Anorectal malformations  Congenital pyloric stenosis  Duodenal atresia (post ampullary)  Jejuno –ileal atresia  Malrotation with or without volvulus (more common with volvulus)  Hirschsprung‘s‘ disease (functional obstruction)  Meconium disease of infancy (Meconium ileus, Meconium peritonitis, cystic Meconium peritonitis, small left colon syndrome, Meconium plug syndrome.) Uncommon causes  Pyloric atresia  Annular pancreas causing duodenal obstruction (undistinguishable from duodenal atresia)  Duodenal web with a hole causing partial duodenal obstruction  Windsock abnormality – duodenal diaphragm getting stretched into distal gut with chronic duodenal obstruction.  Pre ampullary duodenal atresia (non bilious vomiting with double bubble)  Colonic atresia  Rectal atresia  Milk curd obstruction Other important causes  Oesophageal atresia  Obstructed inguinal hernias

Intussusceptions:

History:

 The child appears to have intermittent pain which is colicky, severe and may be associated with the child drawing up the legs.  Episodes typically occur 2-3 times/hour and may increase over the next 12-24 hours  During these episodes of crying the child may look pale. (note: many other causes of infant crying are associated with facial redness rather than pallor).  Pallor and lethargy may be the predominant presenting signs, may be persistent rather than episodic, and in some the crying episodes may not be very vigorous.  Vomiting is usually a prominent feature (but bile stained vomiting is a late sign)  Bowel motions  blood and/or mucus  classic red currant jelly stool is a late sign Diarrhoea is quite common and can lead to a misdiagnosis of gastroenteritis There may be a preceding respiratory or diarrhoeal illness

Examination:

 Pallor, lethargy - may be intermittent, and may look well in between episodes  Abdominal mass - sausage shaped mass RUQ or crossing midline in epigastrium or behind umbilicus, palpable in about two thirds of children.  Distended abdomen later in the course  Stool - Bloody stool/occult blood positive

PR unnecessary if good evidence of intussusception; abdominal mass or PR bleeding, but otherwise should be done for signs of PR blood or mass.

Signs of an acute bowel obstruction Hypovolaemic shock is a late sign

Investigations: Plain abdominal X- ray

 Performed to exclude perforation or bowel obstruction  A normal AXR does not exclude intussusception  Signs of intussusception on a plain Xray include: 1. Target sign - 2 concentric circular radiolucent lines usually in the right upper quadrant 2. Crescent sign - a crescent shaped lucency usually in the left upper quadrant with a soft tissue mass

Ultrasound scan

 Diagnostic investigation of choice (unnecessary if high level of suspicion)  Useful if there is a suggestive history but no mass palpable or signs on plain AXR and may identify other pathology.  Ultrasound signs include:

 target sign (also known as the doughnut sign)  pseudokidney sign  crescent in a doughnut sign

Air enema

 This investigation is both diagnostic and therapeutic and is the initial investigation of choice if strong clinical suspicion of intussusception (eg. palpable mass & bowel obstruction)  There is a small risk of bowel perforation and bacteraemia during the gas enema (the surgical registrar as well as a suitably trained nurse should attend with appropriate resuscitation equipment)

CT scan:

CT has become the modality of choice for assessment of acute abdomen in adults, and thus most frequently images intussusception. In addition short length transient intussusception is a frequent incidental finding.

The appearance of intussusception on CT is characteristic and depends on the imaging plane and where along the bowel images are obtained.

Best know is the so-called bowel-within-bowel configuration, in which the layers of the bowel are duplicated forming concentric rings (CT equivalent of the ultrasonographictarget sign) when imaged at right angles to lumen, and a soft tissue sausage when imaged longitudinally.

At the proximal end of the intussusception there will be two concentric enhancing / hyperdense rings, formed by the inner bowel and the folded edge of the outer bowel. As one images further along the intussusception the mesentery (fat and vessels) will form a crescent of tissue around the compressed inner most lumen, surrounded by the two layers of the outer enveloping bowel. Even further distally the lead point (if present) will be visualized.

Blood tests

 Blood glucose  FBE and U&E's may be useful if child looks unwell  Blood group and hold prior to theatre

Management:

If shocked resuscitate Consider for surgery early

 Secure IV access for all patients who are suspected to have intussusception  Most children will require fluid resuscitation with IV boluses of 20mls/kg normal saline before radiological investigations  Give adequate analgesia (usually morphine)  Keep nil orally  Pass nasogastric tube if bowel obstruction or perforation on AXR  Consider IV antibiotics before air enema.  In children, intussusceptions can be treated without recourse to surgery in most cases. Using a water soluble medium or air, retrograde pressure can be exerted to reduce the intussusception. If symptoms have been protracted, rectal blood is present, there are signs of peritonitis or enema reduction is unsuccessful then surgical intervention is usually required. With no surgical treatment there can be recurrence rates of up to 30%.

Group – C 3. Write short notes on (any two): 2x5 = 10 a) Preoperative preparation of a case of obstructive jaundice. b) Epigastric hernia. c) D.V.T. d) Active immunisation against tetanus. Answer. a) Preoperative preparation of a case of obstructive jaundice.  Proper diagnosis and assessment.  Injection vitamin K 1M 10 mg for 5 days.  Fresh Frozen plasma‐often requires 6 bottles or more.  Blood transfusion in case of anaemia.  Oral neomycin, lactulose Mannitol 100‐200 ml BD IV to prevent hepato renal syndrome.  Hydration  Repeated monitoring by doing prothrombin time,electrolytes.  Antibiotics like third generation cephalosporins.  Calcium supplements as calcium chloride IV.

b) Epigastric hernia. Introduction:  These arise through the midline raphe (linea alba) anywhere between the xiphoid process and the umbilicus, usually midway. When close to the umbilicus they are called supraumbilical hernias.  Epigastric hernias begin with a transverse split in the midline raphe.  The defectis elliptical.  It has been hypothesised that the defect occurs at the site where small blood vessels pierce the linea alba or, more likely, that it arises at weaknesses due to abnormal decussation of aponeurotic fibres related to heavy physical activity.  Epigastric hernia defects are usually less than 1 cm in maximum diameter and commonly contain only extraperitoneal fat which gradually enlarges, spreading in the subcutaneous plane to resemble the shape of a mushroom.  When very large they may contain a peritoneal sac but rarely any bowel.  More than one hernia may be present.

Aetiology:  Incidence 1-5%  Men> women  20% multiple  The primary risk factors are obesity and pregnancy. Clinical features:  The patients are often fit, healthy males between 25 and 40 years of age.  These hernias can be very painful even when the swelling is the size of a pea due to the fatty contents becoming nipped sufficiently to produce partial strangulation.  The pain may mimic that of a peptic ulcer but symptoms should not be ascribed to the hernia until gastrointestinal pathology has been excluded.  A soft midline swelling can often be felt more easily than it can be seen.  It may be locally tender. It is unlikely to be reducible because of the narrow neck. It may resemble a lipoma.  A cough impulse may or may not be felt.

Treatment:  Very small epigastric hernias have been known to disappear spontaneously, probably due to infarction of the fat.  Small to moderate-sized hernias without a peritoneal sac are not inherently dangerous and surgery should only be offered if the hernia is sufficiently symptomatic.

Operation:  This may be done by open or laparoscopic surgery.  The most common cause of ‘recurrence’is failure to identify a second defect at the time of original repair.

c) D.V.T. Causes and features:

 May develop in association with abnormalities of the vein wall, blood flow, or constituents of blood (Virchow's triad).  May be due to vein compression or stasis (immobility, trauma, mass, bed rest, surgery, paralysis, long distance travel including airline travel).  May be due to inherited hypercoaguability (factor V Leiden, protein C, protein S, or antithrombin insufficiency).  May be due to acquired hypercoaguability (surgery, malignancy, polycythaemia, smoking, hormone replacement therapy, OCP, dehydration).  Severity may vary from isolated asymptomatic tibial/calf thrombosis to severe iliofemoral segment thrombosis with phlegmasia caerulea dolens (venous gangrene).

Documented risk factors for DVT:

 Increasing Age  Cardiac Dysfunction  Cancer And Its Treatment  Obesity  Paralysis  Nephrotic Syndrome  Prolonged Immobility  Indwelling Central Venous  Major Surgery  Varicose Veins Catheters  Pregnancy Or Estrogen Use.  Stroke  Previous Venous  Trauma Thromboembolism  Inflammatory Bowel Disease

Clinical features:

 Clinical manifestations may be absent.  Local features of venous engorgement and stasis:  limb swelling;  pain;  erythema and warmth to the touch;  mild fever and tachycardia resulting from release of inflammatory mediators;  Homan's sign calf pain on dorsiflexion of the foot is very unreliable and should not be performed.  Complications:  pulmonary embolism;  venous gangrene (phlegmasia dolens).

Diagnosis and investigations:

 Aim to confirm presence and extent of thrombosis (to decide on necessity and type of treatment, risk of embolization).  Fibrin, Fibrinogen Assays:The basis of fibrin or fibrinogen can be assayed by measuring the degradation of intravascular fibrin. The D-dimer test measures cross- linked degradation products, which is a surrogate of plasmin's activity on fibrin. It is shown that in combination with clinical evaluation and assessment, the sensitivity exceeds 90% to 95%.  Ascending venography: rarely used now.  Duplex scan: investigation of choice. Visualizes anatomy and gives extent of thrombosis. Relies on flow of blood and compressibility of vein. Is operator-dependent and has lower sensitivity for calf DVT.  VQ scan: If suspicion or evidence of pulmonary embolism.  CT pulmonary angiography (CTPA): safest, most sensitive, and most specific investigation for suspected pulmonary embolism.

Recommendations:

 Low Risk Patients – No prophylaxis is needed other than early ambulation  Moderate Risk – Low dose unfractionated heparin (LDUH) (5000 U) BID or low molecular weight heparin (LMWH) (< 3,400 U) QD or intermittent pneumatic compression stocking (IPCS). There is some data suggesting multiple modalities may be synergistic.  High Risk – LDUH (5000U) BID or TID or LMWH (< 3,400 U) QD or IPCS  Highest Risk – LDUH (5000U) BID or TID or LMWH (< 3,400 U) QD and IPCS

Treatment:

 Prophylaxis.  Conservative measures: bed rest, elevation, and good hydration.  Uncomplicated DVT: o Low molecular weight heparin (LMWH), initially in hospital; may be given on an outpatient basis via a dedicated DVT clinic. Subsequent treatment is with oral anticoagulation with warfarin for 3-6 months.  Complicated DVT: o Initially with IV unfractionated heparin (UFH) whilst converting to oral anticoagulation with warfarin.  Thrombolysis or surgical thrombectomy are reserved for severe thrombosis with venous gangrene.  Vena caval filter percutaneously inserted via jugular vein into infrarenal IVC to catch thromboemboli and prevent PE. o Used for patients: with recurrent PEs despite treatment; at risk of major central PE; requiring urgent surgery despite high risk that DVT is present. o Risks include IVC obstruction, renal vein thrombosis, complications of insertion. o d) Active immunisation against tetanus.

Indications: As Tetanus can occur in cases of even minor injuries it is advisable to actively immunize every person in general. With this aim in view, it is desirable: To actively immunize all children from the age of 6 weeks onwards.

 To protect infants against the risks of tetanus neonatorum by immunizing pregnant mothers.  To actively immunize civil population particularly those who are exposed to occupational risks such as road workers, athletes agricultural workers, industrial workers etc.  To actively immunize civil and defense personnel, home guards and police personnel.

Dosage: The full basic course of immunisation against tetanus toxiod consists of three primary doses of 0.5ml at least four weeks apart, followed by booster doses at 18 months, 5 years, 10 years and 16 years and then every 10 years.

Protection of newborn against tetanus: For prevention of neonatal tetanus, tetanus toxoid is recommended for immunization of women of childbearing age, and especially pregnant women. Tetanus toxoid may be safely administered during pregnancy and should be given to the mother at first contact or as early as possible in pregnancy. Pregnancy: After completing the full basic course of 7 doses, there is no need for additional doses during pregnancy at least for the next 10 years; thereafter a single booster would be sufficient to extend immunity for another 10 years. For pregnant woman who have not had previous immunisation, at least 2 doses of tetanus toxoid at four weeks interval, 2 dose at least 2 weeks before delivery should be given during pregnancy so that protective antibody would be transferred to the infant in order to prevent neonatal tetanus.

Vaccination of injured persons: For those subjects who have proof of either completing their course of primary immunizations containing tetanus toxoid or receiving a booster shot within the previous 5 years no additional dose of tetanus toxoid is recommended. If more than 5 years have elapsed, and infection with tetanus because of injury or other cause is suspected, 0.5ml of the adsorbed tetanus toxoid should be given immediately. Where the immunization history is inadequate 1500 IU tetanus antiserum and 0.5ml Tetanus toxoid should be injected, with separate syringes, to different body sites. (If available, 250 units of tetanus immune globulin (human origin) can be substituted for the tetanus antiserum). A second 0.5ml dose of toxoid is recommended after 2 weeks and a third dose after a further 1 month. (A note of caution : if Tetanus antiserum from heterologous origin is used in prophylaxis, the patient should be tested for sensitivity to horse serum protein prior to its administration. It is desirable to have 1 ml of Epinephrine Hydrochloride solution (1:1000) immediately available and the normal precautions followed when injecting antitoxins).

Group – D 4. Write briefly on (any five): 5x4 = 20

a) Trigger finger. b) Ewing’s tumour. c) Mechanism of fracture patella. d) Brodie’s abscess . e) Carpal tunnel syndrome. f) Shoulder dislocation. Answer. a) Trigger finger. Introduction:

 Trigger finger and trigger thumb, also known as “stenosing tenosynovitis,” are conditions affecting the movement of the tendons as they bend the fingers or thumb toward the palm of the hand. Triggering is usually the result of a thickening in the tendon that forms a nodule, or knob.  The pulley ligament may thicken as well. The constant irritation from the tendon repeatedly sliding through the pulley causes the tendon to swell in this area and create the nodule. Rheumatoid arthritis, partial tendon lacerations, repeated trauma from pistol- gripped power tools, or long hours grasping a steering wheel can contribute to the development of trigger finger. Infection or damage to the synovium causes a rounded swelling (nodule) to form in the tendon. Aetiology:

 Trigger finger may be caused by highly repetitive or forceful use of a finger or thumb.  Medical conditions that cause changes in tissues -- such as rheumatoid arthritis, gout, or diabetes -- also may result in trigger finger.  Prolonged, strenuous grasping, such as with power tools, also may aggravate the condition.  Farmers, industrial workers, and musicians can be affected by trigger finger since they rely on their fingers or thumbs for multiple repetitive movements.  Trigger finger is more common in women than in men and tends to occur most frequently in people who are between 40 and 60 years of age. Symptoms:

 Occur when there is a size mismatch between the flexor tendons of the fingers or thumb and the sheath which surrounds the tendons. Instead of gliding smoothly through the sheath, the tendons can meet resistance within the tight sheath, which can be painful. Some patients report “catching” or “clicking” of the finger with attempts at motion.  Some people may feel a tender nodule or "bump" in the palm at the base of the finger/thumb where a portion of the tendons and sheath are thickened and firm. In severe cases, the finger can become stuck or "locked" in flexion, requiring the person the straighten the finger with the other hand.  The affected tendons are tough, fibrous bands of tissue that connect the muscles of the forearm to your finger and thumb bones. Together, the tendons and muscles allow you to bend and extend your fingers and thumb, for example, as in making a fist. Treatment:

 Self limiting 10-20%

 NSAID

 Splint. 6 - 8 weeks (MCP in 10-15 degrees flexion). Effective 66% of the cases

 Steroid injection. Effectiveness 50% - >90% Operative:

 Open or percutanous.

 Success >90%

 More complications (nerve damage, inflammation)

b) Ewing’s tumour. See Question 4.e of Group – D of Supplementary Paper – I of 2014. c) Mechanism of fracture patella.

Biomechanics:

 The patella undergoes approximately 7 cm of translation from full flexion to extension  Only 13-38% of the patellar surface is in contact with the femur throughout its range of motion.  The patella increases the moment arm about the knee - Contributes up to 30% improvement in lever arm  Patella withstands compressive forces greater than 7X body weight with squatting Aetiology of patellar fracture:

 Direct trauma

– Direct blow to flexed knee (dashboard) – Increasing cases with penetrating trauma – Comminution & articular marginal impaction  Indirect trauma

– Flexion force directed through the extensor mechanism against a contracted quadriceps

– Simple, transverse fracture Mechanism:

 Fractures of the patella may result from either direct or indirect mechanisms. The classic indirect mechanism is a fall on the feet in which the quadriceps eccentrically fire to decelerate the body. When the force of the fall overwhelms the resistance to knee flexion, the extensor mechanism fails. Depending on the velocity of the force (owing to viscoelastic properties, ie, time-dependent strain, of tissues) either the tendons will rupture or the bone will break.  Indirect forces typically lead to transverse fractures with, at times, substantial displacement of the fracture fragments. By contrast, a direct blow more likely results in comminution, articular injury, anterior soft tissue damage, and thus open injury. Combinations of patterns are common: a direct impact accompanied by knee flexion and quadriceps contraction can cause marked fragment displacement and soft tissue injury. d) Brodie’s abscess . See Question 4.c of Group – D of Supplementary Paper – I of 2014. e) Carpal tunnel syndrome. See Question 4.d of Group – D of Supplementary Paper – I of 2014. f) Shoulder dislocation.

Introduction:

 Shoulder dislocations are the most common joint dislocation seen by emergency room doctors, accounting for more than 50% of all dislocations treated in hospitals.

 Young adult men and older women tend to be the groups with the highest rate of shoulder dislocations.

 Almost all shoulder dislocations are related to trauma.

 Occasionally, the dislocation occurs after ordinarily harmless motions, such as raising an arm or rolling over in bed. In these mysterious cases, the real cause may be that the shoulder ligaments are abnormally loose. Loose ligaments are sometimes due to an inherited condition that can increase a person's risk of dislocation in other body joints as well.

Anatomical predisposition to shoulder dislocation:

• Glenoid defects • Labral defects • Neuromuscular disorders

Types:

Anterior dislocation Posterior dislocation Inferior dislocation

 The top of the humerus is  The top of the humerus is  The top of the humerus is displaced forward displaced toward the back displaced downward. of the body.

 This is the most common  Posterior dislocations  This type of shoulder type of shoulder account for 2% to 4% of dislocation is the rarest, dislocation all shoulder dislocations occurring in only one out and are the type most of every 200 cases. likely to be related to seizures and electric shock.

 In young people  Posterior dislocations also  It can be caused by various can happen because of a types of trauma in which  In older people fall on an outstretched arm the arm is pushed violently or a blow to the front of downward. the shoulder.

Symptoms Symptoms of a dislocated shoulder include:

 Severe shoulder pain  Limited motion of the shoulder  A distortion in the contour of the shoulder — in an anterior dislocation, the side silhouette of the shoulder has an abnormal squared-off appearance instead of its typical sloping, rounded contour. In a posterior dislocation, the front of the shoulder may look abnormally flat.  A hard knob under the skin near the shoulder — this knob is the top of the humerus that has popped out of its socket.  Shoulder bruising or abrasions if an impact has caused the injury

Diagnosis  Examine both shoulders, comparing the injured shoulder with the uninjured one.

 Check for swelling, shape changes, abrasions, bruising, pain on movement, tenderness and limited motion at the shoulder joint.  Gently press and feel the area around the shoulder to locate the displaced head of the humerus under the skin.

 In addition, because many important blood vessels and nerves travel through the shoulder area, check the strength of the pulses at the wrist and elbow and check the muscle strength and response of the patient on touch to arm, hand and fingers.

 If the results of physical examination suggest that that there is a dislocated shoulder, go for shoulder X-rays to confirm the diagnosis.

Treatment:

 Closed reduction  Surgery - weak shoulder joint or ligaments and tend to have recurring shoulder dislocations despite proper strengthening and rehabilitation.  Immobilization.  Medication- pain reliever or a muscle relaxant  Rehabilitation.

The West Bengal University of Health Sciences

M.B.B.S. 3rd Professional Part – II Examination, 2010

1 Subject: Surgery Time: 2 /2 hrs.

Paper: I Marks: 60

Group –A

1. What are the coagulation factors? Write in detail about the mechanism of hemostasis. 5+10 = 15

Answer.

Mechanisms of hemostasis:

Coagulation is a complex equilibrium among thrombotic, anticoagulant, and fibrinolytic processes. Disruption of these processes can lead to severe hemorrhagic or thrombotic complications.

 Platelets serve a primary role in hemostasis by filling defects in the vessel wall and by releasing thrombotic mediators. Endothelial injury exposes platelets to underlying collagen, which activates them. Von Willebrand factor (vWF) binds to platelet glycoprotein Ib/IX, allowing platelet adhesion. Bridges between activated platelets are formed through fibrinogen interaction with glycoprotein IIb/IIIa. Activated platelets release vasoactive agents and express binding sites for coagulation factors, causing platelet aggregation and thrombus formation.  The coagulation cascade is a series of reactions involving the activation of serine proteases (factors), which eventually leads to the formation of a cross-linked fibrin and platelet thrombus. Activation of two different pathways can initiate the coagulation cascade (Fig. 5- 1). The extrinsic pathway begins when activated factor VIIa interacts with tissue factor, a lipoprotein released by injured cells. The intrinsic pathway is initiated by activation of factor XI to XIa by contact with activated factor XII, plasma prekallikrein, and high-molecular- weight kininogen (HMWK). Although the extrinsic pathway appears to be more important in initiating coagulation, both pathways are required for normal hemostasis. Both pathways merge in the production of factor Xa, the first enzyme of the common pathway. In the final step of the cascade, fibrin is produced by the action of thrombin on fibrinogen. Insoluble fibrin is then cross-linked by activated factor XIII. In addition to the enzymatic clotting factors, cofactors (e.g., factors V and VIII) and calcium are required for normal coagulation to occur on platelet phospholipid surfaces. Deficiencies of any of the coagulation factors, except for factor XII, HMWK, and prekallikrein, can lead to abnormal bleeding.

Group – B

2. Answer any of the following questions: a) What are the causes of upper GI bleed? How will you manage acute variceal bleeding? 8+7=15

Answer.

Common Causes of Upper Gastrointestinal Hemorrhage PORTAL HYPERTENSIVE NONVARICEAL BLEEDING (80%) BLEEDING (20%) Peptic ulcer disease Gastroesophageal varices Mallory-Weiss tears Hypertensive portal gastropathy Dieulafoy's Lesion Gastric Antral Vascular Ectasia Gastritis or duodenitis Isolated gastric varices Esophagitis Arteriovenous malformations Tumors: Leiomyoma stomach Malignancy:  Gastrointestinal stromal tumour  Lymphoma  Carcinoma stomach

Aortoenteric Fistula Other:  Hemobilia  Hemosuccus Pancreaticus  Iatrogenic Bleeding

Algorithm for diagnosis and management of GI hemorrhage related to portal hypertension.

b) What are the causes of obstructive jaundice? Write about the management of carcinoma head of pancreas. 5+10 = 15

Answer.

Causes of obstructive jaundice:

Benign Malignant  Biliary atresia.  Carcinoma of head and periampullary region of thepancreas.  Choledochal cyst.  Cholangio carcinoma.  CBD stones.  Klatskin tumour (Carcinoma at the confluence of hepatic  Chronic pancreatitis ducts above the level of the cystic duct and so will cause hydro hepatosis without GB enlargement ).  Ascending cholangitis.  Extrinsic compression of CBD by lymph nodes or tumours.  Biliary strictures  Carcinoma gall bladder  Parasitic infestations

Signs and Symptoms of Pancreatic Cancer

FREQUENT INFREQUENT Pancreatic Head Cancers Weight loss (92%) Nausea (37%) Pain (72%) Weakness (35%) Jaundice (82%) Pruritus (24%) Dark urine (63%) Vomiting (37%) Light stool (62%) Anorexia (64%) Pancreatic Body or Tail Cancers Weight loss (100%) Jaundice (7%) Pain (87%) Dark urine (5%) Weakness (43%) Light stool (6%) Nausea (45%) Pruritus (4%) Anorexia (33%) Vomiting (37%)

Management of carcinoma head of pancreas:

Investigations:

Blood Tests:

 Patients with pancreatic head lesions frequently have elevated bilirubin and alkaline phosphatase levels suggestive of obstructive jaundice.  Other routine laboratory studies are usually normal.  Haemoglobin may be low.  The two most widely used pancreatic cancer serum markers are the CEA and the Lewis blood group carbohydrate antigen CA 19-9. Both are frequently elevated in patients with advanced disease. Extremely high levels of either CA 19-9 or CEA usually indicate unresectable or metastatic disease.

Imaging Studies:

 The main imaging modalities used for patients with suspected pancreatic neoplasms include right upper quadrant ultrasonography, CT, MRI with or without magnetic resonance cholangiopancreatography (MRCP), endoscopic retrograde cholangiopancreatography (ERCP), and percutaneous transhepatic cholangiography (PTC).  Right upper quadrant ultrasonography is a commonly used initial test. It can detect gallstones, dilatation of the biliary tree, and pericholecystic fluid. This imaging modality can also pick up hepatic metastases, pancreatic masses, peripancreatic and hilar lymphadenopathy, and ascites.  The investigation of choice in the work-up of patients suspected of a pancreatic cancer is a multidetector spiral CT and is probably the single most useful diagnostic and staging modality.

 CT provides more complete and accurate imaging of the pancreatic head and surrounding structures.  It gives very important information about the immediately adjacent vascular structures such as the portal, superior mesenteric, and splenic veins, as well as the superior mesenteric artery and celiac axis.  The involvement of periampullary lymph nodes and retroperitoneal structures may be demonstrated.  Additionally, information about distant metastatic disease can be gleaned if metastatic deposits are seen in the liver or in the peritoneal cavity.  The presence of ascites is usually an ominous sign.

 MRCP is now being utilized to image the biliary tree and the pancreatic duct. It has the advantage of being completely noninvasive.

 The vascular structures can also be visualized with the use of the contrast agent gadolinium and the performance of a magnetic resonance angiogram (MRA).  Thus a single (long) session in a scanner can provide information about tumor size and extent (MRI), the intraductal anatomy of the biliary and pancreatic system (MRCP), and the status of the nearby vasculature (MRA).  The resulting scan has the potential to provide information about tumor size and extent, biliary and pancreatic ductal anatomy, and vascular involvement through a single, noninvasive procedure.

 ERCP sometimes is required to solidify the diagnosis of pancreatic cancer.

 The classic findings of a long, irregular stricture in a pancreatic duct with distal dilation or a "double duct sign" in which there is cutoff of both the pancreatic duct and distal bile duct at the level of the genu of the pancreatic duct are pathognomonic  ERCP may be of benefit in patients with biliary obstruction and cholangitis whereupon an endoscopic stent can be placed for decompression.  ERCP is most useful when there is pancreatic duct obstruction, but no mass is evident on either CT or MRI. In this situation, it is necessary to try to distinguish chronic pancreatitis from pancreatic cancer.

 EUS is one of the most common imaging procedures used to diagnose pancreatic cancer.

 It is often the best procedure to obtain samples of a tumor to make a definitive diagnosis of pancreatic cancer.  EUS may be able to find small pancreatic masses that have not been detected by computed tomography (CT) or magnetic resonance imaging (MRI) scans but suspected by the doctor as a result of symptoms and/or blood test results.  Studies show that EUS is equal to or better than CT scans for detection of early pancreatic cancer.

 Biopsy: Biopsy to confirm the presence and identify the type of cancer is usually required before chemoradiation therapy of unresectable pancreatic tumors or neoadjuvant treatment of resectable tumors. Percutaneous biopsy, performed with either CT or ultrasound guidance, or transduodenal biopsy, performed with endoscopic ultrasound guidance, is routinely employed in these situations.

Treatment and prognosis  Most tumours are not resectable at diagnosis.  Surgery for stage I and II does offer the chance of cure, with however high morbidity (20- 30%) and mortality (5%). Resection is performed with a Whipple operation.  Even when resection is possible, the majority of patients succumb to recurrence, with only a doubling of survival in operated patients from 5 to 10% at 5 years. Almost a quarter of patients are dead 12 months following diagnosis.

Palliative Nonsurgical Treatment of Pancreatic Cancers

 Establishing the diagnosis and relieving symptoms of jaundice, gastric outlet obstruction, and pain are the goals of palliative nonsurgical treatment.  Tissue diagnosis can usually be made by CT- or ultrasound-guided percutaneous fine-needle aspiration of either the tumor or its metastases.  Transduodenal fine-needle aspiration of the tumor with endoscopic ultrasound guidance and duct cytology obtained by brushings is an alternative method of establishing the diagnosis.  Decompression of the obstructed biliary tract can be achieved using either an endoscopic or a percutaneous-transhepatic approach.  Pancreatic tumors can extend into and obstruct the duodenum, leading to gastric outlet obstruction. This commonly occurs in the second portion of the duodenum in patients with pancreatic head cancers. Pancreatic body tumors can invade the third or fourth portion of the duodenum and also cause obstruction. Many of these patients can be palliated by endoscopic placement of expandable endoluminal metal stents into the duodenum.  . Pain, which is a common symptom of pancreatic cancer, is usually caused by tumor invasion of the peripancreatic neural plexus. Most patients can be adequately treated with orally or transcutaneously administered analgesics.  Narcotic medications may be required. When or if this fails, percutaneous CT-guided or endoscopic ultrasound-guided celiac plexus block may be helpful.

Palliative Surgical Management of Pancreatic Cancer

 Most of the symptoms experienced by patients with unresectable pancreatic cancer can be relieved by nonsurgical means.  Surgical palliation is, for the most part, employed for patients who are undergoing laparotomy for anticipated resectable disease and found to be unresectable at the time of surgery.  In that situation, biliary tract decompression can be achieved by creating either a cholecystojejunostomy or a choledochojejunostomy.  Duodenal obstruction can be managed by creation of a side-to-side gastrojejunostomy in which an antecolic jejunal loop is anastomosed to the posterior wall of the gastric antrum.  Palliation of pain can be achieved, intraoperatively, by injecting alcohol into the celiac plexus, and some surgeons routinely perform operative celiac plexus block at the time of surgical palliation.  However, many patients with unresectable pancreatic cancer can be successfully managed with minimal or no narcotic analgesics, and when more severe pain occurs, results similar to those achieved by intraoperative chemical splanchnicectomy can be achieved using a percutaneous approach.

Chemoradiation Therapy

 Many different protocols for chemoradiation treatment of recurrent or unresectable pancreatic cancer have been described.  The best results have been achieved using radiation therapy combined with either 5- fluorouracil or gemcitabine.

Group – C

3. Write short notes on (any two): 2x5 = 10 a) Haemangioma. b) Carotid body tumour. c) Branchial sinus. d) Carcinoid tumour.

Answer. a) Haemangioma.

Definition: A benign skin lesion consisting of dense, usually elevated masses of dilated blood vessels.

 The hemangioma is a true vascular tumor that results from a overgrowth of normal vascular tissue.  It exhibits relatively rapid early growth until approximately 6 to 8 months of age (proliferative phase), followed by regression by 5 to 9 years of age (involutory phase).  It grows by “endothelial proliferation”. During the rapid growth phase, an increased number of mast cells is seen within the endothelial wall.  It is compressible.

Types of haemangioma:

 Capillary Hemangioma: o Salmon patch:  These are very common and occur in about 40% of all newborns.  They are usually small flat patches of pink or red skin with poorly defined borders.  They are commonly found at the nape of the neck (stork bite), on the forehead between the eyebrows (angel's kiss) or on the eyelids.  They become more intense in colour and noticeable when the child is crying.  Most lesions will spontaneously disappear within the first year of life.  Stork bites tend to be more persistent and may remain unchanged into adult life in 50% of cases. o Port wine stain:  Persists throughout the life.  A port wine stain is usually a large flat patch of purple or dark red skin with well-defined borders.  At birth the surface of the port-wine stain is flat, but in time it becomes bumpy and often more unsightly.  The face is most commonly affected although they can occur anywhere on the body.  Where present, they generally appear on one side of the body with a sharp mid- line cut-off. o Strawberry angioma:  Strawberry red mark found on 1 out of 10 babies  Small as a freckle or large as a coaster  Consists of small closely packed blood vessels  95% disappear by the time the child is 10 years old  Cavernous (Deep) Hemangioma:  Deeply situated red-blue spongy mass of tissue filled with blood found on 2 out of 100 babies  Grows rapidly in the first six months  Composed of larger, more mature vascular elements  Some of these lesions disappear on their own

 Compound Hemangioma:  Contains both superficial and deep parts  These are often the largest and the most spreading  Similar characteristics to both the strawberry hemangioma and the cavernous hemangioma

Treatment should be considered if the hemangioma:

 Ulcerates  Bleeds  Causes functional impairment  Causes infection  Grows rapidly and uncontrollably  Causes psychological problems

Treatment options:

 Medical

o Steroid injection o Interferon alfa-2a

Surgical

o Resection o FPDL o YAG laser b) Carotid body tumour.

Introduction:

 Also known as paraganglioma  Rare tumors that arise from specialized neural crest cells associated with autonomic ganglia.

Four extradrenal locations

Group I: Great vessels of chest and neck

GroupII: Vagus nerve

Group III: Aorticosympathetic chain

Group IV: Visceral organs

Head and Neck

 Carotid body paraganglioma  Vagal paraganglioma (nodose ganglia)  Glomus tympanicum - middle ear along tympanic plexus  Glomus jugulare - jugular bulb

Aetiopathology:

 1/30,000 head & neck tumors are paragangliomas  2-3% head/neck paragangliomas have functional hormone secretion  Usually benign- 6% CBTs reported to be malignant  Familial form (10-25%) – present younger and with multiple tumors

Presentation:  Average age = 45  Slow growing  Asymptomatic or mass-related effects  10% present with CN palsy

Imaging studies: CT MRI MRA USG Angiography

 Thin section scanning  Aids in lesion  Noninvasive  Delineates tumor  Demonstrates the from thoracic inlet to diagnosis and  Delineates margins, size and primary arterial skull base in patients localization displacement location supply and with CB or vagal  Differentiates of vasculature  Doppler: collateral vessels paragangliomas or mass from  Demonstrates demonstrates of tumor other palpable neck surrounding tumor hypervascularity  Reveals mass inflammatory vascular of paragangliomas relationships with  Examines integrity of changes, fluid or supply.  Surveys neck for neck blood associated soft tissues vascular other lesions vessels  Detection of multiple structures  Differentiates  Presurgery lesions  More sensitive CBTs from transcatheter  3D reconstruction for delineating vascular anomalies arterial visualizes associated encroachment and embolization vasculature and encasement pseudoaneurysms of vessels  Can obtain US  Images middle guided fine needle ear structures aspiration and bony erosions  Coronal sequences

Radionuclide imaging: Pentetreotide = octreotide radiolabelled with 111 indium-DTPA binds somatostatin type 2 receptors in paragangliomas  Uses: follow recurrent disease, locates multiple lesions, detects familial paragangliomas

Treatment: The main treatment modalities are surgery, embolization and radiotherapy

c) Branchial sinus. Branchial anomalies: Embryology: Branchial anomalies result from improper development of the branchial apparatus Branchial apparatus develops 2nd-6 th week Neck is shaped like a hollow tube with circumferential ridges = Arches (mesoderm) Ridges between arches = Clefts and Pouches Clefts = outside (ectoderm) Pouches = inside (endoderm) Arches: o Each arch: contains Cartilage, Cranial nerve, Artery, Muscle component, All neural crest origin o 6 arches, only 5 form structures in humans 1, 2, 3, 4, and 6. 5th fails to develop.

Branchial Clefts and Pouches:

4 clefts and 4 pouches

5th and 6th contribute to the 4th

Clefts provide “covering” to structures of the corresponding arch and pouch

Branchial cleft anomalies:

1st Cleft: Cysts, EAC atresia or stenosis, pits of lower , preauricular sinuses or tags

2nd Cleft: Cysts ,Cervical sinuses

3rd Cleft: Cysts (rare): Thymic cysts

4th Cleft : Cysts (extremely rare) - Cysts on the Vagus nerve -> cough

Branchial Cleft Cysts:

o Results from failed obliteration of branchial clefts o 2-3% are bilateral o 2nd cleft cyst is the most common type o ~95% of cases

2nd Branchial Cleft Cyst:

o Most common branchial cyst o Presents as a mass just anterior and medial to the upper one third of SCM in the neck o Tract - Anterior neck -> Along carotid sheath -> Between external and internal carotid arteries -> superficial to CN IX and XII -> Opens into

Work-up

 Ultrasound: Round mass with uniform low echogenicity and lack of internal septations Advantages: No radiation, no sedation for children, low cost  CT; Homogeneous lesion with low attenuation centrally and a smooth enhancing rim. More radiation, higher cost, may require sedation (children)  MRI: Hypointense on T1 and hyperintense on T2 Advantages: No radiation Disadvantages: Sedation for children, very expensive  Fluroscopic fistulography or CT fistulography: Inject radiopaque dye into the fistula or sinus to delineate course  Barium swallow esophagography; Help locate fistula tract in type 3 and 4 anomalies  FNA: Usually only done if suspect cancer May cause cyst to collapse -> much harder to remove at time of surgery

Treatment – Infected Cyst;

o Antibiotics o Should cover respiratory flora and Staph aureus (broad spectrum) o Cover 2-4 weeks o Abscess- Consider needle aspiration to drain May work without causing as much scaring as I&D I&D: if needle aspiration doesn’t work Once infection cleared, operate

Treatment – Surgical: complete excision of the tract and cyst

Branchial cleft fistulas (BCF) originate from the 1st to 3rd branchial apparatus during embryogenesis of the head and neck. o Anomalies of the 2nd branchial cleft are by far the most commonly found. o They can be a cyst, a sinus tract or fistulas. o Fistulas (or sinus tract if they end blindly) display themselves as small cutaneous opening along the anterior lower third border of the sternocleidomastoid muscle, communicates proximally with the tonsillar fossae, and can drain saliva or a mucoid secretion. o Management consists of excision since inefficient drainage may lead to infection.

 1st BCF are uncommon, located at the angle of the mandible, and communicating with the external auditory canal. They have a close association with the fascial nerve.  3rd BCF are very rare, run into the piriform sinus and may be a cause of acute thyroiditis or recurrent neck infections. d) Carcinoid tumour.

Introduction:

 Carcinoid tumours, because of their neuroendocrine origin, have neurosecretory capabilities and hence the ability to secrete a variety of peptides and bioactive amines.  Chief among these are 5-hydroxytryptamine( 5-HT) or serotonin, histamine, norepinephrine, ACTH, etc.  They arise from cells of diffuse neuroendocrine system. These cells are found throughout the GI tract and bronchopulmonary system.  Carcinoid tumours occur most frequently in the GI tract, in bronchopulmonary system, and other locations like ovary, gallbladder, thymus, testis, cervix, breast etc. Pathology:  Grossly carcinoid tumours appear as solid, yellow-tan lesions. They are usually solitary, except in stomach and ileum where they may be multicentric.  They are often submucosal and they may cause ulceration. There is intense desmoplasia within and surrounding the tumour.  Histologically they appear as uniform, small, round cells with rare mitotic figures. Two types of silver staining are commonly used to identify neuroendocrine cells—argyrophil reactions and argentafin staining. Foregut and hindgut carcinoid are typically argyrophil and midgut (serotonin secreting) carcinoids are argentafin.  Immunohistochemical stains like chromogranins A, B, and C are reliable serum markers for detection of neuroendocrine tumours. About 80% of carcinoids will have elevated levels of chromogranin A. A high level of chromogranin A is a poor prognostic indicator in individuals with metastatic disease.

Clinical Presentation:  Carcinoid tumours are often discovered incidentally during surgery, endoscopic procedures or imaging studies.  They may also present either by causing symptoms due to mass effect or due to their secretory products.  Mass effect may occasionally cause pain but more commonly causes luminal obstruction (due to tumour growth or scarring).  Symptoms are often non specific consisting of malaise,vague abdominal pain or weight loss.  Carcinoid syndrome is an unusual presentation of GI carcinoids.  The classic product serotonin may produce diarrhea. ACTH producing tumours will cause Cushing’s syndrome, while gastrin producing tumours may cause ZE syndrome.

Classification:  Carcinoid tumours have been classified according to their embryologic region of origin: foregut, midgut, hind gut.  Foregut carcinoids include those arising in , stomach, pancreas, and duodenum.  Midgut carcinoids comprise of those arising in the distribution of superior mesenteric artery distribution, including appendix and ileum.  Hind gut carcinoids include those arising in the distribution of inferior mesenteric artery distribution including the rectum.

Esophagus  Esophageal carcinoids are extremely rare. Dysphagia is the most common presentation.  Mostly treated by esophagogastrectomy. Stomach  Stomach is the most common foregut location for carcinoid tumours.  Pernicious anaemia and chronic atrophic gastritis are risk factors for the development of gastric ECL cell tumour. This may be secondary to the hypergastrinemia caused by the absence of significant acid output in such patients.  They are much more common in those patients with MEN- type 1.  The average age of presentation of patients with gastric carcinoid tumours is 62 years.  Patients are often asymptomatic, carcinoids often found incidentally during upper GI endoscopy.  When symptoms occur, they are usually dyspeptic.  EUS can be helpful in defining the depth of invasion and providing deep submucosal  biopsy specimen. CT and MRI may be helpful in finding hepatic metastasis. All patients with gastric carcinoids have elevations of chromogranin A levels.  Management of gastric carcinoid tumours is controversial.  Carcinoids secondary to pernicious anemia/ atrophic gastritis and MEN- 1 tend to have a more benign course, with occasional spontaneous regression. Pancreas  The differentiation between pancreatic carcinoids and other neuroendocrine tumours (NETs) of pancreas is primarily a matter of definition.  Pancreatic carcinoids have been defined as a tumour with the histologic features of a neuroendocrine tumour and with obviously increased serotonin metabolism.  Pancreatic carcinoids tend to present late. Abdominal pain, diarrhea, and weight loss are the most common presenting symptoms.  Most patients presents with metastasis. Treatment is by surgical resection.  Pancreaticoduodenectomy is often required.  It has better survival rates than those of other pancreatic malignancies but worse than those of carcinoids in other sites. Duodenum  Carcinoids of duodenum represent approximately 2% of carcinoids.  Duodenal carcinoids most commonly discovered during endoscopy.  They rarely produce serotonin and therefore virtually no association with carcinoid syndrome. Half of the cases produce gastrin.  Ampullary carcinoids frequently present with jaundice. Approximately 25% of patients  with periampullary carcinoids have von Recklinghausen’s disease.  The risk of metastasis increases with increased size of the primary tumour.  The treatment of duodenal and ampullary carcinoid tumour is by resection. Small Intestine  Ileum is the most common site for carcinoid tumours, accounting for almost 30 % of all carcinoids.  Carcinoids form approximately 28% to 38% of all small bowel cancers.  Average age of presentation is 62 to 65 years. Small bowel carcinoids are multifocal in 25% of cases.  They present as vague abdominal pain, intermittent intestinal obstruction, infarction of small bowel due to superior mesenteric artery occlusion.  Overt carcinoid syndrome is uncommon as an initial presentation.  Carcinoid syndrome usually indicates hepatic or retroperitoneal metastasis and signifies an unfavourable outcome.  If found early, primary tumour and the associated lymphatics should be resected surgically. Appendix  It is a less common site of GI carcinoids.  The proportion of appendiceal tumours that are carcinoids is high.  Majority are less than 1 cm in size.  Frequency of distant metastasis increases with the size of the tumour.  They cause symptoms early due to appendiceal luminal obstruction with consequent appendicitis. Have a favourable prognosis.  Simple appendicectomy is indicated for tumours less than 1 cm size and a right hemicolectomy for tumours more than 2 cm. size. Colon  Colonic carcinoids account for less than 10% of carcinoid tumours.  They occur more commonly on right side, about two thirds involving ascending or proximal transverse colon.  They tend to present as larger lesions than most oher carcinoids, averaging around 5 cm.  Symptoms are usually due to bulky advanced lesions.  Advanced lesions may cause pain or colonic obstruction.  Surgical resection and lymph node clearance should be performed. No adjuvant treatment is effective. Five year survival is worst Rectum  Constitutes 19 % of all carcinoids.  More than 80 % are still localized at the time of diagnosis.  Appear as local nodules. Tumours may produce symptoms as bleeding, pain, or decreased stool caliber, weight loss, changing bowel habits, may be detected incidentally.  Carcinoid syndrome is distinctly uncommon due to rectal lesions.  Rectal carcinoids have been treated endoscopically, by local excision, (lesions less than 2 cm) and by radical excision with either a low anterior resection or APR (lesions larger than 2 cm).

Carcinoid Syndrome

Typical Carcinoid Syndrome: Most common clinical pattern, caused by metastatic midgut carcinoids. Serotonin is released into circulation. It is then converted into 5- hydroxyindoleacetic acid (5-HIAA),which is excreted in urine. Patients with typical carcinoid syndrome have elevated plasma and platelet serotonin and increased 5-HIAA in urine. Clinically it is characterized by flushing, diarrhea, non specific abdominal pain, bronchospasm, pellagra like skin lesions and progressive right heart failure.

Atypical Carcinoid Syndrome: Most often associated with foregut carcinoid tumours. It presents with a different pattern of flushing, it occurs again in head and neck region but it appears patchier. It also shows central clearing and increased pruritus. Asthma and/ or signs of peptic ulceration are occasional presentations due to increased systemic levels of histamine. Patients have high plasma levels of 5-HTP and normal levels of serotonin. Urinary levels of 5-HIAA are typically normal to slightly elevated.

Diagnosis: Based on typical clinical history, 24 hour urinary 5-HIAA levels.

Carcinoid Crisis:

 Acute exacerbation of carcinoid syndrome may occur in a number of situations but most commonly in the setting of surgical or anaesthetic stresses.  Patients with known carcinoid tumours should receive somatostatin analogs before surgery to suppress mediator release.  Symptoms like hyperglycemia, hypertension, bronchospasm are treated by routine medical means.

Imaging Studies  Visualization of primary carcinoid tumour is more difficult than identification of metastatic disease.  CT scan and MRI are non specific and often non –localizing. They are useful when lesions are more than 1 cm in size. May be used to identify metastatic disease.  Somatostatin Receptor Scanning using radiolabelled somatostatin analogs (Indium¹¹¹ labeled pentetreotide, I123 labeled tyrosine) is helpful in localizing primary occult tumours and in staging of patients with known metastatic disease.

Treatment of Metastatic Disease / Carcinoid Syndrome: Majority of such patients are not the candidates for curative treatment. The focus of therapy is palliation of symptoms.

1. Surgery: For hepatic metastasis: Resection , OLT. Cytoreductive therapy. Hepatic artery embolization and chemoembolization. 2. Immunotherapy: ? Interferon ( controversial). 3. Chemotherapy : Marginal benefit. 4. Pharmacologic Therapy: Mainstay of palliative therapy. . Octreotide, lantreotide(long acting)---- Somatostatin Analogs. . Methysergide, cyproheptadine, ketanserin, ondansetron--- serotonin receptor blockers. . Parachlorophenylalanine--- inhibitor of serotonin synthesis. 5. Radiotherapy: For palliation of metastasis to bone and central nervous system.

Group – D

4. Write briefly on (any five): 5x4 = 20 a) Exostosis . b) Brown tumour. c) Ewing’s sarcoma. d) Spondylolisthesis. e) Bone scan. f) TB spine.

Answer. a) Exostosis. Also known as Osteochondroma. See the Question 4.b of Group – D of Paper – I of 2014. b) Brown tumour.

Definition:

The brown tumor is a bone lesion that arises in settings of excess osteoclastactivity, such as hyperparathyroidism. It is not a true neoplasm, as the term "tumor" suggests; however, it may mimic a true neoplasm. Brown tumours are radiolucent on x-ray.

Pathology:

 Brown tumours consist of fibrous tissue, woven bone and supportingvasculature, but no matrix.  The osteoclasts consume the trabecular bone thatosteoblasts lay down and this front of reparative bone deposition followed by additional resorption can expand beyond the usual shape of the bone, involving the periosteum thus causing bone pain.  The characteristic brown coloration results from hemosiderin deposition into the osteolytic cysts.  Hemosiderin deposition is not a distinctive feature of brown tumors; it may also be seen giant cell tumors of the bone.  Brown tumors may be rarely associated with ectopic parathyroid adenomas. Epidemiology:  Age and gender have an effect on the incidence of these lesions; they are more prevalent in women than men (though still common in both genders), and they appear more frequently with age.  Due to the standard of medical care and screening in developed countries, it is increasingly rare for primary hyperparathyroidism to present with accompanying bone disease.  This is not the case in less developed nations, however, and the two conditions are more often seen together. c) Ewing’s sarcoma. See the Question 4.e of Group – D of Supplementary Paper – I of 2014 d) Spondylolisthesis. Definition: Spondylolisthesis is the forward displacement of a vertebra, especially the fifth lumbar vertebra, most commonly occurring after a break or fracture. Backward displacement is referred to as retrolisthesis. A hangman's fracture is a specific type of spondylolisthesis where the secondcervical vertebra (C2) is displaced anteriorly relative to the C3 vertebra due tofractures of the C2 vertebra's pedicles. Classification: Spondylolisthesis is categorized into five types:

 Isthmic spondylolisthesis is the most common form; also called spondylolytic spondylolisthesis.  Degenerative spondylolisthesis is a disease of the older adult that develops as a result of facet arthritis and joint remodeling.  Traumatic spondylolisthesis is rare and results from acute fractures in the neural arch, other than the pars.

 Pathologic spondylolisthesis has been associated with damage to the posterior elements from metastases or metabolic bone disease. These slips have been reported in cases of Paget's disease of bone, tuberculosis, giant-celltumors, and tumor metastases.  Dysplastic spondylolisthesis is a rare congenital spondylolisthesis occurring because of a malformation of thelumbosacral junction, resulting in small, incompetent facet joint.

Signs and symptoms: Symptoms of spondylolisthesis include:  A general stiffening of the back and a tightening of the hamstrings, with a resulting change in both posture and gait.  A leaning-forward or semi-kyphotic posture may be seen, due to compensatory changes.  A "waddle" may be seen in more advanced causes, due to compensatory pelvic rotation due to decreased lumbar spine rotation.  A result of the change in gait is often a noticeable atrophy in the gluteal muscles due to lack of use.  Generalized lower-back pain may also be seen, with intermittent shooting pain from the buttocks to the posterior thigh, and/or lower leg via the sciatic nerve.  Other symptoms may include tingling and numbness. Coughing and sneezing can intensify the pain. An individual may also note a "slipping sensation" when moving into an upright position. Sitting and trying to stand up may be painful and difficult. Treatment:

Conservative management: Patients with symptomatic isthmic spondylolisthesis are initially offered conservative treatment consisting of activity modification, pharmacological intervention, and a physical therapy consultation.

 Physical therapy can evaluate and address postural and compensatory movement abnormalities.  Anti-inflammatory medications (NSAIDS) in combination with acetaminophen (Tylenol) can be tried initially. If a severe radicular component is present, a short course of oral steroids can be considered. Epidural steroid injections, either interlaminal or transforaminal, performed under fluoroscopic guidance can help with severe radicular (leg) pain. Lumbosacral orthoses may be of benefit for some patients but should be used on a temporary basis to prevent spinal muscle atrophy and loss of proprioception.

Surgical: Degenerative spondylolisthesis with spinal stenosis is one of the most common indications for spine surgery among older adults e) Bone scan.

Introduction: Skeletal scintigraphy is performed by injecting a bone seeking radioactive tracer with the aid of gamma camera. More sensitive than conventional X-ray procedure.

Radiopharmaceuticals:  Technetium(99mTc) - a gamma ray emitting radionuclide is labelled with phosphate or phosphonate(MDP,HDP) which has affinity to bone  Intravenous injection,dose  Adults (15-25mCi). Children's depends on weight & age  Imaging time 2- 3 hours delayed after I.V

PatientPreparation:  No restrictions to eat,drink or to drive. Intake of more fluid after Injection  Frequent voiding is advised  Instruct about the procedure to the patient  Remove all the external metallic objects and prosthesis before to scan Indications:  Rule out of bony metastases, staging, Pre & Post palliative, chemo and radio therapy.  Avascular necrosis, Infection and Inflammation, micro fracture, bonegraft perfusion,biopsy site.  Metabolic bone disease, arthritis, gout

Imaging Techniques-Planar & Spect :  Imaging Techniques-Planar & Spect Gamma Camera fitted with High Resolution collimator  Whole body imaging : Scan speed = 11 cm/min  Spot views : Time mode: 5 min & in count mode : 600kcts  Blood pool image at 5 min post injection is acquired to r/o infection.  SPECT  Gamma camera rotates clockwise or counter in order to obtain 3D images. The information acquired from each projection is translated in to images by suitable processing methods.  Time mode : 30 sec/projection

Radiation Dosimetry: Estimated Absorbed Radiation Doses (MIRD) ADULT Conclusion:  Sensitivity is more but Specificity is less  Radiation dose is less when compare to X-rays.  Contra indication unknown except pregnancy  No Adverse reactions but there is a least chance for hypersensitivity reactions

f) TB spine.

Introduction:

 Spinal TB makes up a relatively small number of TB patients  Presentation can be insidious  Outcome variable and despite treatment can lead to long term disability

Pathology:

 Haematological seeding from lung  MTB affects vertebral body which can then spread to discs (discitis)  Occurs at any level and can be multilevel  More common in thoracic and lumbar  Can lose complete vertebrae  Wedge shaped fractures are common  Approx 1/3 have evidence of TB elsewhere  Approx 1/3 have associated psoas abscess  Only 25% have an abnormal CXR

Symptoms:

 Back pain (95%)  40-50% neurological symptoms –weakness, paresthesia, bowel symptoms  40-50% with systemic symptoms –fever, night sweats, weight loss

Difficulty in diagnosis:

 Back pain is very common  Systemic symptoms are often ignored by patient or forgotten by health professionals

Investigations:

 Standard bloods plus HIV  IGRA / Mantoux  CXR  Samples for AFB

Imaging:

 MRI  CT guided biopsy  US guided drainage of collection - Samples sent for AFB

Medical treatment:

 Standard quadruple therapy  BTS recommend 6 months  Most experts give 9-12 months

Indications of surgical treatment:

 Neurological deterioration  Spinal instability  Decompression of abscesses not amenable to radiological drainage  Post tubercular kyphosis

The West Bengal University of Health Sciences

M.B.B.S. 3rd Professional Part – II Examination, 2009

1 Subject: Surgery Time: 2 /2 hrs.

Paper: I Marks: 60

Group –A

1. Define and Classify wounds? Discuss various factors influencing wound healing. Discuss in brief management of a “Diabetic foot”. 5+5+5 = 15

Answer. See the Question 1 of Group – A of Paper – I of 2011.

Group – B

2. Answer any of the following questions: a) A middle aged male patient presents with an epigastric lump. Discuss the differential diagnosis. How would you investigate to come to a diagnosis? 8+7 = 15

Answer.

Differential diagnosis:

 Parietal swelling: o Epigastric hernia- A small round swelling exactly in the midline anywhere between the xiphisternum and umbilicus. In the first stage, it is sacless herniation of the extraperitoneal fat through a weak spot in the linea alba. There is no symptom at this stage. In the second stage, a pouch of peritoneum is drawn after it. In the last stage, a small tag of omentum gets into the sac and becomes adherent to it. At this stage the patient complains of dragging pain, discomfort or pain after food, not unlike those in peptic ulcer. o Lipoma.  Intra-abdominal swellings:- They occur in connection with the: o Liver and Subphrenic abscess o Hepatocellular carcinoma o Stomach and Duodenum  Congenital pyloric stenosis- Babies about 2 to 4 weeks old when present with projectile vomiting after meals, the diagnosis becomes obvious. On examination visible peristalsis of the stomach is always seen. Sometime a definite lump may be felt at the pylorus of the stomach.  Gastric outlet obstruction due to peptic ulcer  Carcinoma of the stomach o Transverse colon-  Intussusception: In intussusception there will be emptiness at the right iliac fossa. The patient complains of colicky pain, a lump in the epigastrium and "red current jelly" in the stool.  Diverticulitis: In inflammatory conditions a tender and irregular mass may be felt.  Hyperplastic tuberculosis  Neoplasms: In carcinoma the swelling is the presenting symptom. The swelling is irregular, hard and may be mobile above downwards and very slightly in the sideways or may be fixed. Anaemia, anorexia and occult blood in the stool are the features which helps in the diagnosis. Barium enema X-ray reveals constant 'filling defect' of the colon.  Omentum: In the tubercular peritonitis, the omentum is rolled up to form a transverse ridge in the epigastrium. Enlarged lymph nodes or adherent coils of intestine are also come across in this condition.  Pancreas: It hardly gives rise to a palpable swelling. The condition that forms lump in connection with this organ is the pseudocyst. True cyst of the pancreas is extremely rare. The pseudocyst is a collection of fluid in the lesser sac of the peritoneal cavity resulting from acute pancreatitis or trauma. It forms a smooth rounded swelling with fluctuation test positive. X-ray with barium meal will show the exact position of the swelling which is situated behind the stomach and is best seen in the lateral X-ray.  Abdominal aorta: Aneurysm of this part of the aorta is not uncommon. It presents a swelling in the epigastrium with characteristic expansile pulsation.  Lymph nodes: In addition to the usual causes of enlargement of lymph nodes, the followings are more important in this region: (i) tabes mesenterica, (ii) lymphosarcoma and (iii) secondary malignant growth from the neighbouring organs and also from the testis.  Retroperitoneal sarcoma and teratoma are the two conditions commonly seen in the posterior abdominal wall arising from the retroperitoneal tissue.

Investigations:

Laboratory investigations:

 Complete haemogram: In case of malignancy anaemia may be present.  Amylase and lipase: may be raised in pseudocyst pancreas  Tumour markers: AFP, Beta HCG, LDH for testicular cancer  Liver and kidney function tests

Imaging studies:

 Early USG and CECT abdomen: they can tell about origin, extent and number of abdominal lump. CECT can help in the diagnosis of pseudocyst pancreas, any retroperitoneal mass, diverticulitis and abdominal aortic aneurysm.  Barium enema: Used for colonic growth  CT and MRI: They can detect colonic mass, aortic aneurysms and lymphnode.  CT and MR Angiography: They can detect any vascular lesion.

Upper GI endoscopy: Used to detect lesions of stomach and duodenum. b) What are the causes of weeping umbilicus? Discuss the problem related to vitell - intestinal duct and their remedy. 5+5+5 = 15

Answer.

Causes of weeping umbilicus:

Urachal abnormalities Vitello Intestinal Duct Inflammatory and related abnormalities disorders

 Urachal Sinus  Umbilical Adenoma /  Umbilical granuloma  Urachal Diverticulum Polyp  Omphalitis  Urachal Cyst  Vitelline Sinus  Vesicoumbilical fistula  Umbilical fistula  Alternating sinus

Vitelline Duct is connection between midgut and yolk sac

 Usually involutes in 7th – 9th weeks

Treatment:

Symptomatic children with vitello-intestinal or omphalomesenteric duct remnants should be resuscitated before intervention. Those with significant haemorrhage should be transfused. Patients with obstructive symptoms should be resuscitated as rapidly as possible to obviate the need for ischaemic bowel resection. The incision chosen varies with the symptoms and the age of the patient. Children with faeculent umbilical drainage or prolapse of the omphalomesenteric duct remnant can be explored by a small infraumbilical incision.

When connected to the ileum, a patent vitello-intestinal duct may have a wide lumen or be a high-output fistula. This may lead to fluid and electrolyte deficits, especially in neonates, and treatment should be expeditious. The treatment of a patent vitello-intestinal is wedge or segmental resection.

Children with Meckel’s diverticulitis or a bleeding Meckel’s diverticulum are operated on by using a transverse appendectomy incision with medial extension if necessary. Patients with suspected intestinal obstruction should be explored through a generous laparotomy incision. An open diverticulectomy includes the following steps:

1. A transverse appendectomy incision or subumbilical incision is made.

2. The caecum and ileum are identified. 3. The ileum is followed proximally to find Meckel’s diverticulum, approximately 60 cm from the ileocaecal valve.

4. The diverticulum with the ileum are delivered into the wound.

5. The diverticulum is excised with the adjacent ileum and primary ileal end-to-end anastomosis is fashioned

Group – C

3. Write short notes on (any two): 2x5 = 10

a) Arterio-venous fistula. b) Basal cell carcinoma. c) Preoperative preparation of a patient of Pyloric stenosis. d) Auto transfusion.

Answer.

a) Arterio-venous fistula.

Introduction:

An arteriovenous fistula is an abnormal connection or passageway between an artery and a vein. It may be congenital, surgically created for hemodialysis treatments, or acquired due to pathologic process, such as trauma or erosion of an arterial aneurysm.

Typical AV Fistula

“Venous” needle

Thrill “Arterial” needle

Typical AV Fistula

Venous pressure

Arterial pressure

Problems with A-V fistula:

 Poor development

 Difficulty needling

 Inadequate dialysis

– Low flow rates

– Recirculation

 High pressures and prolonged bleeding

 Thrombosis

 Almost all due to stenoses which are recurrent

Treatment:

• Angioplasty

– Standard balloon angioplasty

– High pressure balloon

– Cutting or scoring balloon

• Surgery – Refashion anastomosis

– Patch stenosis

– Higher fistula

b) Basal cell carcinoma.

Epidemiology and aetiology:

 >400,000 new patients annually

 Age usually over age 40

 Sex Males >Females

 Race rare in brown and black skinned patient.

Clinical features:

 Pearly papules or  Telangiectases  Crusting  Locally malignant nodules  Rolled border  Central ulcer  Bleed easily  Rodent ulcer

Sites: Tear drop cancer

 Head, neck  Forehead 

 Nose  Ears  Upper trunk

Biopsy:

 Biopsy: Shave, Punch,Excision

 Specimen to reliable dermatopathologist or pathologist

Treatment:

 Electrodesiccation and  Excision  Moh’s Surgery  5-Fluorouracil curettage  Cryosurgery  Radiation  Aldara (Imiquimod)

c) Preoperative preparation of a patient of pyloric stenosis. See the Question 1 of Group –A of Supplementary Paper – I of 2013. d) Auto transfusion

Definition:

Autotransfusion is a process wherein a person receives their own blood for a transfusion, instead of banked allogenic (separate-donor) blood.

There are two main kinds of autotransfusion: Blood can be autologously "pre-donated" (termed so despite "donation" not typically referring to giving to one's self) before a surgery, or alternatively, it can be collected during and after the surgery using an intraoperative blood salvage device (such as a Cell Saver).

Advantages:

 Immediate availability  Normothermic  Conserves allogeneic blood for real  pH relatively normal emergencies  Eliminates clerical errors  Lower risk of infectious diseases  Is truly cost effective  Functionally superior cells  High levels of 2,3-DPG  Lower potassium (compared to stored blood)

Autotransfusion triggers:

 Anticipated blood loss is equal to or greater than 1000 ml

 Procedures where 2 units of blood are routinely cross matched

 Procedures where 20% of the patients are routinely transfused

 Emergency procedures

 Patients with rare blood types or incompatibilities

 Patients with religious objections to allogeneic blood components.

Indications for use:

• Start with a “stand-by” setup to include a collection reservoir, reservoir connector, suction/anticoagulant line and anticoagulant • Cost of the “stand-by” setup can be less expensive than cross-matching two (2) units of allogeneic blood

• If major blood loss is certain, collection and processing of recovered blood can occur

• Applicable in open-heart, vascular, total joint, spinal, liver transplant, ruptured ectopic pregnancy and trauma surgeries.

Disadvantages:

 The disadvantage of autotransfusion is the depletion of plasma and platelets. The washed autotransfusion system removes the plasma and platelets to eliminate activated clotting factors and activated platelets which would cause coagulopathy if they were reinfused to the patient, generating a packed red blood cell (PRBC) product. This disadvantage is only evident when very large blood losses occur. The autotransfusionist monitors blood loss and will recommend the transfusion of fresh frozen plasma (FFP) and platelets when the blood loss and return of autotransfusion blood increase. Typically the patient will require FFP and platelets as the estimated blood loss exceeds half of the patient's blood volume. When possible diagnostic tests should be performed to determine the need for any blood products (i.e. PRBC, FFP and platelets).

Contraindications:

The use of blood recovered from the operative field is contraindicated in the presence of bacterial contamination o rmalignancy. The use of autotransfusion in the presence of such contamination may result in the dissemination of pathologic microorganisms or malignant cells. The following statements reflect current clinical concerns involving autotransfusion contraindications.

Group – D

4. Write briefly on (any five): 5x4 = 20 a) Myositis ossificans. b) Pathological fracture. c) Carpal tunnel syndrome. d) Fracture neck femur – types and complications. e) Aetiopathogenesis of acute osteomyelitis. f) Spina bifida

Answer. a) Myositis ossificans. Myositis ossificans (MO) is a benign process characterised by heterotopic ossificationusually within large muscles. Its importance stems in large part from its ability to mimic more aggressive pathological processes.

Epidemiology:  Most cases of myositis ossificans occur as a result of trauma, and thus the main demographic is young adults.  Another group which are particularly prone to myositis ossificans are paraplegics, usually without evidence of trauma.

Clinical presentation  Typically presents as a painful, tender, enlarging mass, which in 80% of cases is located in large muscles of the extremities, often following recognised local trauma, although a definite traumatic event is not always recalled.  In the case of paraplegics, recognised episodes of trauma are often absent, and the disease occurs particularly around the knees and hips.

Pathology Myositis ossificans is essentially metaplasia of the intramuscular connective tissue resulting in extraosseous bone formation (without inflammation).

It has a zonal organization:

 Peripheral well organized mature lamellar bone  Intermediate osteoid region  Central immature non-ossified cellular (fibroblasts) focus Unfortunately the histologically of myositis ossificans can appear similar toosteosarcoma, and thus can actually lead to inappropriate management.

Radiographic features The typical radiographic appearance of myositis ossificans is circumferential calcification with a lucent centre, and a radiolucent cleft (string sign) that separates the lesion from the cortex of the adjacent bone.

Radiograph Calcification usually begins to become apparent on plain radiographs within 2-6 weeks, and the lesion reaches the classic well circumscribed peripherally calcified appearance by 2 months. Over the following 4 or so months, they typically become smaller and denser.

Cleft between it and the subjacent bone may be difficult to see on plain radiographs.

CT CT appearances are similar to those of plain radiography, demonstrating mineralisation proceeding from the outer margins towards the center. The cleft between it and the subjacent bone is usually visible. The peripheral rim of mineralisation is usually visible within 4-6 weeks.

MRI MRI appearances change with the age of the lesion.

Early features can be misleading because the peripheral calcification is not well seen, and oedema in the soft tissues may extend beyond the often inapparent calcific rim.

Nuclear medicine Non-specific increased uptake on flow and blood pool images is seen early on in the development of the lesion, and gradually decreases as the lesion matures.Increased uptake on delayed phase is typical.

Treatment and prognosis Myositis ossificans is benign and there is no compelling evidence that malignant degeneration ever occurs. As such treatment is reserved for symptomatic lesions, and surgical resection is usually curative.

Differential diagnosis Imaging differential considerations include:

 Parosteal osteosarcoma: calcifies in centre and continues towards the periphery  Soft tissue sarcomas including o Malignant fibrous histiocytoma o Synovial sarcoma b) Pathological fracture.

Definition:

A pathological facture is one in which a bone is broken, through an area, weakened by pre- existing disease, by a degree of stress, that would have left the normal bone intact. In other words, a fracture involving “abnormal bone” is a pathological fracture.

Etiology:

 Development disorders of bone : o Congenital defects of bone tissue : Osteogenesis imperfecta Osteopetrosis o Disorder of cartilage growth : Achondroplasia Diaphysealaclasis (multiple exostosis) Dyschondroplasia (Ollier’s disease) o Nutritional and vitamin deficiencies: Scurvy, Rickets, Osteomalacia  Hormonal imbalance : o Hyperparathyroidism ,Cushing’s syndrome. o Pathological fracture from cortisone treatment Frohlich’s syndrome (hypopituitarism)  Atrophic conditions of bone : Disuse osteoporosis Senile osteoporosis  Pathological fracture through infected bone : Osteomyelitis  Cystic disorders and fibrous dysplasia of bone : o Unicameral bone cyst o Aneurysmal bone cyst o Non – osteogenic fibroma of bone o Monostotic and polyostotic fibrous dysplasia o Paget’s disease of bone  Primary and secondary tumors of bone: o Primary benign tumours: Chondroma, Benign chondroblastoma, Chondromyxoid fibroma, Haemangioma of bone ,Giant cell tumour of bone ,Disappearing bone disease. o Malignant tumours : Osteosarcoma,Chondrosarcoma ,Fibrosarcoma,Malignant fibrous histiocytoma, Malignant round cell tumour, Multiple myelomatosis o Metastatic tumours of bone – lungs, thyroid, kidney, GI tract, prostrate  Marrow cell disorder: Histiocytos Gaucher’s disease  Parasitic disease of the bone : Hydatid disease  Neurotrophic dystrophies of the bone : Tabes dorsalis ,Syringomyelia, Diabetic neuropathy  Iatrogenic pathological fracture : o Through screw hole stress protection phenomenon o Through biopsy o After removal of infected bone o Through a donor site for a bone graft

Pathological fracture is suspected when fracture occurs:

 Spontaneously  After minor trauma  Unusual fracture pattern  History of recent several fractures  Older patient  History of primary malignancy

Investigations:

 Radiography:

Plain –X-ray:

 Study the Fracture  Lesion location: Usually eccentric, Cortical involvement, Diaphyseo-metaphyseal junction  Don’t ignore the perifracture changes  Densities within the lesion: Bone formation suggests – Osteosarcoma;Calcification suggests – Chondrosarcoma  Other lesions  Reaction (periosteal / endosteal) should be examined.  Alteration in density and architecture  Zone of transition  Extra osseous masses or abnormalities  Moth eaten or permeative pattern of bone reaction

Laboratory studies:

General investigations Search for occult primary carcinoma

 Complete haemogram o Breast - Examination  Peripheral smear Mammography  Serum glucose o Lung – Chest X-ray  Serum albumin o Kidney – Ultrasonography o  Serum calcium, Phosphate Thyroid – Digital palpation o  Alkaline phosphatase Prostate – Serum PSA, Digital prostate examination  LFT o Myeloma Bone marrow  Urine sugar and albumin – examination; Bence-Jones Bence-Jones proteins  proteins; Serum and urine  Serum electrophoresis electrophoresis  Tumor markers: Ca 125, Ca 19.9, CEA o Skeletal Survey - X-ray skull, spine and pelvis. o Other organs

MRI:

 Marrow disease  Epidural and nerve root compression can be detected  Localize the disease

Bone scan

PET scan

Treatment:

o Management of pain o Avoiding the fracture o Bone stabilization o Conservative measures Systemic Therapy:

 Hormone therapy : Ca breast and prostate  Chemotherapy  Bisphosphonates : Zolindronic acid  Targeted therapy : Denosumab

Radiotherapy :

 External-beam radiotherapy  Stereotactic Body Radiotherapy(SBRT)  Radiopharmaceuticals : Unsealed source therapy with bone-seeking radionueclides

Surgical management:

 Indication: Palliative

Fracture:

Ambulation / Pain relief

Impending fracture c) Carpal tunnel syndrome. See the Question 4.d of Group – D of Supplementary Paper – I of 2014 d) Fracture neck femur – types and complications.  Neck of femur fractures are considered intracapsular fractures (also called proximal femoral fractures).  Intracapsular fractures include:

o Subcapital: femoral head/neck junction o Transcervical: midportion of femoral neck o Basicervical: base of femoral neck Further, severity of a subcapital fracture is graded by the Garden classification of hip fractures.

The Garden classification of proximal femoral fractures is the most widely used, and is useful as it is both simple and predicts the development of AVN.

o Garden stage I: undisplaced incomplete, including valgus impacted fractures. o Garden stage II : undisplaced complete o Garden stage III : complete fracture, incompletely displaced o Garden stage IV : complete fracture, completely displaced

Complications:

 Osteonecrosis:

o Incidence of 10-45% o Recent studies fail to demonstrate association between time to fracture reduction and subsequent avn o Increased risk with . Increase initial displacement . Avn can still develop in nondisplaced injuries . Nonanatomical reduction o Treatment . Major symptoms not always present when avn develops . Young patient . > 50% involvement then treat with fvfg vs tha . Older patient . Prosthetic replacement (hemiarthroplasty vs tha)

 Nonunion:

o Incidence of 5 to 30% . Increased incidence in displaced fractures . No correlation between age, gender, and rate of nonunion o Varus malreduction most closely correlates with failure of fixation after reduction and cannulated screw fixation. o Treatment: . Valgus intertrochanteric osteotomy . Free vascularized fibula graft (FVFG) . Arthroplasty . Revision ORIF

 Dislocation:

o Higher rate of dislocation with THA (~ 10%) . About seven times higher than hemiarthroplasty

 Mortality and Morbidity

o Pre-injury mobility is the most significant determinant for post-operative survival o Mortality rates of 14-36% at one year in the geriatric population . Mortality rates equal to age-matched population thereafter o Functional assessment during recovery can be predictive of persistent use of ambulatory devices versus return to near-baseline function . The timed get up and go test predicts use of ambulatory aids at 2 years post-op e) Aetiopathogenesis of acute osteomyelitis.  Most common type of bone infection, usually seen in children  Decrease in incidence, could be due to higher standard of living and improved hygiene.  Bimodal distribution- younger than 2 years, and 8-12 years  More common in males  Caused by a bacteraemia  Bacteriological seeding of bone generally is associated with other factors such as localized trauma, chronic illness, malnutrition or an inadequate immune system.  Adults with increased risk o Vascular disorders o Genitourinary and respiratory infections o Spread infection from blood to bone o Vascular-rich bone sites . Pelvis . Tibia . Vertebrae

f) Spina bifida. See the Question 4.e of Group – D of Paper – I of 2014

The West Bengal University of Health Sciences

M.B.B.S. 3rd Professional Part – II Examination, 2008

1 Subject: Surgery Time: 2 /2 hrs.

Paper: I Marks: 60

Group –A

1. Define and classify shock. How will you assess and treat a case of haemorrhagic shock? Mention the complications of blood transfusion. 2+2+5+3+3 = 15

Answer. See Question 1 of Group – A of paper – I of 2014 See Question 1 of Group – A of Supplementary paper – I of 2014 See Question 1 of Group – A of paper – I of 2012

Group – B

2. Answer any of the following questions:

a) Enumerate the causes of bleeding per rectum. Mention how it is diagnosed? Outline the management of bleeding haemorrhoids. 5+5+5 = 15

Answer. See Question 2.b of Group – B of Paper – I of 2014.

Classification:

 Grade I - bleeding without prolapse.

 Grade II - prolapse with spontaneous reduction.

 Grade III - prolapse with manual reduction.

 Grade IV - incarcerated, irreducible prolapse.

Treatments are classified into three categories:

1) Dietary and lifestyle modification.

2) Non operative/office procedures.

3) Operative hemorrhoidectomy.

 Dietary and lifestyle modifications:

o The main goal of this treatment is to minimize straining at stool. o Achieved by increasing fluid and fiber in the diet, recommending exercise, and perhaps adding fiber agents to the diet such as psyllium. o If necessary, stool softeners may be added.  Office Treatments: o Rubber band ligation:  Grade I or Grade II hemorrhoids and, in some circumstances, Grade III hemorrhoids.  Complications include bleeding, pain, thrombosis and life threatening perineal sepsis.  Successful in two thirds to three quarters of all individuals with first and second degree hemorrhoids. o Infrared coagulation:  Generates infrared radiation which coagulates tissue protein and evaporates water from cells.  Is most beneficial in grade i and small grade ii hemorrhoids. o Bicap electrocoagulation:  It works, in theory, similar to photocoagulation or to rubber banding.  The probe must be left in place for ten minutes.  Poor patient tolerance minimized the effect of this procedure. o Sclerotherapy:  Injection of an irritating material into the submucosa in order to decrease vascularity and increase fibrosis.  Injecting agents have traditionally been phenol in oil, sodium morrhuate, or quinine urea.  Surgical Treatment of Hemorrhoids: o Haemorrhoidectomy:  The triangular shaped hemorrhoid is excised down to the underlying sphincter muscle.  Wound can be closed or left open  Stapled hemorrhoidectomy has been developed as an alternative to standard hemorrhoidectomy

b) What are the causes of obstructive jaundice? How do you establish the diagnosis? Discuss the various options in the management of choledocholithiasis. 5+5+5 = 15

Answer. See Question 2.b of Group – B of Paper – I of 2010.

See Question 2.b of Group – B of Paper – I of 2013.

Group – C

3. Write short notes on (any two): 2x5 = 10 a) Methods of sterilization. b) Biochemical abnormality of pyloric stenosis. c) Universal precaution. d) Nipple discharge.

Answer. a) Methods of sterilization.

Definition: It is the process of destruction or removal of all microorganisms from article, surface or medium, including spores.

Classification of the method of sterilization/Disinfection

A. Physical

1- Sun Light

2- Drying

3- Heat: i- Dry

Ii-Moist

4- Filtration

5- Gas

6- Irradiation

7- Ultra sonic cleaning

B. Chemical

1. Phenol Derivatives : Phenol, Cresol, resorcinol, chloroxylenol

2. Oxidizing agents :Pot.Permanganate, Hydrogen Peroxide,Benzoyol Peroxide

3. Halogens : Iodine, chlorine

4. Biguanide : Chlorhexidine

5. Quarternary Ammonium (Cationic) : Cetrimide, Zephiran 6. Alcohols : Ethanol, Isopropanol.

7. Aldehydes : Formaldehyde, Glutaraldehyde

8. Acids : Boric acid, acetic acid

9. Metallic salts ; Silver Nitrate, Zince Sulfate, Zinc Oxide, calamine,

10. Dyes : Gentian violet, proflamine, Acriflamine

11. Furan derivatives : Nitro flurazone

Heat:

Most common and one of the most effective methods of sterilization. Factors influencing sterilization by heat are : -

i. Nature of heat

a. Dry

b. Moist

ii.Temperature & time

iii. No. of organism present

iv.Whether organism has sporing capacity

v. Type of material from which organism is to be eradicated

Hot air oven:

It is used to sterilize items, which do not get damaged by high temp. such as laboratory glass, flasks, instruments with sharp cutting edges, B.P. handles, Powders, Dapen dishes, mouth mirrors.

Autoclave:

Steam is the effective means of sterilization, because of its

1. High penetrating capacity.

2. It gives of large amount of heat to surface with which it comes in contact.

To achieve sterility, a holding time of at least 15 minutes at 121 °C at 15 lb / square inch pressure is required.

Irradiation: Radiation used for sterilization is of two types

1. Ionizing radiation, e.g., X-rays, gamma rays, and high speed electrons .

2. Non-ionizing radiation, e.g. ultraviolet light, and infrared light.

These forms of radiation can be used to kill or inactivate microorganisms.

Ethylene Oxide Sterilization (ETO):

 Ethylene oxide is a colorless liquid with a boiling point of 10.7 °C. Highly penetrating gas with sweet ethereal smell.

 Highly inflammable & in conc. greater than 3%, highly explosive.

 By mixing with inert gases such as CFC or CO2, explosive tendency is eliminated.

 Plastics, rubber & photographic equipments can be sterilized by this method.

 Also used for mass sterilization of disposable items, plastic syringes, needles, catheters, blades etc. b) Biochemical abnormality of pyloric stenosis. See Question 1 of Group –A of Supplementary Paper – I of 2013. c) Universal precaution.

Universal precautions refers to the practice, in medicine, of avoiding contact with patients' bodily fluids, by means of the wearing of nonporous articles such as medical gloves, goggles, and face shields. Under universal precautions all patients are considered to be possible carriers of blood-borne pathogens. The guideline recommends wearing gloves when collecting or handling blood and body fluids contaminated with blood, wearing face shields when there is danger of blood splashing on mucous membranes and disposing of all needles and sharp objects in puncture- resistant containers. Universal precautions are designed for doctors, nurses, patients, and health care support workers who are required to come into contact with patients or bodily fluids. This includes staff and others who may not come into direct contact with patients. Universal precautions are typically practiced in any environment where workers are exposed to bodily fluids, such as: . Blood . Synovial fluid . Pleural fluid . Semen . Amniotic fluid . Peritoneal fluid . Vaginal secretions . Cerebrospinal fluid . Pericardial fluid

Bodily fluids that do not require such precautions include: . Feces . Vomitus . Sputum . Nasal secretions . Perspiration . Saliva . Urine

Universal precautions are the infection control techniques that were recommended following the AIDS outbreak in the 1980s. Every patient is treated as if infected and therefore precautions are taken to minimize risk. Protective clothing includes but is not limited to:

. Barrier gowns . Gloves . Eyewear (goggles or glasses) . Face shields 1. Alt patients must be regarded as potentially infected with blood-borne pathogens. 2. Strict hand washing must be practiced before and after each patient contact. Hands must be washed immediately if they are contaminated with blood or body fluids 3. Gloves should be worn: A. If soiling with blood or body fluids is anticipated, b. For placement of intravenous lines. C. Gloves should not be reused. 4. Gowns or aprons should be worn during procedures that are likely to generate splashes of blood or other body fluids and if soiling with blood or body fluids is likely. 5. Masks and protective eyewear (goggles) should be worn during procedures that are likely to generate droplets or splashes of blood or other body fluids to prevent exposure of mucous membranes of the mouth, nose arid eyes. 6. Articles contaminated with blood or body fluids should be discarded: A. If disposable, in red bags labeled "infectious waste." b. Non-disposable items should be cleaned with a hospital approved disinfectant and sent to Central Services for sterilization. 7. Care should be taken to avoid needle-stick injuries. Used needles should not be recapped or bent; they should be placed in a prominently labeled puncture resistant container designated specially for such disposal. One may prefer to administer all drugs, via stopcock, to avoid recapping needles. 8. Blood spills should be cleaned up promptly with a solution of 5.25% sodiumhypochforite diluted with Water (1:10 household bleach). 9. Health care workers who have exudative lesions or weeping dermatitis should refrain from all patient care and handling equipment until the condition is resolved. 10. To minimize the need for emergency mouth-to-mouth resuscitation, disposable mouth pieces, resuscitation bags or other ventilation devices should be available for use, 11. Pregnant health care workers should strictly adhere to precautions to minimize the risk of HIV transmission. 12. All specimens must be contained in a leak-proof plastic bag labeled 'bio-hazard." d) Nipple discharge.

Discharges from the nipple:  Discharge from the surface o Paget’s disease o Skin diseases (eczema, psoriasis) o Rare causes (e.g. chancre)  Discharge from a single duct o Blood-stained  Intraduct papilloma  Intraduct carcinoma  Duct ectasia o Serous (any colour)  Fibrocystic disease  Duct ectasia  Carcinoma  Discharge from more than one duct o Blood-stained  Carcinoma  Ectasia  Fibrocystic disease o Black or green  Duct ectasia o Purulent  Infection o Serous  Fibrocystic disease  Duct ectasia  Carcinoma o Milk: o Lactation o Rare causes (hypothyroidism, pituitary tumour)  Discharge can occur from one or more lactiferous ducts.  Management depends on the presence of a lump (which should always be given priority in diagnosis and treatment) and the presence of blood in the discharge or discharge from a single duct.  Mammography is rarely useful except to exclude an underlying impalpable mass. Cytology may reveal malignant cells but a negative result does not exclude a carcinoma or in situ disease.  A clear, serous discharge may be ‘physiological’ in a parous woman or may be associated with a duct papilloma or mammary dysplasia.  Multiduct, multicoloured discharge is physiological and the patient may be reassured.  A blood-stained discharge may be caused by duct ectasia, a duct papilloma or carcinoma. A duct papilloma is usually single and situated in one of the larger lactiferous ducts; it is sometimes associated with a cystic swelling beneath the areola.  A black or green discharge is usually the result of duct ectasia and its complications. Treatment:  Treatment must first be to exclude a carcinoma by occult blood test and cytology.  Simple reassurance may then be sufficient but, if the discharge is proving intolerable, an operation to remove the affected duct or ducts can be performed (microdochectomy).  Microdochectomy: o It is important not to express the blood before the operation as it may then be difficult to identify the duct in theatre. o A lacrimal probe or length of stiff nylon suture is inserted into the duct from which the discharge is emerging. o A tennis racquet incision can be made to encompass the entire duct or a periareolar incision used and the nipple flap dissected to reach the duct. o The duct is then excised. A papilloma is nearly always situated within 4–5 cm of the nipple orifice. o Ductoscopy (inspection of the internal structure of the duct system) using microendoscopes is technically feasible but generally disappointing. o The affected duct may not be visualised and biopsy systems are currently rudimentary.  Cone excision of the major ducts (after Hadfield)(subareolar resection): o When the duct of origin of nipple bleeding is uncertain or when there is bleeding or discharge from multiple ducts, the entire major duct system can be excised for histological examination without sacrifice of the breast form. o A periareolar incision is made and a cone of tissue is removed with its apex just deep to the surface of the nipple and its base on the pectoral fascia. o The resulting defect may be obliterated by a series of purse-string sutures although a temporary suction drain will reduce the chance of long-term deformity. o It is vital to warn the patient that she will be unable to breastfeed after this and may experience altered nipple sensation. Group – D

4. Write briefly on (any five): 5x4 = 20

a) Frozen shoulder. b) Complications of supracondylar fracture of humerus. c) Sequestrum. d) Volkman’s contracture. e) Talipes equines. f) Bladder problem in spinal paraplegia.

Answer. a) Frozen shoulder. See the Question 4.b of Group – D of supplementary Paper – I of 2014. b) Complications of supracondylar fracture of humerus. See the Question 4.c of Group – D of Supplementary Paper – I of 2009. c) Sequestrum. See the Question 4.d of Group – D of Supplementary Paper – I of 2012. d) Volkman’s contracture. See the Question 4.a of Group – D of Paper – I of 2013. e) Talipes equines. See the Question 4.e of Group – D of Supplementary Paper – I of 2011. f) Bladder problem in spinal paraplegia.

Neural control of bladder:

 Vesico sympathetic reflex– relaxes the detrusor muscle, and tightens the urethra, it has the inhibitory control on the parasympathetic transmission.  Vesico parasympathetic reflex– contracts detrusor and relaxes the urethra.  Vesicopudendal reflex—motor cells of S3 level, controls the striated muscles of the pelvic floor and the external sphincter and is inhibited during the micturition.  Spinal Cord Lesion:  Automatic bladder: o Also result in Spastic bladder or overactive bladder. o Paraplegic or quadriplegic have lower extremity spasticity. o Initially, after spinal cord trauma, the individual enters a spinal shock phase. o After 6-12 weeks, the nervous system reactivates, it causes hyperstimulation of the affected organs o These people experience urge incontinence. o The bladder empties too quickly and too frequently. o The voiding disorder is similar to that of the brain lesion except that the external sphincter may have paradoxical contractions as well. o If both the bladder and external sphincter become spastic at the same time, the affected individual will sense an overwhelming desire to urinate but only a small amount of urine may dribble out. o The term for this is Detrusor-sphincter Dyssynergia because the bladder and the external sphincter are not in synergy. o Even though the bladder is trying to force out urine, the external sphincter is tightening to prevent urine from leaving.  Autonomous bladder: o If a sensory neurogenic bladder is present, the affected individual may not be able to sense when the bladder is full. o In the case of a motor neurogenic bladder, the individual will sense the bladder is full and the detrusor may not contract, a condition known as Detrusor Areflexia. o These individuals have difficulty eliminating urine and experience overflow incontinence; the bladder gradually overdistends until the urine spills out. o Typical causes are a sacral cord tumor, herniated disc, and injuries that crush the pelvis. o This condition also may occur after a lumbar laminectomy.

. Neurogenic bladder is a malfunctioning bladder due to any type of neurologic disorder. . Detrusor hyperreflexia refers to overactive bladder symptoms due to a suprapontine upper motor neuron neurologic disorder. External sphincter functions normally. The detrusor muscle and the external sphincter function in synergy (in coordination). . DSD-DH refers to overactive bladder symptoms due to neurologic upper motor neuron disorder of the suprasacral spinal cord. . Paradoxically, the patient is in urinary retention. Both the detrusor and the sphincter are contracting at the same time; they are in dyssynergy (lack of coordination). . Overactive bladder refers to symptoms of urinary urgency, with or without urge incontinence, usually associated with frequency and nocturia. The cause may be neurologic or nonneurologic. . Detrusor areflexia is complete inability of the detrusor to empty due to a lower motor neuron lesion (eg, sacral cord or peripheral nerves).